Download SSC CGL Tier I Last 5 Year Papers e Book Www.sscportal.in (1)

97

description

hihihoichihr

Transcript of Download SSC CGL Tier I Last 5 Year Papers e Book Www.sscportal.in (1)

Page 1: Download SSC CGL Tier I Last 5 Year Papers e Book Www.sscportal.in (1)
Page 2: Download SSC CGL Tier I Last 5 Year Papers e Book Www.sscportal.in (1)

SOLVED PAPERSSC COMBINED GRADUATE LEVEL

TIER-I EXAMHeld On: 19-06-2011 (1st Sitting)

REASONINGDirections (1-9): In each of thefollowing questions select the relatedletters/word/number from the givenalternatives.

1. CFIL : ORUX :: GDJM : ?(a) HJLN (b) NQST(c) PSVY (d) RTVX

2. BEHK : YVSP :: DGJM : ?(a) JGDA (b) ROLI(c) WTQN (d) ZWTQ

3. 24 : 60 : : 120 : ?(a) 160 (b) 220(c) 300 (d) 108

4. 392 : 28 : : 722 : ?(a) 18 (b) 28(c) 38 (d) 48

5. 123 : 36 : : 221 : ?(a) 52 (b) 69(c) 72 (d) 25

6. Timid : Ass : : Cunning : ?(a) Ant (b) Fox(c) Rabbit (d) Horse

7. Ecstasy : Gloom : : ?(a) Congratulations : Occasion(b) Diligent : Successful(c) Measure : Scale(d) Humiliation : Exaltation

8. Architect : Building : : Sculptor : ?(a) Museum (b) Stone(c) Chisel (d) Statue

9. MKQO : LNPR : : ? XVTZ(a) YSUW (b) SVWY(c) VTWY (d) WYTS

Directions (10-15): In each of thefollowing questions, find the oddnumber/letters/word from the givenalternatives.

10. (a) PQXZ (b) BCQN(c) ABDF (d) MNPR

11. (a) ABYZ (b) CDWX(c) EFUV (d) GHTV

12. (a) 626 (b) 841(c) 962 (d) 1090

13. (a) 5720 (b) 6710(c) 2640 (d) 4270

14. (a) Swimming(b) Sailing(c) Dividing(d) Driving

15. (a) Perception(b) Discernment(c) Penetration(d) Insinuation

Directions: In the following questionfrom among the given alternative selectthe one in which the set of numbers ismost like the set of number given in thequestions.

Give Set: (4, 25, 81)16. (a) (4, 36, 79)

(b) (9, 48, 81)(c) (16, 64, 100)(d) (9, 49, 143)

17. In the following question, numberof letters are skipped in betweenby a particular rule. Which of thefollowing series observes therule?(a) BAFHTU(b) ACEGJL(c) ACFJOU (d) ADFHJL

Directions: In the following question,which one of the given response wouldbe a meaningful order of the following?

18. (1) Ocean (2) Rivulet(3) Sea (4) Glacier(5) River

(a) 5,2,3,1,4 (b) 4,2,5,3,1(c) 5,2,3,4,1 (d) 4,2,1,3,5

Directions: Arrange the following wordsas per order in the dictionary.

19. (1) Preposition(2) Preperatively(3) Preposterous(4) Preponderate(5) Prepossess

(a) 2,4,1,5,3 (b) 1,5,2,4,3(c) 5,4,2,3,1 (d) 4,2,5,1,3

20. Which one of set of letters whensequentially placed at the gaps inthe given letter series shallcomplete it?Ac__cab__baca__aba__aca__

Page 3: Download SSC CGL Tier I Last 5 Year Papers e Book Www.sscportal.in (1)

1.2: Solved Paper 19-06-2011 (1st Sitting)

Join Online Coaching for SSC Exams:http://sscportal.in/community/courses

Buy Printed Study Material for SSC Exams:http://sscportal.in/community/study-kit

(a) acbcc (b) aacbc(c) babbb (d) bcbba

Directions (21-24): In each of thefollowing questions, a series is given,with one/two term(s) missing. Choose thecorrect alternative from the given onesthat will complete the series.

21. _?_DREQ, GUHT, JXKW(a) EFRS (b) TGSF(c) JWVI (d) AOBN

22. 56, 90, 132, 184, 248, __?__(a) 368 (b) 316(c) 362 (d) 326

23. 0, 4, 8, 24, 64, 176, __?__(a) 180 (b) 480(c) 280 (d) 300

24. WTPMIFB __?__ __?__(a) ZV (b) XU(c) YU (d) YV

25. Mrs. Sushella celebrated herwedding anniversary on Tuesday,30th September 1997. When willshe celebrate her next weddinganniversary on the same day?(a) 30 September 2003(b) 30 September 2004(c) 30 September 2002(d) 30 October 2003

26. A clock gains five minutes everyhour. What will be the angletraversed by the second hand inone minute?(a) 360o (b) 360.5o

(c) 390o (d) 380o

27. B is twice as odd as A but twiceyounger than F.C is half the age ofA but is twice older than D. Whois the second oldest?(a) B (b) F(c) D (d) C

28. A two member committeecomprising of one male and onefemale member is to be constitutedout of five males and threefemales. Amongst the females,Ms. A refuses to be a member ofthe committee in which Mr. B is

taken as the member. In how manydifferent ways can the committeebe constituted?(a) 11 (b) 12(c) 13 (d) 14

29. Find the wrong number in theseries.6, 12, 21, 32, 45, 60(a) 6 (b) 12(c) 21 (d) 32

30. Ramesh ranks 13th in a class of 33students. There are 5 studentsbelow Suresh rankwise. How manystudents are there betweenRamesh and Suresh?(a) 12 (b) 14(c) 15 (d) 16

31. Naresh’s age is 4 years 4 yearsless than twice the age of hisbrother. Which of the followingrepresents the equation to find hisage?(a) 2x + 4 (b) 4x + 2(c) x – 4 (d) 2x – 4

32. Suresh’s sister is the wife of Ram.Ram is Rani’s brother. Ram’s fatheris Madhur. Sheetal is Ram’sgrandmother. Rema is Sheetal’sdaughter-in-law. Rema is Sheetal’sdaughter-in-law. Rohit is Rani’sbrother ’s son. Who is Rohit toSuresh?(a) Brother-in-law(b) Son(c) Brother(d) Nephew

33. In a survey, 70% of thosesurveyed owned a car and 75% ofthose surveyed owned a TV. If55% owned both a car and a TV,what percent of those surveyeddid not own either a car on a TV?(a) 25% (b) 20%(c) 10% (d) 5%

34. If DANCE is coded as GXQZHthen how will RIGHT be coded?(a) UFJEW (b) SGKFX(c) UFJWE (d) UFWJE

35. If LOSE is coded as 1357 and

GAIN is coded as 2468, what dothe figures 84615 stand for?(a) NAILS (b) SNAIL(c) LANES (d) SLAIN

36. A statement is given followed byfour alternative arguments. Selectthe alternative which is mostappropriate.Statements:Is it necessary that educationshould be job oriented?Arguments:

I. Yes, the aim of education is toprepare person for earning.

II. Yes, educated person shouldstand on his own feet aftercompletion of education.

III. No, education should be for sakeof knowledge only.

IV. No, one may take up agriculturewhere education is not necessary.(a) Only I and II arguments are

strong(b) Only III and IV argument are

strong(c) Only I argument is strong(d) Only I and III arguments are

strong37. A man started walking West. He

turned right, then right again andfinally turned left. Towards whichdirection was he walking now?(a) North (b) South(c) West (d) East

38. Sudha travels 8 km to the South.Then she turns to the right andwalks 4 km. Then again the turnsto her r ight and moves 8 kmforward. How many km away isshe from the starting point?(a) 7 (b) 6 (c) 4 (d) 8

39. From the given alternative words,select the word which cannot beformed using the letters of thegiven words:‘COMPREHENSION’(a) MUTUAL(b) LIMITATION(c) ONION (d) PREACH

Page 4: Download SSC CGL Tier I Last 5 Year Papers e Book Www.sscportal.in (1)

1.3: Solved Paper 19-06-2011 (1st Sitting)

Join Online Coaching for SSC Exams:http://sscportal.in/community/courses

Buy Printed Study Material for SSC Exams:http://sscportal.in/community/study-kit

40. From the given alternative words,select the word which can beformed using the letters of thegiven word:‘MULTIPLICATION’(a) MUTUAL(b) LIMITATION(c) APPLICATION (d)

NOTIONDirections (41-42): In each of thefollowing questions, select the missingnumber from the given response.

41.

6 5

125 ?

(a) 127 (b) 31(c) 217 (d) 328

42.

2 1 221 22 ?1 2 5

20 23 43

(a) 40 (b) 48(c) 50 (d) 36

43. If ‘–‘ stands for division ‘+’ standsfor subtraction, ‘÷’ stands formultiplication, ‘×’ stands foraddition, then which one of thefollowing equations is correct?(a) 70 – 2 + 4 ÷ 5 × 6 = 44(b) 70 – 2 + 4 ÷ 5 × 6 = 21

(c) 70 – 2 + 4 ÷ 5 × 6 = 341(d) 70 – 2 + 4 ÷ 5 × 6 = 36

44. The school result is increasingyear after year. Forecast the resultof 2012 from the followinginformation.

Year 2006 2007 2008 2009 20102011 2012Result 34.25% 36.75% 39.75% 43.25% 47.25%

51.75% ?

(a) 51.75%(b) 56.75%(c) 48.00%(d) 57.25%

45. Which diagram correctlyrepresents the relationshipbetween politicians, poets andwomen?

(a) (b) (d)(c)

46. How many triangles are there inthe following figures?

(a) 29 (b) 27(c) 23 (d) 30

47. Some equations are solved on thebasis of a certain system. Find the

correct answer for the unsolvedequation on that basis.If 324 × 289 = 35, 441 × 484 = 43,625 × 400 = 45, find the value of256 × 729.(a) 33 (b) 35 (c) 43 (d) 34

Directions: From the given answerfigures, select the one in which thequestion figures is hidden/embedded.

48. Question Figure:

Answer figures:

(a) (b) (d)(c)

Directions: If a mirror is placed on theline MN, then which of the answerfigures is the correct image of the givenquestion figure?

49. Question Figure:N

MAnswer figures:

(a) (b) (d)(c)

51. In the budget for 2011-2012, thefiscal deficit (% of GDP) for 2011-2012 has been projected at(a) 5.1 (b) 5.0(c) 4.6 (d) 3.4

52. Which was described by Dr. B.R.Ambedkar as the ‘heart and soul’of the Constitution?(a) Right to Equality

GENERAL AWARENESS(b) Right against Exploitation(c) Right to Constitutional

Remedies(d) Rights to Freedom of Religion

53. Which is not the concern of thelocal government?(a) Public Health(b) Sanitation

(c) Law and order(d) Public Utility Services

54. In India, the concept of singlecitizenship is adopted from(a) England (b) U.S.A.(c) Canada (d) France

55. Who had proposed partylessdemocracy in India?

Page 5: Download SSC CGL Tier I Last 5 Year Papers e Book Www.sscportal.in (1)

1.4: Solved Paper 19-06-2011 (1st Sitting)

Join Online Coaching for SSC Exams:http://sscportal.in/community/courses

Buy Printed Study Material for SSC Exams:http://sscportal.in/community/study-kit

(a) Jaya Prakash Narayan(b) Mahatma Gandhi(c) Vinoba Bhave(d) S.A. Dange

56. Disinvestment in Public Sector iscalled(a) Liberalisation(b) Globalisation(c) Industrialisation(d) Privatisation

57. ‘Darwin finches’ refers to a groupof(a) Fishes(b) Lizards(c) Birds (d) Amphibians

58. An individual’s actual standard ofliving can be assessed by(a) Gross National Income(b) Net National Income(c) Per Capita Income(d) Disposable Personal Income

59. As announced by the FinanceMinister in his Budget speech on28th February 2011, with a view toassist States in modernising theirstamp and registrationadministration, E-stamping isexpected to be rolled out in all thedistricts of different States in aperiod of(a) one year (b) two year(c) three year(d) five year

60. Which Sikh Guru called himself the‘Sachcha Badshah’?(a) Guru Gobind Singh(b) Guru Hargoving(c) Guru Tegh Bahadur(d) Guru Arjun Dev

61. The Civil DisobedienceMovement was launched byMahatma Gandhi in(a) 1928 (b) 1930(c) 1931 (d) 1922

62. What is the maximum strengthprescribed for State LegislativeAssemblies?(a) 350 (b) 600(c) 500 (d) 750

63. Fa-hien visited India during theregion of(a) Chandragupta II(b) Samundragupta(c) Ramagupta(d) Kumaragupta

64. At which place in Bengal was theEast India Company givenpermission to trade and build afactory by the Mughals in 1651?(a) Calcutta(b) Cassim Bazar(c) Singur(d) Burdwan

65. Diu is an island off(a) Daman(b) Goa(c) Gujarat(d) Maharashtra

66. Which from the following is aland-locked sea?(a) Red Sea (b) Timor Sea(c) North Sea(d) Aral Sea

67. Shivasamudram Falls is found inthe course of river(a) Krishna (b) Godavari(c) Cauvery (d) Mahanandi

68. Who gave the slogan “InquilabZindabad”?(a) Chandrashekhar Azad(b) Subhash Chandra Bose(c) Bhagat Singh(d) Iqbal

69. The position of India Railwaysnetwork in the world is(a) second (b) third(c) fourth (d) fifth

70. The oldest international airline is(a) Dutch. KLM(b) Air Canada(c) Qantas Airways(d) Air Sahara

71. Which of the following can beused to absorb neutrons tocontrol the chain reaction duringnuclear fission?(a) Boron (b) Heavy water

(c) Uranium(d) Plutonium

72. Hydrogen bomb is based on theprinciple of(a) controlled fission reaction(b) uncontrolled fission reaction(c) controlled fusion reaction(d) uncontrolled fusion reaction

73. Troposphere is the hottest part ofthe atmosphere because(a) it is closest to the Sun(b) there are charged particles in

it(c) it is heated by the Earth’s

surface(d) heat is generated in it

74. The outermost layer of the Sun iscalled(a) Chromosphere(b) Photosphere(c) Corona(d) Lithosphere

75. Where was India’s computerinstalled?(a) Indian Institute of

Technology, Delhi(b) Indian Institute of Science,

Bangalore(c) Indian Iron & Steel Co. Ltd.,

Burnpur(d) Indian Statistical Institute,

Calcutta76. The dried flower buds are used as

a spice in(a) Cardamom(b)Cinnamon(c) Cloves (d) Saffron

77. The tissue in man where no celldivision occurs after birth is(a) skeletal (b) nerves(c) connective(d) germinal

78. DNA fingerprinting is used toidentify the(a) Parents (b) Rapist(c) Thieves (d) All the above

79. The normal cholesterol level inhuman blood is(a) 80 – 120 mg%(b) 120 – 140 mg%

Page 6: Download SSC CGL Tier I Last 5 Year Papers e Book Www.sscportal.in (1)

1.5: Solved Paper 19-06-2011 (1st Sitting)

Join Online Coaching for SSC Exams:http://sscportal.in/community/courses

Buy Printed Study Material for SSC Exams:http://sscportal.in/community/study-kit

(c) 140 – 180 mg%(d) 180 – 200 mg%

80. Which of the following are warm-blooded animals?(a) Whales(b) Whale Sharks(c) Alytes(d) Draco

81. Syrinx is the voice box in(a) Amphibians(b) Reptiles(c) Birds (d) Mammals

82. The largest ecosystem of the Earthis(a) Biome(b) Hydrosphere(c) Lithosphere(d) Biosphere

83. In a food chain, the solar energyutilized by plants is only(a) 10 per cent(b) 1 per cent(c) 0.1 per cent(d) 0.01 per cent

84. Supersonic jet causes pollution bythinning of(a) O3 layer (b) O2 layer(c) SO2 layer(d) CO2 layer

85. OTEC stand for(a) Ocean Thermal Energy

Conservation(b) Oil and Thermal Energy

Conservation(c) Ocean Thermal Energy

Conversion(d) Ocean Thermal Energy

Conversion86. As per Railway Budget for 2011-

2012, the senior citizen concessionfor men has been increased from(a) 25 % to 30%(b) 30 % to 40%(c) 40 % to 50%(d) 30 % to 50%

87. The monster earthquake-cum-tsunami which his Japan onMarch, 11, 2011 has moved the

country’s main island, Honshu, byabout(a) two feet(b) three feet(c) five feet(d) eight feet

88. What is the commonly used unitfor measuring the speed of datatransmission?(a) Mega Hertz(b) Character per second(c) Bits per second(d) Nano seconds

89. Which type of reaction producesthe most harmful radiation?(a) Fusion reaction(b) Fission reaction(c) Chemical reaction(d) Photo-Chemical reaction

90. Which type of glass is used formaking glass reinforced plastic?(a) Pyrex glass(b) Flint glass(c) Quartz glass(d) Fibre glass

91. Which of the following metalscauses Itai-Itai disease?(a) Cadmium(b) Chromium(c) Cobalt(d) Copper

92. Glycol is added to aviationgasoline because it(a) reduces evaporation of petrol(b) increases efficiency of petrol(c) prevents freezing of petrol(d) reduces consumption of petrol

93. Who invented penicillin?(a) Alexander Fleming(b) Louis Pasteur(c) Dresser(d) Edward Jenner

94. 2018 FIFA World Cup would beheld in(a) Russia(b) Qatar(c) France(d) Netherlands

95. Who of the following was not therecipient of Nobel Prize inEconomics 2010?(a) Peter A. Diamond(b) Christopher Pissarides(c) Lui Xiaobo(d) Dale Mortensen

96. Which troubled model wasrecalled recently by the Indianautomobile giant Maruti Suzuki forreplacement of faulty fuel pumpgasket?(a) Maruti 800(b) Maruti A-Star(c) Wagon R(d) Baleno

97. As a part of Human ResourceDevelopment Ministry’s ‘braingain’ policy to attract global talent,14 world class universities toattract global talents are proposedto be set up in various states.Name the state from the followingwhere no university of theaforesaid type is proposed to belocated.(a) Punjab(b) Bihar(c) Rajasthan(d) Uttaranchal

98. Who discovered South Pole?(a) Robert Peary(b) Amundsen(c) John Cabot(d) Tasman

99. Which of the following folkdances is associated withRajasthan?(a) Rauf(b) Jhora(c) Veedhi(d) Suisini

100. Which of the following books isauthored by V.S. Naipalu?(a) The Rediscovery of India(b) A House for Mr. Biswas(c) Witness the Night(d) Tender Hooks

Page 7: Download SSC CGL Tier I Last 5 Year Papers e Book Www.sscportal.in (1)

1.6: Solved Paper 19-06-2011 (1st Sitting)

Join Online Coaching for SSC Exams:http://sscportal.in/community/courses

Buy Printed Study Material for SSC Exams:http://sscportal.in/community/study-kit

101. If p = 124, 23 ( 3 3) 1p p p+ + + =?(a) 5 (b) 7(c) 123 (d) 125

102. If 3 3

1100 5x

- = , then x equals

(a) 2 (b) 4(c) 16 (d) (136)1/3

103. I multiplied a natural number by 18and another by 21 and added theproducts. Which one of thefollowing could be the sum?(a) 2007(b) 2008(c) 2006(d) 2002

104. The product of two numbers is 45and their difference is 4. The sumof squares of the two numbers is(a) 135 (b) 240(c) 73 (d) 106

105. 8 57 38 108 169+ + + +

= ?(a) 4 (b) 6(c) 8 (d) 10

106. The square root of 14 + 6 5 is

(a) 2 5+ (b) 3 5+

(c) 5 3+ (d) 3 2 5+107. When 231 is divided by 5 the

remainder is(a) 4 (b) 3(c) 2 (d) 1

108. The value of

11 11 1

1 11 2

13

++

++

+

(a)2113 (b)

173

(c)3421 (d)

85

109. The unit digit in the product(122)173 is(a) 2 (b) 4(c) 6 (d) 8

110. The value of

2 3 2 3 3 12 3 2 3 3 1+ - +

+ +- + -

is

(a) 16 3+ (b) 4 3-

(c) 2 3- (d) 2 3+111. If a * b = 2a + 3b – ab, then the

value of (3 * 5 + 5 * 3) is(a) 10 (b) 6(c) 4 (d) 2

112. Simplify:3

2 2

0.0347 0.0347 0.0347 (0.9653)(0.0347) (0.347) (0.09653)

× × +- +

(a) 0.9306 (b) 1.0009(c) 1.0050 (d) 15

113. A copper wire is bent in the formof an equilateral triangle, and hasare 121 3 cm2. If the same wire isbent into the form of a circle, thearea (in cm2) enclosed by the wire

in (Take 227

π= )

(a) 364.5 (b) 693.5(c) 346.5 (d) 639.5

114. A child reshapes a cone made upof clay of height 24 and radius 6cminto a sphere. The radius (in cm) ofeh sphere is(a) 6 (b) 12(c) 24 (d) 48

115. Water flows into a tank which is200m long and 150m wide, througha pipe of cross-section 0.3m ×

0.2m at 20 km/hour. Then the time(in hour) for the water level in thetank to reach 8cm is(a) 50 (b) 120(c) 150 (d) 200

116. The product of two numbers is2028 and their H.C.F. is 13. Thenumber of such pairs is(a) 1 (b) 2(c) 3 (d) 4

117. Two equal vessels are filled withthe mixture of water and milk in theratio of 3:4 and 5:3 respectively. Ifthe mixtures are poured into athird vessel, the ratio of water andmilk in the third vessel will be(a) 15 : 12 (b) 53 : 59(c) 20 : 9 (d) 59 : 53

118. I am three times as old as my son.15 years hence, I will by twice asold as my son. The sum of ourages is(a) 48 years (b) 60 years(c) 64 years (d) 72 years

119. Three bells ring simultaneously at11 a.m. They ring at regularintervals of 20 minutes, 30minutes, 40 minutes respectively.The time when all the three ringtogether next is(a) 2 p.m. (b) 1 p.m.(c) 1.15 p.m. (d) 1.30 p.m.

120. A and B together can do a worksin 12 days. B and C together do itin 15 days. If A’s efficiency istwice that of C, then the daysrequired for B alone to finish thework is(a) 60 (b) 30(c) 15 (d) 15

121. A and B can do a work in 12 days,B and C can do the same work in15 days, C and A can do the sametime work in 20 days. The timetaken by A, B and C to do thesame work is

NUMERICAL APTITUDE

Page 8: Download SSC CGL Tier I Last 5 Year Papers e Book Www.sscportal.in (1)

1.7: Solved Paper 19-06-2011 (1st Sitting)

Join Online Coaching for SSC Exams:http://sscportal.in/community/courses

Buy Printed Study Material for SSC Exams:http://sscportal.in/community/study-kit

(a) 5 days (b) 10 days(c) 15 days (d) 20 days

122. A is 50% as efficient as B. C doeshalf of the work done by A and Btogether. If C alone does the workin 20 days, then A, B and Ctogether can do the work in

(a)2

53 days (b)

26

3 days

(c) 6 days (d) 7 days123. The ratio of the volumes of water

and glycerine in 240cc of a mixtureis 1 : 3. The quantity of water (incc) that should be added to themixture so that the new ratio of thevolume of water and glycerinebecomes 2:3 is(a) 55 (b) 60(c) 62.5 (d) 64

124. At present, the ratio of the ages ofMaya and Chhaya is 6:5 andfifteen years from now, the ratiowill get changed to 9:8. Maya’spresent age is(a) 21 years (b) 24 years(c) 30 years (d) 40 years

125. The ratio of the income to theexpenditure of a family is 10:7. Ifthe family’s expenses are Rs.10,500, then savings of the familyis(a) Rs. 4,500 (b) Rs. 10,000(c) Rs. 4,000 (d) Rs. 5,000

126. The average mathematics marksof two Sections A and B of ClassIX in the annual examination is 74.The average marks of Section A is77.5 and that of Section B is 70.The ratio of the number ofstudents of Section A and B(a) 7 : 8 (b) 7 : 5(c) 8 : 7 (d) 8 : 5

127. The average weight of a group of20 boys was calculated to be 89.4kg and it was later discovered thatone weight was misread as 78kginstead of 87 kg. The correctaverage weight is

(a) 88.95 kg (b) 89.25 kg(c) 89.55 kg (d) 86.85 Kg

128. The diameter of a wheel is 98 cm.The number of revolutions inwhich it will have to cover adistance of 1540 m is(a) 500 (b) 600(c) 700 (d) 800

129. In an equilateral triangle ABC ofside 10cm, the side BC is trisectedat D. Then the length (in cm) ofAD is(a) 3 7 (b) 7 3

(c)10 7

3(d)

7 103

130. The cost price of an article is Rs.800. After allowing a discount of10%, a gain of 12.5% was made.Then the marked price of the articleis(a) Rs. 1,000(b) Rs. 1,100(c) Rs. 1,200(d) Rs. 1,300

131. A man bought an article listed atRs. 500 with a discount of 20%offered on the list price. Whatadditional discount must beoffered to man to bring the netprice to Rs. 1,104?(a) 8% (b) 10%(c) 12% (d) 15%

132. If a c eb d f

= = = 3,

then 2 2 2

2 2 2

2 3 42 3 4

a c eb d f

+ ++ + = ?

(a) 2 (b) 3(c) 4 (d) 9

133. The floor of a room is of size 4m ×3m and its height is 3m. The wallsand ceiling of the room requirepainting. The area to be painted is(a) 66 m2 (b) 54 m2

(c) 43 m2 (d) 33 m2

134. When the price of an article wasreduced by 20% its sale increased

by 80%. What was the net effecton the sale?(a) 44% increase(b) 44% decrease(c) 66% increase(d) 66% decrease

135. The price of sugar goes up by20%. If a housewife wants theexpenses on sugar to remain thesame, she should reduce theconsumption by

(a)1

155 % (b)

213

3 %

(c) 20% (d) 25%136. In a factory 60% of the workers

are above 30 years and of these75% are males and the rest arefemales. If there are 1350 maleworkers above 30 years, the totalnumber of workers in the factory(a) 3000 (b) 2000(c) 1800 (d) 1500

137. Walking at 34 of his usual speed, a

main is 1

12 hours late. His usual

time to cover to same distance, inhours, is

(a)1

42 (b) 4

(c)1

52 (d) 5

138. The selling price of 10 oranges isthe cost price of 13 oranges. Thenthe profit percentage is(a) 30% (b) 10%(c) 13% (d) 3%

139. The marked price of a ratio is Rs.480. The shopkeeper allows adiscount at 10% and gains 8%. Ifno discount is allowed, his gainpercent would be(a) 18% (b) 18.5%(c) 20.5% (d) 20%

140. A man sold 20 apples for Rs. 1000

Page 9: Download SSC CGL Tier I Last 5 Year Papers e Book Www.sscportal.in (1)

1.8: Solved Paper 19-06-2011 (1st Sitting)

Join Online Coaching for SSC Exams:http://sscportal.in/community/courses

Buy Printed Study Material for SSC Exams:http://sscportal.in/community/study-kit

and gained 20%. How many applesdid he buy for Rs. 100?(a) 20 (b) 22(c) 24 (d) 25

141. A rectangular sheet of metal is 40cm by 15 cm. Equal squares ofside 4 cm are cut off at the cornersand the remainder is folded up toform an open rectangular box. Thevolume of the box is(a) 896 cm3 (b) 986 cm3

(c) 600 cm3 (d) 916 cm3

142. If 78 is divided into three parts

which are in the ratio 1: 1 1

:3 6 , the

middle part is

(a)1

93 (b) 13

(c)1

173 (d)

118

3143. The simple interest on a sum of

money is 19 of the principle and

the number of years is equal torate per cent annum. The rate perannum is

(a) 3% (b)13 %

(c)1

33 % (d)

310 %

144. The difference between simpleinterest and compound interest ofa certain sum of money at 20% perannum for 2 years is Rs. 48. Thenthe sum is(a) Rs. 1,000 (b) Rs. 1,200(c) Rs. 1,500 (d) Rs. 2,000

145. Shri X goes to his office byscooter at a speed of 30 km/h andreaches 6 minutes earlier. If hegoes at a speed of 24 km/h, hereaches 5 minutes late. Thedistance of his office is(a) 20 km (b) 21 km(c) 22 km (d) 24 km

146. A sum of money becomes eighttimes in 3 years, if the rate iscompounded annually. In howmuch time will the same amount atthe same compound rate becomesixteen times?(a) 6 years (b) 4 years(c) 8 years (d) 5 years

Directions (147-150): The pie chartgiven below show the spending of afamily on various heads during a month.Study the graph and answer thequestions 147 to 150.

14% Rent

45% food

6% O

thers

15% Education

9% Fuel

11% Clothing

147. If the total income of the family isRs. 25,000, then the amount spenton Rent and Food together is(a) Rs. 17,250(b) Rs. 14,750(c) Rs. 11,250(d) Rs. 8,500

148. What is the ratio of the expensesof Education to the expenses onFood?(a) 1 : 3 (b) 3 : 1(c) 3 : 5 (d) 5 : 3

149. Expenditure on Rent is whatpercent of expenditure of Fuel?(a) 135% (b) 156%(c) 167% (d) 172%

150. Which three expenditures togetherhave a central angle of 108o?(a) Fuel, Clothing and Others(b) Fuel, Education and Others(c) Clothing, Rend and Others(d) Education, Rent and Others

ENGLISH COMPREHENSIONDirections (151-155): In the followingquestion, some of the sentences haveerrors and some are correct. Find outwhich part of a sentence has an error.The number of the part is you answer. Ifa sentence is free from errors, then youanswer is (d) i.e., No error.151. A senior doctor (1) / expressed

concern (2) / about physiciansrecommended the vaccine. (3) / Noerror. (4)

152. We have discussing (1) / all the

known mechanisms (2) / ofphysical growth, (3) / No error. (4)

153. Children enjoy listening to (1) /ghosts stories (2) / especially onHalloween night. (3) / No error. (4)

154. I (1) / have (2) / many works to do.(3) / No error. (4)

155. There are so many filths (1) / allaround (2) / the place. (3) / Noerror. (4)

Directions (156–160): In the followingquestions, sentences are given with

blanks to be filled in with an appropriateword. Four alternatives are suggestedfor each question. Choose the correctalternative out of the four.156. The building is not safe and must

be _____ down.(a) pull (b) pulling(c) pulled (d) pulls

157. There is something wonderful ___him.(a) of (b) about(c) for (d) inside

Page 10: Download SSC CGL Tier I Last 5 Year Papers e Book Www.sscportal.in (1)

1.9: Solved Paper 19-06-2011 (1st Sitting)

Join Online Coaching for SSC Exams:http://sscportal.in/community/courses

Buy Printed Study Material for SSC Exams:http://sscportal.in/community/study-kit

158. The song in the play cannot bedeleted it is ____ to the story.(a) intervened(b) innate(c) exacting(d) integral

159. She remained a _____ all her life.(a) spinster(b) bachelor(c) unmarried(d) single

160. Do not stay in the grasslands afterdark, as some animals become____ when they see humans.(a) provoked(b) alerted(c) aggressive(d) threatened

Directions (161-165): In the following,questions out of the four alternatives,choose one which best expresses themeaning of the given word.161. LUCIDITY

(a) Fluidity(b) Politeness(c) Clarity(d) Fluency

162. INDICT(a) Implicate(b) Elude(c) Appreciate(d) Judge

163. DELUGE(a) Confusion(b) Deception(c) Flood(d) Weapon

164. PREPONDERANCE(A) Pre-eminence(B) Dominance(C) Domineering(D) Preoccupation

Directions (166-170): In the followingquestion no. 166 to 170, out of the fouralternatives, choose the word oppositein meaning to the given word.

166. INVINCIBLE(a) Small(b) Invisible(c) Vulnerable(d)Reachable

167. INOFFENSIVE(a) Sensitive(b) Organic(c) Sensible(d) Rude

168. DIVULGE(a) Conceal(b) Disguise(c) Oppress(d) Reveal

169. DISCORD(a) Harmony(b) Serenity(c) Acceptance(d) Placidity

170. MAMMOTH(a) Quiet(b) Significant(c) Huge (d) Small

Directions (171-175): In the followingfour alternatives are given for themeaning of the given Idiom/Phrase.Choose the alternative which bestexpresses the meaning of the Idiom/Phrase.171. To take to heart

(a) to be encouraged(b) to grieve over(c) to like(d) to hate

172. Yeoman’s service(a) medical help(b) excellent work(c) social work(d) hard work

173. To face the music(a) to enjoy a musical recital(b) to bear the consequences(c) to live in a pleasant

atmosphere(d) to have a difficult time

174. To put up with(a) to accommodate

(b) to adjust(c) to understand(d) to tolerate

175. To call it a day(a) to conclude proceedings(b) to initiate proceedings(c) to work through the day(d) None of the above

Directions (176-180): In the followingquestions, a sentence or bold partthereof is given which may needimprovement. Alternative are given at(A), (B) and (C) below, which may be abetter option. In case, no improvementis needed, your answer is (D).176. The man whom I thought was

thoroughly honest proved to be aswindler.(a) The man whom I thought was

thoroughly honest proved asswindler.

(b) The man who I thought wasthoroughly honest proved tobe a swindler.

(c) The man to whom I thoughtwas thoroughly honestproved to be a swindler,

(d) No improvement177. No sooner had the dividend been

declared, the notices were sentout.(a) The company had hardly

declared the dividend till thenotices were sent for mailing.

(b) They had no sooner declaredto dividend then the noticeswere sent out.

(c) Hardly had the dividend beendeclared than the notices weresent out.

(d) No improvement.178. Riding upon his horse, the tiger

jumped at him.(a) Riding upon the tiger, the

horse jumped at him.(b) The tiger jumped at him while

he was riding upon his horse.

Page 11: Download SSC CGL Tier I Last 5 Year Papers e Book Www.sscportal.in (1)

1.10: Solved Paper 19-06-2011 (1st Sitting)

Join Online Coaching for SSC Exams:http://sscportal.in/community/courses

Buy Printed Study Material for SSC Exams:http://sscportal.in/community/study-kit

(c) The tiger rode at him while hewas jumping upon his horse.

(d) No improvement179. I in black and white must have

your terms down.(a) I must have in black in white

your terms down.(b) I must have your terms in black

and white down.(c) I must have your terms down

in black and white.(d) No improvement.

180. When we are came out of therestaurant it was half past eleven.(a) When we had come out of the

restaurant(b) After we came out of the

restaurant(c) When we have come out of the

restaurant(d) No improvement

Directions (181-185): In the followingquestions, out of the four alternatives,choose the one which can besubstituted for the given words/sentence.181. Code of diplomatic etiquette and

precedence(a) Statesmanship(b) Diplomacy(c) Hierarchy(d) Protocol

182. To renounce a high position ofauthority or control(a) Abduct(b) Abandon(c) Abort(d) Abdicate

183. Not to be moved by entreaty(a) Rigorous(b) Negligent(c) Inexorable(d) Despotic

184. An object or portion serving as asample(a) Specification(b) Spectre

(c) Spectacle(d) Specimen

185. The practice of submitting aproposal to popular vote(a) Election (b) Reference(c) Popularity(d) Referendum

Directions (186-190): In the followingquestions, groups of four words aregiven. In each group, one word iscorrectly spelt. Find the correctly speltword.

186. (a) mandatary(b) circulatory(c) temporary(d) regulatory

187. (a) convinience(b) initative(c) concessional(d) exaggerate

188. (a) diologue(b) giraffe(c) scissors(d) humourous

189. (a) assiduous(b) nefarious(c) macabre(d) loquacious

190. (a) cortege(b) damage(c) milege(d) plumege

Directions (191-200): In the followingpassage some of the words have beenleft out. First read the passage over andtry to understand what it is about. Thenfill the blanks with the help of thealternative given.

PassageScience means knowledge, but not allknowledge is science. I know from myown eyesight that our dog Chippy liespapaya; I know from a book that Akbarwas the (191) of Babar, and (192) in1605; and I know (193) the ratio that Indiadid not do well in the (194) Test matches.We can call these (195) of knowledge(196) but they are not science.

Science (197) with facts, but not withfacts which have (198) to do with eachother, like the facts about our dog, cricketand the Mughal ruler; those facts are notrelated (199), and so have nothing (200)with science. Science stars withobservation.

191. (a) grandson(b) grandfather(c) grand nephew(d) son

192. (a) gone(b) died(c) disappeared(d) absented

193 (a) on(b) in(c) since(d) from

194. (a) old(b) ordinary(c) latest(d) outdated

195. (a) pieces(b) peace

(c) whole(d) block

196. (a) lies(b) charges(c) facts(d) blame

197. (a) starts(b) stops(c) passes(d) drives

198. (a) no(b) neither(c) nor(d) nothing

199. (a) by that way(b) in any way(c) from the side(d) in addition to

200. (a) to lie(b) to speak(c) to sleep

(d) to do

Page 12: Download SSC CGL Tier I Last 5 Year Papers e Book Www.sscportal.in (1)

1.11: Solved Paper 19-06-2011 (1st Sitting)

Join Online Coaching for SSC Exams:http://sscportal.in/community/courses

Buy Printed Study Material for SSC Exams:http://sscportal.in/community/study-kit

ANSWERS1. (c) 2. (c) 3. (c) 4. (c) 5. (d) 6. (b) 7. (d) 8. (d) 9. (a) 10. (b)

11. (d) 12. (b) 13. (d) 14. (d) 15. (d) 16. (c) 17. (c) 18. (b) 19. (a) 20. (b)21. (d) 22. (d) 23. (d) 24. (c) 25. (a) 26. (b) 27. (a) 28. (d) 29. (a) 30. (b)31. (d) 32. (d) 33. (c) 34. (a) 35. (a) 36. (a) 37. (a) 38. (c) 39. (d) 40. (c)41. (c) 42. (a) 43. (b) 44. (b) 45. (d) 46. (b) 47. (c) 48. (d) 49. (b) 50. (c)51. (c) 52. (c) 53. (c) 54. (a) 55. (a) 56. (d) 57. (c) 58. (d) 59. (c) 60. (b)61. (b) 62. (c) 62. (a) 64. (b) 65. (c) 66. (d) 67. (c) 68. (c) 69. (c) 70. (a)71. (d) 72. (d) 73. (c) 74. (c) 75 (d) 76. (c) 77. (b) 78. (d) 79. (d) 80. (a)81. (c) 82. (a) 83. (c) 84. (a) 85. (d) 86. (b) 87. (d) 88. (c) 89. (b) 90. (d)91. (a) 92. (c) 93. (a) 94. (a) 95. (c) 96. (b) 97. (d) 98. (b) 99. (d) 100. (b)

101. (d) 102. (b) 103. (a) 104. (d) 105. (a) 106. (b) 107. (b) 108. (c) 109. (b) 110. (a)111. (a) 112. (d) 113. (c) 114. (a) 115. (d) 116. (b) 117. (d) 118. (b) 119. (b) 120. (c)121. (b) 122. (b) 123. (b) 124. (c) 125. (a) 126. (c) 127. (d) 128. (a) 129. (c) 130. (a)131. (a) 132. (d) 133. (b) 134. (a) 135. (b) 136. (a) 137. (a) 138. (a) 139. (d) 140. (c)141. (a) 142. (c) 143. (c) 144. (b) 145. (c) 146. (b) 147. (b) 148. (a) 149. (b) 150. (b)151. (c) 152 (a) 153. (c) 154. (c) 155. (a) 156. (c) 157. (b) 158. (d) 159. (a) 160. (c)161. (c) 162. (c) 163. (d) 164. (c) 165. (b) 166. (c) 167. (d) 168. (a) 169. (a) 170. (d)171. (b) 172. (a) 173. (b) 174. (d) 175. (d) 176. (d) 177. (a) 178. (b) 179. (c) 180. (d)181. (d) 182. (d) 183. (d) 184. (d) 185. (d) 186. (c) 187. (d) 188. (c) 189. (b) 190. (a)191. (a) 192. (b) 193. (a) 194. (c) 195. (a) 196. (c) 197. (a) 198. (d) 199. (b) 200. (d)

EXPLANATIONS

1.

C F I L O R U X

+12+12

+12+12

Similarly,

D G J M P S V Y

+12+12

+12+12

3. 24 × 2.5 = 6012 × 2.5 = 300

4. 14 × 14 × 2 = 72214 × 2 = 28Similarly,19 × 19 × 2 = 39219 × 2 = 38

5. 1 + 2 + 3 = 6; (6)2 = 362 + 2 + 1 = 5 ; (5)2 = 25

6. Ass is considered at Timid. Fox isconsidered as Cunning.

7. Ecstasy is opposite of Gloom.Similarly, Humiliation is opposite ofExamination.

8. Architect construct buildings.Similarly, Sculptor carves statue.

10.

P A X Z

B C Q N

A B D F

M N P R

+1 +7 +2

+1

+1

+1

+14

+2

+2

-3

+2

+2

12. The number 841 is a perfect square.841 = 29 × 29

13. Except the number 4270, all othernumbers are completely divisible

by 11.

572011 = 520;

671011 = 610;

264011 = 240

But, 427011

= 388.1814. Except (Driving), all other

activitiers are performed in water.15. Insinuation is different from the

other words.Insinuate (Verb) means “tosuggest sombody/something in anunpleasant and indirect way”.Insinuation (Noun) means “theaction of insinuating something”.

16. The numbers given in the Set are

Page 13: Download SSC CGL Tier I Last 5 Year Papers e Book Www.sscportal.in (1)

1.12: Solved Paper 19-06-2011 (1st Sitting)

Join Online Coaching for SSC Exams:http://sscportal.in/community/courses

Buy Printed Study Material for SSC Exams:http://sscportal.in/community/study-kit

Perfect Squares.4 = (2)2;25 = (5)2;81 = (9)2

Similarly,16 = (4)2;64 = (8)2;100 = (10)2

17.

B A F H T U

A C E G J L

A C F J O U

A D F H J L

-1 +5 +2 +12 +1

-2

-3

-4

+2

+3

+2

+2

+4

+2

+5

+5

+2

+2

+6

+2

18. Meaningful order of the givenwords:4. Glacier 2. Rivulet 5. River 3. Sea 1. Ocean

19. Arrangement of words as perdictionary2. Preparatively 4. Preponderate 1. Preposition 5. Prepossess 3. Preposterous

20. ac a c / ab a b / a c a

c / aba b / a c a c

23.

0 4 8 24 64 176 300

+4 +4 +16 +40 +112 +124

+0 +12 +24 +72 +12

24.W T P M I

F B Y U

-3 -4 -4-3

-3 -4 -3 -4

25. 30th September 1998 Wednesday

30the Semptember 1999 Thursday

30th September 2000 Saturday

Beause 2000 is a Leap Year andthere is one extra day in the monthof February.30th September 2001 Sunday30th Sepeember 2002 Monday30th Sempember 2003 TuesdayAn ordinary year has one odd day.

26. Each second-space equals 1o. Aclock gains five minutes everyhour.

It means the clock gain 5

60minutes in one minute.

5360

60× = 30

27. B = 2AF = 2BA = 2CC = 2DF > B > A > C > D

28. Total number of way in which thecommittee can be formed= 5 × 3 =15But Ms A refused to be a memberof treh committee in which Mr. B istaken as a memeber. T herefore ,the required answer= 15 – 1 = 14

29.

5 12 21 32 45 60

+7 +9 +11 +13 +15

+2 +2 +2 +2

Therefore, the number 6 is wrong.in the series.

30. The rank of Suresh = 28th

R .................. S14 Students13th 28th

31. Suppose the age of Naresh’sbrother = x years∴ Age of Naresh = 2x – 4

32. Ram is brother of Rani.Rohit is son of Ram.

Suresh is material uncle of Rohit.Therefore, Rohit is nephew ofSuresh.

33. TV 20 15 55

Car

Per cent of those surveyed whodid not own either a car or a TV= 100 – (20 + 55 + 15)= 100 – 90 = 10%

35.

L O S E G A I N

1 3 5 7 2 4 6 8

Therefore, 8 4 6 1 5

N A I L S36. Only I and II arguments are

strong. Education is meant forboth knowledge and securing ajob. Any skills is acquired to use itpurposefully.

37.

North

West East

SouthNow, he is walking towards North.

38.

Starting Point

8 km

4 km

8 km

39. There is no ‘A’ letter in thekeybord.

40. There is only one ‘U’ in thekeyword.There are one ‘A’ and one ‘P’ inthe keyword.

Page 14: Download SSC CGL Tier I Last 5 Year Papers e Book Www.sscportal.in (1)

1.13: Solved Paper 19-06-2011 (1st Sitting)

Join Online Coaching for SSC Exams:http://sscportal.in/community/courses

Buy Printed Study Material for SSC Exams:http://sscportal.in/community/study-kit

There is only one ‘O’ in thekeyword.

M U L T I P L I C

A T I O N

41. (5)3 + 1 = 125 + 1 = 126(6)3 + 1 = 216 + 1 =217

42. (21 + 1) – 2 = 22 – 2 = 20(22 + 2) – 1 = 24 – 1 = 23(? + 5) – 2 = 43⇒ ? = (43 + 2 ) – 5⇒ ? = 45 – 5 = 40

43. 70 ÷ 2 – 4 × 5 + 6 = 44⇒ 15 + 6 ≠ 4470 ÷ 2 – 4 × 5 + 6 = 21⇒ 35 – 20 + 6 = 21⇒ 41 – 20 = 21

44. 34.25 + 2.50 = 36.7536.75 + 3.00 = 39.7539.75 + 3.50 = 43.2543.25 + 4.00 = 47.2547.25 + 4.00 = 47.2547.25 + 4.50 = 51.7551.75 + 5.00 = 56.75

45. Some politicians may be poets andvice-versa.No poet can be women as womenpoet is called poetess.

Poets

Polit

icia

ns

Women

46.

A

D

F

I

B M N Q C

L

H

E

G

K J

The triangles are:Δ ABC; Δ ADE; Δ AFH;

Δ AIL;Δ DFG; Δ DIK; Δ DBO; Δ GDE;Δ EGH; Δ EGL; Δ EMC; Δ FIJΔ FBN; Δ JFG; Δ GJK; Δ KGH;Δ IBM; Δ MIJ; Δ JMN; Δ NJK;Δ KNO; Δ OKL; Δ LOC;

47. 324 = 18; 289 = 1718 + 17 = 35

441 = 21; 484 = 2221 + 22 = 43

625 = 25; 400 = 2025 + 20 = 45

256 = 16; 729 = 2716 + 27 = 43

48.

49.

50.

101. Expression

= 23 ( 3 3) 1p p p+ + +

= 3 23 3 3 1p p p+ + +

=1

3 3( 1)p +

=133( 1)p

×+

= p + 1

When p = 124,p + 1 = 124 + 1 = 125

102.3 31

100 25x

− =

Squaring both sides,3 9

1100 25x

− =

⇒3 9 25 9 16

1100 25 25 25x −

= − = =

⇒ x3 = 16 10025

× = 64

∴ x = 33 64 4 4 4= × × = 4103. Let the natural numbers be x and

y.∴Required sum = 18x + 21 y= 3(6x + 7y)Hence, the sum is divisible by 3.∴Required answer = 2007

104. Let teh numbers b x and y and x >y.∴ xy = 45∴ x2 + y2 = (x – y)2 + 2xy= (4)2 + 2 × 45 = 16 + 90= 106

105. Expression=

8 57 38 108 169+ + + +

=

88 57 38 108 13+ + + +=

8 57 38 38 121+ + + +

= 8 57 38 11+ + +

= 8 57 49+ +

= 8 57 7+ +

Page 15: Download SSC CGL Tier I Last 5 Year Papers e Book Www.sscportal.in (1)

1.14: Solved Paper 19-06-2011 (1st Sitting)

Join Online Coaching for SSC Exams:http://sscportal.in/community/courses

Buy Printed Study Material for SSC Exams:http://sscportal.in/community/study-kit

= 8 8+

= 16= 4

106. 14 + 6 5 = 14 + 2 × 3 × 5=9 + 5 + 2 × 3 × 5=(3)2 + ( 5 )2 + 2 × 3 × 5= (3 + 5 )2

∴ 14 6 5+

= ( )23 5+

= 3 + 5

107. 231 =48(2 ) + 2 = (256)4 ÷ 2

=.....6

2 = .......3

Clearly, the remainder will be 3when divided by 5.Illustration:23 ÷ 5 gives remainder = 383 ÷ 5 gives remainder = 3

108. Expression

=

11 11 11 11 3 23

++

++ +

=

11 11 11 315

++

++

=

11 11 11 5 35

++

+ +

=

11 11 11 58

++

+ =

11 11 8 5

8

++ +

=

11 8113

++ =

11

13 813

++

=13121

+ =21 13

21+

= 3421

109. 21 = 2; 22 = 4;23 = 8; 24 = 16; 25 = 32∴ Unit’s digit in the producet of(122)173

=Unit’s digit in (122)1 = 2(1 = remainder when 173 dividedby 4).

110. Expression

=2 3 2 32 3 2 3

+ −+ − + +

3 1 3 13 1 3 1

+ +×

− +

=

2 2(2 3) (2 3)(2 3)(2 3)

+ + −

− + +

( )23 1

3 1

+

=2(4 3) 3 1 2 3

4 3 2+ + +

+−

( )2 2

2 2

( ) ( )

=2

a b a b

a b

+ + −

+

Q

=14 + 2 + 3 = 16 + 3111. Given that

a * b = 2a + 3b – ab

∴ 3 * 5 + 5 * 3= (2 × 3 + 3 × 5 – 3 × 5) + (5 × 2 + 3× 3 – 5 × 3)=(6 + 15 – 15) + (10 + 9 – 15)= 6 + 4 = 10

112. Let 0.0347 = aand, 0.9653 = b

∴ Expression = 3 3

2 2

a ba ab b

+− +

= a + b= 0.0347 + 0.9653 = 1

113. Area of the equilateral triangle

=3

4 side2

= 121 3 = 23 side4

×

∴ Side2 = 121 3 4

= 121 × 4

∴ Side = 121 4×= 11 × 2 = 22 cm∴ Total length of wire = 3 × 22= 66cmIf the radius of the circly be r cm,then

2 rπ = 66

⇒ r =2 22

7r×

× = 66

⇒ r = 66 72 22

××

= 212 cm

∴Area of the circle = 2rπ

=22 21 217 2 2

× ×

= 346.5 cm2

114. Volume of the cone = 21

3r hπ

= 6 6 243π

× × × cm3

Page 16: Download SSC CGL Tier I Last 5 Year Papers e Book Www.sscportal.in (1)

1.15: Solved Paper 19-06-2011 (1st Sitting)

Join Online Coaching for SSC Exams:http://sscportal.in/community/courses

Buy Printed Study Material for SSC Exams:http://sscportal.in/community/study-kit

= Volume of the sphereIf the radius of the sphere be rcm, then

343

rπ = 6 6 243π

× × ×

⇒ r3 = 6 × 6 × 6

∴ r = 6 6 6× × = 6 cm.115. Water supplied by pipe in 1 hour

= (0.3 × 0.2 × 20 × 1000)cubic meter = 1200 cubic metre∴Total time=

Volume of water to be filled in the tank1200

=200 150 8

1200× ×

= 200 hours116. Let the number be 13x and 13y

where x and y are prime to eachother.∴ LCM = 13xy∴ Product of numbers= HCF × LCM⇒ 2028 = 13 × 13xy

⇒ xy = 2028

13 13× = 12

Pairs satisfying the condition= (1, 12), and (3, 4).

117. Let the capacity of each vessel= 1 litre∴ First Vessel

Water = 37 litre

Milk = 47 litre

Second Vessel

Water = 58 litre

Milk = 38 litre

In 2 litres of mixture,Water : milk

= 3 5 4:7 8 7 8

3+ +

=24 35

56+

: 32 21

56+

= 59 : 53118. Let the son’s present age

= x years.∴ Father’s present age= 3x yearsAfter 15 years,3x + 15 = 2(x + 15)⇒ 3x + 15 = 2x + 30⇒ x = 30 – 15 = 15∴ Sum of their present ages= x + 3x = 4x=4 × 15 = 60 years

119. LCM of 20, 30 and 40 minutes =120 minutesHence, the bells will toll togetheragain after 2 hours i.e., at 1 p.m.

120. (A + B)’s 1 day’s work

= 1

12 (i)

(B + C)’s 1 day’s work

= 1

15 (ii)

∴ Difference between A and C’s 1day’s work

=1 1 5 4 5 4 1

12 15 15 60 60− −

− = = =

⇒ x = 30∴B’s 1 day’s work

= 1 1

12 30− [From equation (i)]

=5 2 3 160 60 20−

= =

Hence, B alone will complete thework in 20 days.

121. (A + B)’s 1 day’s work = 1

12

(B + C)’s 1 day’s work = 1

15

(C + A)’s 1 days work = 120

Adding all three,2(A + B + C)’s 1 day’s work

=1 1 1

12 15 20+ +

=5 4 3

60+ +

= 12 160 5

=

⇒ (A + B + C)’s 1 day’s work

=1

10∴ Required answer = 10 days

122. If B alone complete the work in xdays, A will do the same in 2x days.∴ (A + B)’s 1 day’s work

=1 1 2 1 3

2 2x x x x+

+ = =

∴ C’s 1 day’s work = 3

4x

∴ 3

4 20x1

=

⇒ 4x = 3 × 20

⇒ x = 3 20

= 15

∴ (A + B + C)’s 1 days work

=1 3 1 1 1

2 4 30 15 20x x x1

+ + = + +

=2 4 3 9 3

60 60 20+ +

= =

Hence, all three together willcomplete the work in

20 263 3

= days.

123. In the original mixture,water = 60 ccGlycerine = 180 ccLet x cc of water be mixed.

∴ 60 2180 3

x+=

Page 17: Download SSC CGL Tier I Last 5 Year Papers e Book Www.sscportal.in (1)

1.16: Solved Paper 19-06-2011 (1st Sitting)

Join Online Coaching for SSC Exams:http://sscportal.in/community/courses

Buy Printed Study Material for SSC Exams:http://sscportal.in/community/study-kit

⇒ 180 + 3x = 360⇒ 3x = 360 – 180 = 180

∴ x = 180

3 = 60 cc

124. Let the original mixture,water = 60 60 ccLet x cc of water be mixed.

∴60 2180 3

x+=

⇒ 48x + 120 = 45x + 135⇒ 48x – 45x = 135 – 120⇒ 3x = 15⇒ x = 5∴Maya’s present age = 6x=6 × 5 = 30 years

125. Income of the family

=10 105007

× =Rs. 15000

Saving = 15000 – 10500= Rs. 4500

126. If the number of students insection A be x and that in sectionB by y, then

74 = 77.5 70x y

x y× + ×

+

⇒ 74x + 74y = 77.5x + 70y⇒ 77.5x – 74x = 74y – 70y⇒ 3.5x = 4y

⇒4 8

35 7xy

= =

127. Difference in weight= 87 – 78 = 9 kg∴ Correct average weight

= 89.4 + 920

= 89.4 + 0.45 = 89.85 kg128. Distance covered by wheel in one

revolution = circumference of thewheel

=π × diameter = 22 987

× = 308

∴ Number of revolutions

= 1540 100

308×

= 500

129.

A

B C D E

AE ⊥ BC∴ BE = EC = 5 cm, AC = 10 cm

AE = 2 210 5−

= 5100 25 75 3− = = cmDE = DC – EC

=2 510 53 3

× − = cm

∴25 25 675759 9

++ =

=700 10 79 3

= cm

130. Let the marked price of the articlebe Rs. x.

∴ 90 800 112.5100 100

x ×=

⇒910

x = 900; x =

900 109×

= Rs. 1000131. SP after a discount of 20%

=1500 80

100×

= Rs. 1200

Second discount= 1200 – 1104, = Rs. 96If the discount per cent be x% then

1200100

x× = 96

⇒ x = 96001200 = 8

132.a c rb d f

= = = 3

⇒ a = 3b; c = 3d; e = 3f

∴2 2 2

2 2 2

2 3 42 3 4

a c eb d f

+ ++ +

=2 2 2

2 2 2

2 9 3 9 4 92 3 4b d f

b d f× + × + ×

+ +

=( )2 2 2

2 2 2

9 2 3 42 3 4

b d fb d f

+

+ + = 9

133. Area of thefour walls of the room=2 × height (length × breadth)=2 × 3 (4 + 3) = 42 sq. metreArea of ceiling = 4 × 3= 12 sq. metre∴ Required area = 42 + 12= 54 sq. metre

134. Required effect

=80 2080 20

100× − − %

= (60 – 16)%= 44%Positive sign shows increase.

135. Required percentage

=20 100

100 20×

+

=20 100

100×

=503 =

2163 %

136. Let the total number of workers inthe factory be x.

∴ 60 75

100 100x × × = 13500

⇒x

=1350 100 100

60 75× ×

× = 3000

137. Time and speed are inverselyproportional.

∴ 43 of usual time –

usual time

Page 18: Download SSC CGL Tier I Last 5 Year Papers e Book Www.sscportal.in (1)

1.17: Solved Paper 19-06-2011 (1st Sitting)

Join Online Coaching for SSC Exams:http://sscportal.in/community/courses

Buy Printed Study Material for SSC Exams:http://sscportal.in/community/study-kit

= 32

⇒13 × usual time =

32

∴ Usual time = 3 3 9

2 2×

=

=142 hours

138. Let the CP of 1 orange= Re. 1∴ SP or 10 oragnes = Rs. 13

∴ Gain percent = 13 10 100

10−

×

= 30%139. If the CP of ratio be Rs. x, then

108100 of x =

480 90100

×

⇒ 108

100x ×

= 432

⇒ x = 432 100

108×

= Rs. 400

Gain per cent if no discount isallowed

= 80 100400

×

= 20%140. If the CP of 20 apples be Rs. x,

then

120100

x × = 100

⇒ x = 100 100

120×

= Rs. 2503

Q Rs. 250

3 = 20 apples

∴ Rs. 100 = 20 3 100

250× ×

= 24 apples141. Length of the box

= 40 – 2 × 4 = 32 cmBreadth of the box = 15 – 2 × 4= 7cmHeight of the box = 4 cm∴Volume of the box = 32 × 7 × 4= 896 cu. cm.

142. Ratio = 1 : 1 1:3 6

= 6 : 2 : 1Sum of the ratios = 6 + 2 + 1 = 9

∴ Middle part = 2 789

×

=52 1173 3

=

143.Simple interest 1

Principal 9=

If the annual rate of interest be r%,then

Rate = S.I. × 100

Principal × Time

⇒ r = 1 1009 r

×

⇒ r2 = 100

9

⇒ r = 100 10 13

9 3 3= = %

144. Difference of two years

= P 2

10000r

⇒ 48 = P400

10000

⇒ 48 = 25P

⇒ P = 48 × 25 = Rs. 1200145. Let the distance of office be x km.

∴ 11

24 30 60x x

− =

⇒5 4 11

120 60x x−

=

⇒11

120 60x

=

⇒ x = 11 12060

×

= 22 km.146. Let the principle be Re. 1

∴ A = PTR1+

100

⇒ 8 = 13R1+

100

⇒ 23 = 3R1+

100

⇒ 2 = 1r1+

100

⇒ 24 = 4r1+

100

∴ Time = 4 years147. Q 100% = Rs. 25000

∴ 59% = 25000 59100

×

= Rs. 14750148. Required ratio = 15 : 45

= 1 : 3149. Required percentage

= 14 1009

×

= 156150. Q360o = 100%

∴ 108O = 100 108360

×

= 30%Fuel + Education + Others= 9 + 15 + 6= 30%

Page 19: Download SSC CGL Tier I Last 5 Year Papers e Book Www.sscportal.in (1)

SOLVED PAPERSSC COMBINED GRADUATE LEVEL

TIER–I EXAMHeld On: 08-07-2012 (1st Sitting)

Directions (1-9): In each of thefollowing questions, select the relatedletters / words / numbers from the givenalternatives.

1. 9 : 80: : 100 : ?(a) 901 (b) 1009(c) 9889 (d) 9999

2. 25: 125 :: 36 : ?(a) 180 (b) 206(c) 216 (d) 318

3. 335: 21 6:: 987:?(a) 868 (b) 867(c) 872 (d) 888

4. BEGK: ADFJ :: PSVY:?(a) ROUX (b) ORUX(c) LQUT (d) LOQT

5. AZBY: CXDW :: EVFU : ?(a) GTHS (b) GHTS(c) GS.TH (d) TGSH

6. BDFH: SUWY :: CEGI : ?(l) QTWZ (b) PTVX(c) JLNP (d) TVXZ

7. food: man:: fuel:?(a) wood (b) fire(c) heat (d) smoke

8. Spiritual: Belief:: Orchestral:?(a) Theatre (b) Situation(c) Music (d) Direction

9. Latter :Former :: Hazardous: ?(a) Risky (b) Comfortable(c) Safe (d) Harmful

Directions (10-18): In each of thefollowing questions, find the odd word/number / letters / number pair from thegiven alternatives.

10. (a) VWY (b) QRT(c) LMO (d) JKL

11. (a) B E (b) G J(c) N P (d) Q R

12. (a) 400 (b) 484(c) 625 (d) 729

13. (a) 1000 (b) 1725(c) 2744 (d) 4096

14. (a) 12-16 (b) 60-80(c) 30-50 (d) 36-48

15. (a) C X (b) D W(c) J Q (d) L R

16. (a) Cyclotron (b) Basic(c) Pascal (d) Fortran

17. (a) Rooster (b) Buck(c) Gander (d) Peahen

18. (a) Booklet (b) Pamphlet(c) Voucher (d) Brochure

19. Which word will appear third inthe dictionary?(a) BALLIUM(b) BALLISTICS(c) BALLERINA(d) BALLISTITE

20. Which word will appear last in thedictionary?(a) laugh(b) latch(c) laurels (d) latitude

21. Arrange the given words in ameaningful order:1. INFANT2. ADOLESCENT3. CHILD4. OLD5. ADULT

(a) 3, 1,2,4,5 (b) 1, 3, 2, 5, 4(c) 3, 2, 4, 5, 1(d) 5, 4, 3, 2, 1

Directions (22-23) : In each of thefollowing questions, which one set ofletters when sequentially placed at thegaps in the given letter series shallcomplete it ?

22. c _ ab _ ca _ bc _ a(a) bcab (b) abcb(c) bacb (d) cbac

23. _ ba _ bab _ babb _ b(a) baaa (b) abbb(c) babb (d) abab

Directions (24-27): In each of thefollowing questions, select the missingnumber from the given responses.

24. 500, 484, 459, __?__, 374(a) 384 (b) 432(c) 418 (d) 423

25. MNOPWXYZRSTUBCD __?__(a) A (b) E(c) 1 (d) F

26. 60, 69, 85, ___?__ ,146(a) 110 (b) 117(c) 109 (d) 120

27. 46, 50, 47, 55, 49, 61, __?__(a) 54 (b) 52(c) 57 (d) 51

28. The average age of 25subordinates in an office is 30years. If the age of Manager isadded, the average age increasesto 31 years. What is the age of theManager?

REASONING

Page 20: Download SSC CGL Tier I Last 5 Year Papers e Book Www.sscportal.in (1)

1.2: Solved Paper 08-07-2012 (1st Sitting)

Join Online Coaching for SSC Exams:http://sscportal.in/community/courses

Buy Printed Study Material for SSC Exams:http://sscportal.in/community/study-kit

(a) 26 (b) 36(c) 46 (d) 56

29. Class A has students twice that ofclass B. After adding 20 studentsto class A and 30 students to classB, the total number of students inboth the classes is 140. What isthe number of students in class Ain the beginning?(a) 30 (b) 60(c) 80 (d) 140

30. Find the lowest number which isdivided by 8, 12, 15 and 20 leavesthe remainder 2.(a) 360 (b) 242(c) 122 (d) 82

31. If B becomes A and P becomes O,what will K become in the Englishalphabet?(a) L (b) J(c) H (d) N

32. The given word is followed by 4responses, one of which cannotbe formed by using the letters ofthe word given ‘CAMBRIDGE’.Find out the word.(a) BRIDGE (b) BRIDE(c) CAME (d) CAMP

33. If SEASONAL is written asESSANOLA, how can SEPA RATEbe written in that code?(a) SEAP ARET(b) ESPARATE(c) ESPAARTE(d) ESAPARET

34. In certain code, RAGHAVAN iswritten as GARVAHNA. In thatcode which word will be written asMATHAVAN?(a) TAMVAHNA(b) TAMVAHAN(c) TAMHAVNA(d) MATVAHNA

35. If ‘FRIEND’ is coded as‘IULHQG’, how will you code‘ENEMY’?(a) HQHPB (b) HQHPA(c) HQEMY (d) HQHPG

36. In a coded language, if the wordEQUATION is coded asGSWCVKQP, then how is theword DONKEY coded?

(a) FQPMGA (b) YEKNOD(c) GWCVKJ (d) PQKUCW

37. If38 + 15 = 66 and 29 + 36 = 99,then 82 + 44 = ?(a) 77 (b) 88(c) 80 (d) 94

38. If + means ÷, - means x, x means +,÷ means –, give the value for45 + 9 - 3 × 15 ÷ 2(a) 40 (b) 36(c) 56 (d) 28

Directions (39-40): In each of thefollowing questions, select the missingnumber from the given responses.39. 7 8 6

4 9 53 2 ?25 70 29(a) 9 (b) 8(c) 1 (d) 5

40. 3 4 56 7 89 1 257 11 ?(a) 42 (b) 21(b) 11 (d) 18

41. From the given details, estimatethe number of people affected byTuberculosis in particular localityin the year 1994.1994 1995 1996 1997 1998? 92 113 141 176(a) 9 9 (b) 8 5(c) 7 1 (d) 7 8

42. A man started from a place walkedtowards North for 5 km thenturned 90° to his right and walkedanother 5 km. Then he turned 45°to his right and walked 2 km andturned 45° to his left. What is hisdirection now?(a) South (b) South East(c) East (d) South West

43. A boy running towards South,turns to his right and runs, Thenhe turns to his right and finallyturns to his left. Towards whichdirection is he running now?(a) East (b) West(c) South (d) North

Directions (44-45): In the followingquestions, one statement is given,followed by two conclusions I and II.You have to consider the statements tobe true even if they seem to be atvariance from commonly known facts.You have to decide which of the givenconclusions, if any, follow from thegiven statements.

44. StatementTo keep myself updated, I alwayslisten to 9 O’clock news on radio.Conclusions:

I. The person does not readnewspaper.

II. Recent news is available only onradio.(a) Only conclusion I is implicit(b) Only conclusion II is implicit(c) Neither conclusion I nor II is

implicit(d) Both conclusions I and II are

implicit45. Statement

All wealthy people are happy inlife.Conclusions:

I. Wealth and happiness are directlyrelated.

II. Wealth leads to happiness in life.(a) Only conclusion I follows(b) Only conclusion II follows(c) Both conclusions I and II

follow(d) Neither conclusion I nor II

followsDirections: In the following questions,which answer figure will complete thequestion figure?

46. Question Figure

Answer figure

(a) (b) (d)(c)

Page 21: Download SSC CGL Tier I Last 5 Year Papers e Book Www.sscportal.in (1)

1.3: Solved Paper 08-07-2012 (1st Sitting)

Join Online Coaching for SSC Exams:http://sscportal.in/community/courses

Buy Printed Study Material for SSC Exams:http://sscportal.in/community/study-kit

47. From the given answer figures,select the one in which thequestion figure is hidden!embedded.Question Figure

Answer Figures

(a) (b) (d)(c)

Directions: In the following question, apiece of paper is folded and cut asshown below in the question figures.From the given answer figures, indicatehow it will appear when opened.

48. Question Figures:

Answer Figures:

(a) (b) (c) (d)

49. Select the correct option that willbe the mirror reflection of theproblem figure.Question Figure

Answer Figures

(a) (b) (c) (d)

Directions: In the following question, aword is represented by only one set ofnumbers as given in anyone of thealternatives. The sets of numbers givenin the alternatives are represented bytwo classes of alphabets as in twomatrices given below. The columns androws of Matrix I are numbered from 0 to4 and that of Matrix II are numberedfrom 5 to 9. A letter from these matricescan be represented first by its row andnext by its column, e.g., ‘P’ can berepresented by 11, 32, etc. and ‘K’ canbe represented by 65, 78, etc. Similarly,you have to identify the set for theword SALM.

50. MATRIX-I

0 1 2 3 4

0 P A I V R

1 I P R A V

2 A R V P I

3 V I P R A

4 R V A I P

MATRIX· II

5 6 7 8 9

5 S L K M E

6 K M S E L

7 M E L K S

8 L K E S M

9 E S M L K

(a) 55, 20, 56, 69(b) 79, 13, 69, 75(c) 96, 34, 76, 89(d) 67, 21, 85, 97

51. Which one of the following is anexample for a non-economic good?(a) Doctor’s service(b) Teacher’s service(c) Mother’s service(d) Banker’s service

52. Which one of the followingcommittee is associated withbanking sector reforms in India ?(a) L. C. Gupta (b) Narsimhan(c) Chakravarty(d) Kelkar

53. Which one of the following is nota qualitative credit controlmeasure of the RBI?(a) Fixing margin requirements(b) Variable interest rates

GENERAL AWARENESS(c) Open market operations(d) Credit rationing

54. The 13th Five Year Plan will beoperative for the period(a) 2010 - 2015(b) 2011 - 2016(c) 2012 - 2017(d) 2013 - 2018

55. The national income of a nation isthe(a) Government’s annual revenue(b) Sum total of factor incomes(c) Surplus of public sector

enterprises(d) Exports minus imports

56. The Constitutional Amendment

Act that has introducedsafeguards against the misuse ofproclamation of nationalemergency is the(a) 42nd Amendment Act(b) 43rd Amendment Act(c) 44th Amendment Act(d) 45th Amendment Act

57. The Fundamental Rights can besuspended by the(a) Governor(b) President(c) Law Minister(d) Prime Minister

58. The main reason for the growth ofcommunalism in India is

Page 22: Download SSC CGL Tier I Last 5 Year Papers e Book Www.sscportal.in (1)

1.4: Solved Paper 08-07-2012 (1st Sitting)

Join Online Coaching for SSC Exams:http://sscportal.in/community/courses

Buy Printed Study Material for SSC Exams:http://sscportal.in/community/study-kit

(a) Educational and economicbackwardness of minoritygroups

(b) Political consciousness(c) Social inequalities(d) Imposing ban on communal

organisations59. A Retired Judge of a High Court is

not permitted to practice as alawyer in(a) Supreme Court(b) Any Court in India(c) High Courts(d) Except the High Court where

he retired60. Which one of the following does

not match?(a) Hindu Marriage Act : 1955(b) Medical Termination of

Pregnancy Act: 1971(c) Domestic Violence on Women

Act: 1990(d) Cruelty against Women: 1995

61. Who among the following wasthe First Viceroy of India ?(a) Lord Ripon(b) Lord Curzon(c) Lord Mountbatten(d) Lord Canning

62. Prithvi Raj Chauhan was defeatedin the Second Battle of Tarain by(a) Mahmud Ghazni(b) Muhammad Ghori(c) Qutbuddin Aibak(d) Yalduz

63. The original name of NanaPhadnavis was(a) Mahadaji Sindhia(b) Tukoji Holkar(c) Narayan Rao(d) Balaji Janardan Bhanu

64. Who among the following firstpropounded the idea of BasicEducation?(a) Jawahar Lal Nehru(b) Raja Ram Mohan Roy(c) Mahatma Gandhi(d) Dayanand Saraswati

65. Arrange the following inchronological order:

I. Dandi MarchII. Simon CommissionIII. Poona PactIV. Gandhi Irwin Pact(a) II, I, III, IV(b) II, I, IV, III(c) IV, III, I, II(d) IV, III, II I

66. Which one of the following is notcorrectly matched?(a) Darjeeling - West Bengal(b) Mount Abu - Rajasthan(c) Kodaikanal - Tamil Nadu(d) Simla - Uttar Pradesh

67. The earth is at its maximumdistance from the Sun on(a) January 30th(b) December 22nd(c) September 22nd(d) July 4th

68. Consider the following pairs;Tributary Main Piver1. Chambal Yamuna2. Sone Narmada3. Manas Brahmputra

Which one of the pairs givenabove is/are correctlymatched?

(a) 1, 2 and 3(b) 1 and 2 only(c) 2 and 3 only(d) Only 3

69. A form of condensation thatreduces visibility and causesbreathing problems is(a) Dew (b) Frost(c) Smog(d) Mist

70. The total population divided byavailable arable land area isreferred to as(a) Population density(b) Nutritional density(c) Agricultural density(d) Industrial density

71. Green glands are associated with(a) Reproduction(b) Excretion(c) Respiration(d) Digestion

72. During respiration, the gasesenter into the blood and leavethe same by the process of(a) Active transport(b) Diffusion(c) Diffusion and active transport(d) Osmosis

73. Heart is devoid of(a) Cardiac muscle(b) Involuntary muscle(c) Voluntary muscle(d) Smooth muscle

74. The soil salinity is measured by(a) Conductivity meter(b) Hygrometer(c) Psychrometer(d) Auxanometer

75. Which of the following is a fungaldisease?(a) Leucoderma(b) Eczema(c) Ringworm(d) Elephantiasis

76. Chickenpox is caused by(a) DNA virus(b) Variola virus(c) Streptococcus(d) Vi rio cholerae

77. Instruments can be shielded fromoutside magnetic effects bysurrounding them with(a) Iron shield(b) Rubber shield(c) Brass shield(d) Glass shield

78. Find the odd one.(a) Marble (b) Blackboard(c) Chalk (d) Slaked lime

79. The following is a pseudo-force(a) Centrepetal force(b) Centrifugal reaction force(c) Centrifugal force(d) Strong nuclear force

80. The hydraulic brake used inautomobiles is a direct applicationof(a) Archimedes principle(b) Torricellean law(c) Bernoulli’s Theorem(d) Pascal’s law

Page 23: Download SSC CGL Tier I Last 5 Year Papers e Book Www.sscportal.in (1)

1.5: Solved Paper 08-07-2012 (1st Sitting)

Join Online Coaching for SSC Exams:http://sscportal.in/community/courses

Buy Printed Study Material for SSC Exams:http://sscportal.in/community/study-kit

81. Of the following, which is thefastest?(a) CD-ROM(b) RAM(c) Registers(d) Cache

82. An alternate name for thecompletely interconnectednetwork topology is(a) Mesh (b) Star(c) Tree(d) Ring

83. Amides can be converted toamines by the reaction named(a) Perkin(b) Claisen(c) Claisen(d) Clemmesen

84. The base used as an antacid is(a) Calcium hydroxide(b) Barium hydroxide(c) Magnesium hydroxide(d) Silver hydroxide

85. A process which is not helpful inthe prevention of rusting of iron is(a) annealing(b) applying grease(c) galvanising(d) painting

86. Denatured alcohol(a) is a form of alcohol(b) is unfit for drinking as it

contains poisonous substances(c) contains coloured impurities(d) is sweet’ to taste

87. Phenolics as pollutants can beremoved from waste water by u s eof(a) Ion exchange resin technique(b) Electrolyte decomposition

technique(c) Reverse osmosis method(d) Polymeric adsorbents

88. The stability of a pond ecosystemdepends on(a) micro-organisms and fishes(b) micro-organisms and zoo

planktons(c) fishes and reptiles(d) producers and consumers

89. Supersonic air planes create ashock wave called(a) Transition wave(b) Ultrasound(c) Transverse wave(d) Sonic boom

90. The main factor which determinesbalance of nature is(a) human activities(b) Rabit and habitat(c) environmental conditions(d) availability of food

91. The danger signals are red whilethe eye is more sensitive to yellowbecause(a) absorption in red is less than

yellow and hence red is visible from a distance

(b) scattering in yellow light isless than red

(c) the wavelength of red light ismore than yellow light

(d) none of the above reasons92. One of the following Chief

Ministers received the HighestCivilian Award “Bharat Ratna”(a) Lalu Prasad Yadav of Bihar(b) Late M. G. Ramachandran of

Tamil Nadu(c) Jyothi Basu of West Bengal(d) Late N. T. Rama Rao of Andhra

Pradesh93. The first ‘disabled athelete’ to

qualify for an able bodied event inthe 2012 London Olympics in 400metres race is(a) Oscar Pistorius(b) Walter Rallis(c) Raymond Irchovich(d) Gerald Hangovin

94. Which one of the following filmactors has been conferred theHonorary Doctorate in Arts andCulture by the Bedford University,London?(a) Shahrukh Khan(b) Amir Khan

(c) Saif Ali Khan(d) Anil Kapoor

95. The city of Prayag was namedAllahabad -the city of Allah by(a) Aurangzeb(b) Akbar(c) Shahjehan(d) Bahadur Shah Zafar

96. Spot the odd one from thefollowing:(a) Tsunami(b) Earthquakes(c) Windmills(d) Cyclones

97. Who among the following haswon the Pantaloon’s Femina MissIndia World, 2012 Award?(a) Prachi Misra(b) Vanya Misra(c) RocheIla Maria(d) Sandhya Agarwal

98. Which one of the followingstatements is not correct aboutIndia’s population as per the 2011census?(a) Literacy rate has gone up to

74% from 65%(b) Kerala has the highest literacy

rate(c) Bihar has the lowest literacy

rate(d) Men outnumber women in

growth of literates99. Who advocated the adoption of

‘PURA’ model to eradicate ruralpoverty ?(a) Dr. A. P. J. Abdul Kalam(b) Sri Abhijit Sen(c) Maulana Abdul Kalam Azad(d) Prof. A. M. Patha

100. Which one of the followingthough called a garden is infact,not a garden?(a) Vrindavan Garden of My sore(b) Hanging Garden of Mumbai(c) Eden Garden of Kolkata(d) Shalimar Garden of Kashmir

Page 24: Download SSC CGL Tier I Last 5 Year Papers e Book Www.sscportal.in (1)

1.6: Solved Paper 08-07-2012 (1st Sitting)

Join Online Coaching for SSC Exams:http://sscportal.in/community/courses

Buy Printed Study Material for SSC Exams:http://sscportal.in/community/study-kit

101. The next term of the series 1, 5,12 24, 43 is(a) 51 (b) 62(c) 71 (d) 78

102. The least multiple of 13 whichwhen divided by 4, 5, 6, 7 leavesremainder 3 in each case is(a) 3780 (b) 3783(c) 2520 (d) 2522

103. The simplest value of

1 12 3 3 4

++ + +

1 14 5 5 6

++ + is

(a) 3 ( 2 1)−

(b) 2 ( 3 1)−

(c) 3 1− (d) 2 1−104. A and B are partners in a business.

A contributes 14 of the capital for

15 months and B received 23 of

the profit. Find for how long B’smoney was used?(a) 6 months (b) 8 months(c) 10 months (d) 12 months

105. If 21 is added to a number, itbecomes 7 less than thrice of thenumber. Then the number is(a) 14 (b) 16(c) 18 (d) 19

106. Two men A and B started a job inwhich A was thrice as good as Band therefore took 60 days lessthan B to finish the job. Howmany days will they take to finishthe job, if they take to finish thejob, if they start workingtogether?(a) 15 days (b) 20 days

(c)1222 days(d) 25 days

107. A rectangle garden is 100 m × 80m. There is a path along thegarden and just outside it. Widthof the path is 10m. The area of thepath is(a) 1900 sq m (b) 2400 sq m(c) 3660 sq m (d) 4000 sq m

108. A dealer offered a machines forsale for Rs. 27,500 but even if hehad charge 10% less, the wouldhave made a profit of 10%. Theactual cost of the machine is(a) Rs. 22000 (b) Rs. 24500(c) Rs. 22500 (d) Rs. 22275

109. An employer reduces the numberof employees in the ratio 8 : 5 andincreases their wages in the ratio 7: 9. As a result, the overall wagesbill is(a) Increased in the ration 56 : 69(b) Decreased in the ratio 56 : 45(c) Increased in the ratio 13 : 17(d) Decreased in the ratio 17 : 13

110. The average age of a jury of 5 is40. If a member aged 35 resignsand man aged 35 resigns and aman aged becomes a member,then the average age of the newjury is(a) 30 (b) 38 (c) 40 (d) 42

111. With average speed of 40 km/hour, a train reaches itsdestination in time. If it goes withan average speed of 35 km hour, itis late by 15 minutes. The totaljourney is(a) 30 km (b) 40 km(c) 70 km (d) 80 km

112. A man makes a profit of 20% onthe sale by selling 20 articles forRs. 1. The number of articles hebought by Rs. 1 is(a) 20 (b) 24(c) 25 (d) 30

113. The number of seats in anauditorium is increased by 25%.The price of a ticket is also

increased by 12%. Then theincrease in revenue collection willbe(a) 40% (b) 35%(c) 45% (d) 48%

114. A ship is moving at a speed of 30km/hr. To know the depth of theocean beneath it, it sends a radiowave which travels at a speed 200m/s. The ship receives the signalafter it has moved 500 m. Thedepth of the ocean is

(a)1432

km (b) 12 km

(c) 6 km (d) 8 km115. A person takes a loan of Rs.

10,0000 partly from a bank at 8%p.a. and remaining from anotherbank at 10% p.a. He pays a totalinterest of Rs. 950 per annum.Amount of loan taken from thefirst bank (in Rs.) is(a) 2500 (b) 5200(c) 2050 (d) 5020

116. If 2

21a

a+ = 98(a > 0), then the

value of 3

31a

a+ will be

(a) 535 (b) 1030(c) 790 (d) 970

117. If x = 1 + 2 + 3 , then thevalue of (2x4 – 8x3 – 5x2 + 26 x –28) is(a) 6 6 (b) 0

(c) 3 6 (d) 2 6118. If the distance between two point

(0, -5) and (x, 0) is 13 unit, then x=(a) 10 (b) ± 10(c) 12 (d) ± 12

119. If 4x = 18y, then the value of

1xy

is

QUANTITATIVE APTITUDE

Page 25: Download SSC CGL Tier I Last 5 Year Papers e Book Www.sscportal.in (1)

1.7: Solved Paper 08-07-2012 (1st Sitting)

Join Online Coaching for SSC Exams:http://sscportal.in/community/courses

Buy Printed Study Material for SSC Exams:http://sscportal.in/community/study-kit

(a)13 (b)

72 (c)

23 (d)

32

120. If x + 1x = 5, then the value of

42

2

1

3 1

xx

x x

+

− + is

(a) 70 (b) 50 (c) 110 (d) 55121.If x = 2 + 3 , y = 2 - 3 , then the

value of 2 2

3 3x yx y

+

+ is

(a)7

3 8 (b)7

4 0

(c)7

1 9 (d)7

2 6122. If a2 + b2 + c2 = 2 (a – b – c) – 3

then the value of 2a – 3b + 4c is(a) 3 (b) 1(c) 2 (d) 4

123. If 12

2x

x− = 6, then the value of

22

116

xx

+ is

(a)192 (b)

172

(c)1 83 (d)

152

124. If 15

3a

a+ = 5, the value of

22

1925

aa

+ is

(a)345 (b)

395

(c)425 (d)

525

125. The area of the triangle formed bythe line 5x + 7y = 35, 4x + 3y =12 and x-axis is

(a)1 6 01 3 sq. unit

(b)15013 sq. unit

(c)14013 sq. unit

(d) 10 sq. unit126. If an obtuse-angled triangle ABC,

A∠ is the obtuse angle and O isthe orthocenter. If BOC∠ = 54°,then BAC∠ is(a) 108° (b) 126°(c) 136° (d) 116°

127. If the ratio of areas of two similartriangles is 9 : 16, then the ratioof their corresponding sides is(a) 3 : 5 (b) 3 : 4(c) 4 : 5 (d) 4 : 3

128. Let BE and CF be the twomedians of a ABC∆ and G betheir intersection. Also let EF cutAG at O. Then AO : OG is(a) 1 : 1 (b) 1 : 2(c) 2 : 1 (d) 3 : 1

129. If S is the circumcentre of ABC∆and A∠ = 50°, then the value of

BCS∠ is(a) 20° (b) 40° (c) 60° (d) 80°

130. AC and BC are two equal cords ofa circle. BA is produced to anypoint P and CP, when joined cutsthe circle at T. Then(a) CT : TP = AB : CA(b) CT : TP = CA : AB(c) CT : CB = CA : CP(d) CT : CB =CP : CA

131. PQ is a direct common tangent oftwo circle of radii r1 and r2touching each other externally atA. Then the value of PQ2 is(a) r1r2 (b) 2 r1r2(c) 3 r1r2 (d) 4r1r2

132.

BC is the chord of a circle withcentre O. A is a point on major areBC as shown in the above figure.What is the value of BAC∠ +

OBC∠ ?(a) 120° (b) 60°(c) 90° (d) 180°

133. Two circles with radii 5cm and 8cm touch each other externally ata point A. If a straight line throughthe point A cuts the circles atpoints P and Q respectively, theAP : AQ is(a) 8 : 5 (b) 5 : 8(c) 3 : 4 (d) 4 : 5

134. If I is the In-centre of ABC∆ and<A = 60°, then the value of

BIC∠ is(a) 100° (b) 120°(c) 150° (d) 110°

135. The external bisectors of B∠ andC∠ of ABC∆ meet at point P. If

BAC∠ = 80°, the BPC∠ is(a) 50° (b) 40°(c) 80° (d) 100°

136. When a pendulum of length 50cm oscillates, it produces an arc of16 cm. The angle so formed indegree measure so formed indegree measure is (approx)(a) 18°25’ (b) 18°35’(c) 18°20’ (d) 18°08’

137. If x, y are positive acute angles, x+ y < 90° and sin (2x – 20°) = cos(2y + 20°), then the value of sec(x + y) is

(a) 2 (b)12

(c) 1 (d) 0138. If 5 tan θ = 4, then the value of

5sinθ - 3cosθ 5sinθ 3cosθ

+

is

(a)17 (b)

27

(c)57 (d)

25

Page 26: Download SSC CGL Tier I Last 5 Year Papers e Book Www.sscportal.in (1)

1.8: Solved Paper 08-07-2012 (1st Sitting)

Join Online Coaching for SSC Exams:http://sscportal.in/community/courses

Buy Printed Study Material for SSC Exams:http://sscportal.in/community/study-kit

139. The least value of4sec2 θ + 9cosec2 θ ) is(a) 1 (b) 19(c) 25 (d) 7

140. If tan (x + y) tan (x – y) = 1, then

the value of tan 23x

is

(a)13 (b)

23

(c) 3 (d) 1141. If x = cosec θ - sin θ and y = sec θ

- cos θ , then the value of x2y2 (x2

+ y2 + 3) is(a) 0 (b) 1 (c) 2

(d) 3

142. If 0 ≤ θ ≤ 2π

, 2y cos θ = sin θ

and 2x sec θ - y cosec θ = 3, thenthe value of x2 + 4y2 is(a) 1 (b) 2 (c) 3

(d) 4143. When the angle of elevation of the

sun increases from 30° to 60°, theshadow of a post is diminished by5 metres. Then the height of thepost is

(a)5 3

2m (b) 2 3

2m

(c) 25 3 m (d) 4

5 3 m144.A rail road curve is to be laid out on

a circle. What radius should beused if the track is to changedirection by 25° in a distance of40 metres?(a) 91.64 metres(b) 90.46 metres(c) 89.64 metres(d) 93.64 metres

145. If sin θ + sin2 θ = 1, then thevalue of cos12 θ + 3cos10 θ +cos6 θ - 1 is(a) 0 (b) 1(c) -1 (d) 2

Directions (146-150): Population of thefive adjacent areas of a town, in the yearof 2020, are represented in the followingPie-chart. The ratio of the numbers ofmales to that of females in these areasare states in the table below. The total ofthe population in all the five areas is 72lakh. Study the Pie-chart and the tableand then answer the questions.FIGURERatio of number of males (M)

to females (F)Areas S1 S2 S3 S4 S5Ratio M:F 3 : 2 4 : 1 7 : 3 2 : 3 13 : 7

146. 12 lakh in the population of thearea

(a) S1 (b) S3(c) S5 (d) S4

147. The number of males in the areasS1 and S4 together is(a) 13.8 lakh(b) 8.2 lakh(c) 16.2 lakh(d) 15.8 lakh

148. The ratio of number of females inthe areas S2 in the areas S5 is(a) 7 : 9 (b) 36 : 13(c) 9 : 7 (d) 13 : 36

149. If, in the year 2010, there was anincrease of 5% population in theareas S1 and 8% increase inpopulation of the area S3compared to the previous year,then the ratio of population in theareas S1 and S2, in the year 2009was(a) 3 : 10(b) 27 : 10(c) 27 : 70(d) 10 : 3

150. The average of female populationin all the five areas is lower thanthe female population in each ofthe areas(a) S1 and S2(b) S2 and S5(c) S2 and S4(d) S4 and S5

Directions (151-155): In thefollowing questions, some parts of thesentences have errors and some havenone, Find out which part of a sentencehas an error. The number of that part isyour answer. If there is no error then youranswer is (d) i.e. No error.151. Air pollution, together with

littering. (a)/ are causing manyproblems(b)/ Vin our cities. (c)/No error (d)

152. The accused refused (a)/ toanswer to the policeman (b)/ onduty.(c)/No error (d).

ENGLISH COMPREHENSION153. What is (a)/ the use of me (b)/

attending the session? (c)/ Noerror (d)

154. We met our prospective employer(a) // for a briefing session(b) / inthe Taj Hotel. (c) / No error (d).

155. Because of the severe snow stormand the road blocks, (a)/ the airforce dropped food and (b)/medical supplies close to the city.(c) No error (d)

Directions (156-160): In the followingquestions, sentences are given with

blanks to be filled in with an appropriateword(s). Four alternatives are suggestedfor each question. Choose the correctalternative out of the four as youranswer.156. ______ pollution control measures

are expensive, many industrieshesitate to adopt them.(a) Although(b) However(c) Because (d) Despite

157. It is not for a man to be confinedto the pursuit of wealth.

Page 27: Download SSC CGL Tier I Last 5 Year Papers e Book Www.sscportal.in (1)

1.9: Solved Paper 08-07-2012 (1st Sitting)

Join Online Coaching for SSC Exams:http://sscportal.in/community/courses

Buy Printed Study Material for SSC Exams:http://sscportal.in/community/study-kit

(a) healthy (b) easy(c) possible (d) common

158. ___ his being innocent of thecrime, the judge sentenced him toone year imprisonment.(a) Inspite of(b) In case of(c) On account of(d) In the event of

159. It is a story of two men and abatch of armoured cars.(a) deceased (b) diseased(c) decrepit (d) defeated

160. Although there is ________ gun-fire, there is no stiff resistance tothe revolutionary army,(a) bitter (b) meagre(c) continuous (d) sporadic

Directions (161-165): In the followingquestions, out of the four alternatives,choose the one which best expressesthe meaning of the given ‘word as youranswer.161. Vociferous

(a) violent (b) loud(c) secret (d) true

162. Fictional(a) genuine (b) authentic(c) fanciful (d) real

163. Trivial(a) crucial (b) significant(c) vital (d) ordinary

164. Impudent(a) Vigilant (b) Astute(c) Insolent (d) Arrogant

165. Pompous(a) Pretentious (b) Supportive(c) Demanding (d) Flashy

Directions (166·170) : In the followingquestions, choose the word opposite inmeaning to the given word as youranswer.166. Cultivated

(a) Crude (b) Genteel(c) Suave (d) Refined

167. Impertinent(a) Insolent (b) Impudent(c) Cheeky (d) Courteous

168. Divulge

(a) Disseminate(b) Dissemble(c) Publicize (d) Transmit

169. Appreciation(a) Aspersion (b) Admiration(c) Commendation(d) Compliment

170. Supple(a) Pliant (b) Pliable(c) Rigid (d) Flexible

Directions (171-175): In the followingquestions’, four alternatives are givenfor the idiom/phrase and bold italicisedin the sentence. Choose the alternativewhich best expresses the meaning of theidiom/phrase as your answer. -171. Once the case reached the court,

the police washed their hands offit.(a) waited for a response to(b) claimed credit for(c) disassociated themselves from(d) seemed eager to continue

172. She wanted to go hitch-hiking buther mother put her foot down andnow she’s ‘going by bus.(a) took a firm stand(b) expressed her displeasure(c) scolded her badly(d) got irritated

173. Adolescence is a period of halocyan days.(a) hard days(b) of mental pressure(c) happy days(d) days of preparation

174. My sincere advice to my maid-servant fell on stony ground.(a) was counter productive(b) had a strong impact(c) made one stubborn(d) had little success

175. He has all his ducks in a row, he iscomplacent.(a) has everything ready(b) is wall organised(c) always scores a zero(d) never gets confused

Directions (176-180): In the followingquestions, a part of the sentence is

printed in bold. Below are givenalternatives to the bold part at (a), (b)and (c) which may improve thesentence. Choose the correctalternative. In case no improvement isneeded your answer is (d).176. You shall have attended if the

court had instructed you to do so.(a) would have had to attend(b) would attend(c) would have to(d) No improvement

177. The relics of Greece over whichsuch a great deal of evidence hasbeen collected should bepreserved.(a) from which(b) on which(c) ascent which(d) No improvement

178. When the beverage was ready,they drank possibly as much asthey could.(a) as much as they possibly

could(b) as much as possibly they

could(c) as much as they could

possibly(d) No improvement

179. A citizen is expected to giveallegiance to his country of origin.(a) homage(b) loyalty(c) obedience(d) No improvement

180. We were with daggers drawndespite attempts to understandeach other;(a) in (b) on(c) at(d) No improvement

Directions (181-185): In the followingquestions, out of the four alternatives.choose the one which can besubstituted for the given words/sentence.181. A round or cylindrical container

used storing things such as food,chemicals or rolls of film

Page 28: Download SSC CGL Tier I Last 5 Year Papers e Book Www.sscportal.in (1)

1.10: Solved Paper 08-07-2012 (1st Sitting)

Join Online Coaching for SSC Exams:http://sscportal.in/community/courses

Buy Printed Study Material for SSC Exams:http://sscportal.in/community/study-kit

(a) tankard (b) vessel(c) canister (d) casket

182. A place of permanent residence(a) abode (b) dormitory(c) domicile (d) apartment

183. That cannot be altered orwithdrawn(1 ) irrevocable (b) irretrievable(c) irrefutable (d) irresistible

184. Money paid to employees onretirement(a) gratuity (b) gift(c) pension (d) arrears

185. A place where clothes are kept(a) closet (b) drawer(c) wardrobe (d) cupboard

Directions (186-190): In the followingquestions four words are given in eachquestion, out of which only one word iscorrectly spelt. Find the correctly speltword as your answer.186. (a) garulous (b) garrulous

(c) garullous (d) garrullous187. (a) marquee (b) markue

(c) marnuei (d) marquie188. (1 ) puissant (b) puiscant

(c) puiscent (d) puissent189. (a) disconncerting

(b) disconserting(c) discuncerting(d) disconcerting

190. (a) exilarate (b) exsilerate(c) exsilarate (d) exhilarate

Directions (191-200): In the followingquestions, you have two brief passageswith 5 questions in each passage, Readthe passages carefully and choose thebest answer to each question out of thefour alternatives.

PASSAGE –I(Question Nos. 191 to 195)

Stuck with be developmentdilemma? Stay away from managementcourses. Seriously, one of the biggestcomplaints that organisations haveabout management courses is that theyfail to impact the participants’ on-the-jobbehaviour. Some management trainersstress the need for follow-up andreinforcement on the job. Some go so faras briefing the participants’ managers on

what behaviour they should bereinforcing back on the job. Othersinclude a follow-up training day toreview the progress of the participants.None of this is really going far enough.

The real problem is that coursepromoters view development assomething which primarily, takes place ina classroom. A course is an event andevents are, by definition limited in time.When you talk about follow-up after acourse, it is seen as a nice idea, but notas an essential part of the participants’development programme. Any rational,empowered individual should be able totake what has been learnt in a course andtransfer it to the work place or so theargument goes. Another negative aspectof the course mindset is that, primarily,development is thought to be about skill-acquisition.

So, it is felt that the distinctionbetween taking the course and behavingdifferently in the work place parallels thedistinction between skill-acquisition andskill-application. But can such a sharpdistinction be maintained ? Skills arereally acquired only in the context ofapplying them on the job, finding themeffective and therefore, reinforcing them.

The problem with courses is thatthey are events, while development is anon-going process which, involves,within a complex environment, continualinteraction, regular feedback andadjustment. As we tend to equatedevelopment with a one-off event, it isdifficult to get seriously motivatedabout the follow-up. Anyone paying fora course tends to look at follow-up as anunnecessary and rather costly frill.191. What is the passage about?

(a) personal management(b) development dilemma(c) management courses(d) course promoters’ attitude

192. Which of the following statementsis false?(a) Some management trainers

stress the need for follow-upand reinforcement on the job

(b) Some suggest a follow-uptraining day to review theprogress of the participants

(c) Some go to the extent ofbriefing the participants’managers on what behaviourthey should be reinforcingback on the job

(d) The real problem is thatcourse promoters viewdevelopment as ‘somethingwhich does not take placeduring a course

193. The writer’s attitude, as reflectedin the passage, is(a) critical (b) ironic(c) sympathetic(d) philosophical

194. The course promoters’ attitude is(a) self-righteous(b) indifferent (c) easy-going(d) unprogressive

195. The words ‘mindset’ here means(a) determined mind(b) a (fixed) attitude of mind(c) an open mind(d) mindful

PASSAGE II(Question Nos. 196 to 200)

One may look at life, events,society, history, in another way. A waywhich might, at a stretch, be described asthe Gandhian way, though it may be fromtimes before Mahatma Gandhi came onthe scene. The Gandhian reaction to allthe grim poverty, squalor anddegradation of the human being wouldapproximate to effort at self-change andself-improvement, to a regime of livingregulated by discipline from within. Tochange society, the individual must firstchange himself. In this way of looking atlife and society, words too begin to meandifferently. Revolution, for instance, is aterm frequently used, but not always inthe sense it has been in the lexicon ofthe militant. So also with words likepeace and struggle. Even society maymean differently, being some kind oforganic entity for the militant, and moreor less a sum of individuals for the

Page 29: Download SSC CGL Tier I Last 5 Year Papers e Book Www.sscportal.in (1)

1.11: Solved Paper 08-07-2012 (1st Sitting)

Join Online Coaching for SSC Exams:http://sscportal.in/community/courses

Buy Printed Study Material for SSC Exams:http://sscportal.in/community/study-kit

Gandhian. There is yet another way,which might, for want of a betterdescription, be called the mystic. Themystic’s perspective measures theseconcerns that transcend politicalambition and the dynamism of thereformer, whether he be militant orGandhian. The mystic measures theterror of not knowing the remorselessmarch of time; he seeks to know whatwas before birth, what comes after death.The continuous presence of death, ofthe consciousness of death, sets hispriorities. and values: militants andGandhians kings and prophets mustleave all that they have built; all that theyhave un-built and depart whenmessengers of the buffalo-riding Yamacome out of the shadows. Water will towater, dust to dust. Think ofimpermanence. Everything passes.

196. The Gandhian reaction to povertyis(a) a total war on poverty(b) self-discipline(c) self-abnegation(d) a regulated distribution of

wealth197. According to Gandhianism, the

individual who wants to changesociety(a) should destroy the existing

society(b) must re-form society(c) must change himself(d) may change society without

changing himself198. Who, according to the passage,

finds new meaning for words likerevolutions, peace and struggle?(a) A Gandhian who believes in

nonviolent revolution

(b) A militant(c) A mystic(d) A Gandhian who disciplines

himself from within199. The expression ‘water will to

water, dust to dust’ means(a) water and dust can mix well(b) man will become water after

death(c) man will one day die and

become dust(d) man will become dust and

water after death200. What does society mean to a

Gandhian?(a) a sum of individuals(b) an organic entity(c) a regime of living regulated by

discipline from within(d) a disciplined social community

ANSWERS1.(d) 2.(c) 3. (a) 4.(b) 5. (a) 6. (d) 7. (c) 8. (c) 9. (c) 10. (d)

11. (a) 12. (d) 13. (b) 14. (c) 15. (d) 16. (a) 17. (b) 18. (c) 19. (d) 20. (c)21. (b) 22. (c) 23. (b) 24. (d) 25. (b) 26. (a) 27. (b) 28. (d) 29. (b) 30. (c)31. (b) 32. (d) 33. (d) 34. (a) 35. (a) 36. (a) 37. (c) 38. (d) 39. (c) 40. (b)41. (d) 42. (c) 43. (b) 44. (c) 45. (c) 46. (c) 47. (c) 48. (d) 49. (b) 50. (b)51. (c) 52. (b) 53. (c) 54. (*) 55. (b) 56. (c) 57. (b) 58. (a) 59. (d) 60. (c)61. (d) 62. (b) 63. (d) 64. (c) 65. (b) 66. (d) 67. (d) 68. (a) 69. (c) 70. (a)71. (b) 72. (b) 73. (c) 74. (a) 75. (c) 76. (b) 77. (a) 78.(a) 79. (c) 80. (d)81. (c) 82. (a) 83. (c) 84. (c) 85. (a) 86. (b) 87. (d) 88. (d) 89. (d) 90. (c)91. (a) 92. (b) 93. (a) 94. (a) 95. (b) 96. (c) 97. (b) 98. (d) 99. (a) 100. (c)

101. (c) 102.(b) 103. (b) 104. (c) 105. (a) 106. (c) 107. (d) 108. (c) 109. (b) 110. (b)111. (c) 112. (b) 113. (a) 114. (a) 115. (a) 116. (d) 117. (a) 118. (c) 119. (b) 120. (d)121. (d) 122. (b) 123. (a) 124. (b) 125. (a) 126. (b) 127. (b) 128. (c) 129. (b) 130. (c)131. (d) 132. (c) 133. (b) 134. (b) 135. (a) 136. (c) 137. (a) 138. (a) 139. (*) 140. (a)141. (b) 142. (d) 143. (a) 144. (a) 145. (a) 146. (c) 147. (a) 148. (c) 149. (*) 150. (c)151. (b) 152. (b) 153. (b) 154. (b) 155. (a) 156. (c) 157. (c) 158. (a) 159. (c) 160. (d)161. (b) 162. (c) 163.(d) 164. (c) 165. (a) 166. (a) 167. (d) 168. (b) 169. (a) 170. (c)171. (c) 172. (a) 173. (c) 174.(d) 175. (b) 176.(a) 177. (b) 178. (a) 179. (b) 180. (c)181. (b) 182. (a) 183. (a) 184. (a) 185. (c) 186. (b) 187. (a) 188. (a) 189. (d) 190. (d)191. (b) 192. (d) 193. (a) 194. (d) 195. (b) 196. (b) 197. (c) 198. (d) 199. (d) 200. (a)

Page 30: Download SSC CGL Tier I Last 5 Year Papers e Book Www.sscportal.in (1)

1.12: Solved Paper 08-07-2012 (1st Sitting)

Join Online Coaching for SSC Exams:http://sscportal.in/community/courses

Buy Printed Study Material for SSC Exams:http://sscportal.in/community/study-kit

1. The relationship between thenumber isx : (x2 - 1)(9)2 - 1 = 81 - 1 = 80(100)2 - 1 = 10000 - 1 = 9999

2. The relationship between thenumber is:x2 : x3

(5)2= 25; (5)2= 5(6)2= 36; (5)6= 216

3. The relationship between thenumber is:x : x - 119335 - 119 = 216987 - 119 = 868

4.

Similarly

5. Pairs of consecutive oppositeletters are given:AZ, BY; CX, DW; EV, FU;GT, HS

6.

7. Food is necessary for man.Similarly, fuel is necessary toproduce heat.

8. Spiritual and Belief are inter-related terms. Similarly,Orchestral and music are related.

9. Latter and Former are antonymousto each other. Similarly,

EXPLANATIONSHazardous and Safe areantonymous to each other.

10.

But

11.

In letter group BE, one iscosonant and the other is a vowel.

12. Except the number 729, all otherare perfect squares.400 = 20 × 20; 484 = 22 × 22;625 = 25 × 25But, 729 = 9 × 9 × 9

13. Except the number 1725, all othernumbers are completely divisibleby 4. The number 1725 iscompletely divisible by 5.

14. Except the number pair 30 - 50, allother numbers pairs follow thepattern : x × 3 - x × 44 × 3 = 12, 4 × 4 =1620 × 3 = 60; 20 × 4 = 8012 × 3 = 36; 12 × 4 = 48But,10 × 3 = 30; 10 × 5 = 50

15. Except LR, all other pairs are ofOpposite Letters.C ↔ X : D ↔ W:J ↔ QL ↔ O (Not R)

16. Except Cyclotron, all other arelanguages of computer.

17. Rooster, Gander and Peahen arebirds. Buck is an animal.

18. Except Voucher, all other aresmall book containinginformation on a specific subject.

19. Arrangeement of words as perdictionary:

C. BALLERINA

↓B. BALLISTICS

↓D. BALLISTITE

↓A. BALLIUM

20. Arrangement of words as perdictionary:B. Latch

↓D. Latitude

↓A. Laugh

↓C. Laurels

21. Meaningful order of words:A. Infant

↓C. Child

↓B. Adolescent

↓E. Adult

↓4. Old

22. b / / /c a b a c a c b c b a

23. / / /a b a b b ab b b abb b b

24. 500 - (4)2 = 484484 - (5)2 = 459

459 - (6)2 = 423423 - (7)2 = 374

25. 2M N O P R S T U+→

2W X Y Z B C D E+→

26. 60 + (3)2 = 69

Page 31: Download SSC CGL Tier I Last 5 Year Papers e Book Www.sscportal.in (1)

1.13: Solved Paper 08-07-2012 (1st Sitting)

Join Online Coaching for SSC Exams:http://sscportal.in/community/courses

Buy Printed Study Material for SSC Exams:http://sscportal.in/community/study-kit

69 + (4)2 = 85

5+ (5)2 = 110110 + (6)2 = 146

27.46

28. Age of Manager = New AverageAge + (No. of Subordinates ×Change in Average)= 31 + (25 × 1) = 56 years

29. Suppose, in the beginning thenumber of students in Class B = aTherefore, the number of studentsin Class A = 2aNow,2a + 20 + a + 30 = 140⇒ 3a = 140 - 50

∴ a = 9 03 = 30

Number of students in Class A= 2a = 2 × 30 = 60

30. LCM of 8, 12, 15 and 20

∴ LCM = 2 × 2 × 3 × 5 × 2= 120Since the remainder to be left is 2,the number can be given by 120K+ 2, where K is a positive integer120 × 1 + 2 = 122 (K = 1)

31. 1B A−→

1P O−→

1K J−→

32. There is no ‘P’ letter in the givenword.

33.

Therefore,

34.

Similarly,

36.

Similarly,

37. (3 + 8) × (1 + 6)⇒ 11 × 6 = 66(2 + 9) × (3 + 6)⇒ 11 × 9 = 99Similarly,(8 + 2) × (4 + 4)

⇒ 10 × 8 = 8038. 45 + 9 - 3 × 15 ÷ 2

⇒ ? = 45 ÷ 3 × 15 - 2⇒ ? = 5 × 3 + 15 - 2

⇒ ? = 30 - 2 = 2839. First Column

7 × 4 - 3 = 28 - 3 = 25Second Column8 × 9 - 2 = 72 - 2 = 70Third Column6 × 5 - ? = 29

⇒ ? = 30 - 29 = 140. First Column

3 + 6 × 9 = 3 + 54 = 57Second Column4 + 7 × 1 = 4 + 7 = 11

Third Column5 + 8 × 2

⇒ 5 + 16 = 2141. 78 + 14 = 92

72 + 21 = 113113 + 28 = 141141 + 35 = 176

42.

43.

44. Neither Conclusion I norConclusion II follows. The personalways listens to 9 O’clock newson radio to keep himselfupadated. This does not implythat he does not read newspaper.The use of term ‘only’ in theConclusion II makes it unsuitable.

45. Clearly both the conclusionsfollow. If all wealthy people arehappy in life, then it can be saidthat wealth and happiness aredirectly related.

50. S ⇒ 55, 67, 79, 88, 96A ⇒ 01, 13, 20, 34, 42L ⇒ 56, 69, 77, 85, 98M ⇒ 58, 66, 75, 89, 97

Option S A L M(a) 55 20 56 59(b) 79 13 69 75(c) 96 34 76 89(d) 67 21 85 97

101.The pattern is1 + 4 = 55 + 7 (= 4 + 3) = 12

Page 32: Download SSC CGL Tier I Last 5 Year Papers e Book Www.sscportal.in (1)

1.14: Solved Paper 08-07-2012 (1st Sitting)

Join Online Coaching for SSC Exams:http://sscportal.in/community/courses

Buy Printed Study Material for SSC Exams:http://sscportal.in/community/study-kit

12 + 12 (= 7 + 5) = 2424 + 19 (= 12 + 7) = 43

43 + 28 (=19 + 9) = 71102. LCM of 4, 5, 6 and 7 = 420

∴ Required number= 420k + 3 which is exactlydivisible by 13.= 32 × 13k + 4k + 3Hence, 4k + 3 should be divisibleby 13 for some value of kFor k = 9, 4k + 3 = 39 which isdivisible by 13.∴ Required number= 420 × 9 + 3 = 3783

103.1

2 3+

= 1 3 2

3 2 3 2−

×+ −

= 3 23 2

−−

= 3 2−

∴ 1

4 3+ = −4 3 ;

14 5+ = 5 4− ;

15 6+ = 6 5−

∴ Expression=

3 2 4 3 5 4 6 5− + − + − + −

= 6 2−

= ( )2 3 1−

104. A’s profit : B’s profit

= 1 2:3 3 = 1 : 2

∴A's equ ivlen t ca p ita l

B's equ iva len t ca p ita l

= 12

15434

x

x n

×

× =

12 ⇒

15 13 2n

=

⇒ n = 10 months105. If the number is x, then

x + 21 = 3x - 7⇒ 3x - x = 21 + 7⇒ 2x = 28⇒ x = 14

106. If time taken by A is x days, thentime taken by B = 3x days\ 3x - x = 60⇒ 2x - 60⇒ x = 30Time taken by B = 90 days\ (A + B)’s 1 days’ work

= 1 1

30 9 0+ =

3 1 49 0 90+

=

= 2

4 5\ The work will be completed in

452

i.e., 1222

days.

107.

Area of the shaded region= (100 + 2 × 10) (80 + 2 × 10) -100 × 80= 120 × 100 - 8000= 4000 sq.metere

108. If the C.P. of machine is Rs. x,then

x × 1 101 00 =

27500 90100

×

⇒1 110

x = 275 × 90

⇒ x = 275 900

11×

= Rs. 22500

109. Required ratio = 8 × 7 : 5 × 9= 56 : 45

110. Required average

= 4 0 5 3 5 2 5

5× − +

=1 9 0

5 = 38 years

111. If the total length of journey is xkm, then

35 40x x

− = 60x

⇒8 7

2 8 0x x−

= 14

⇒ 2 8 0x

= 14

⇒ x = 14 ×280 = 70 km

112. C.P. of 20 articles

= 1 0 01 2 0 = Re.

56

∴ Number of articles bought forRe. 1

= 56 × 20 = 24

113. Required increase

= 25 1225 12

100× + +

%

= 40%114. Speed of ship = 30 kmph

= 3 0 5

1 8×

m/sec. = 2 53 m/sec.

Time taken in covering 500 metre

Page 33: Download SSC CGL Tier I Last 5 Year Papers e Book Www.sscportal.in (1)

1.15: Solved Paper 08-07-2012 (1st Sitting)

Join Online Coaching for SSC Exams:http://sscportal.in/community/courses

Buy Printed Study Material for SSC Exams:http://sscportal.in/community/study-kit

= 500 3

2 5×

= 60 seconds

Speed of radio waves

= 2 0 0

1 0 0 0 km/sec. = 15 km/sec.

\

2 14

1 15 5

x x++ = 60

⇒ 2 14

x x+ + = 1 6 05

×

= 12

(12 - x)2 = x2 + 14

⇒ 144 + x2 - 24x = x2 + 14

⇒ 24x = 144 - 14 =

5754

⇒ x = 575

4 24× = 6km

115. If the amount of loan taken fromthe first bank is Rs. x, then

8 1 (1 0 0 0 0 ) 1 01 0 0 1 0 0

x x× × − ×+

= 950⇒ 8x + 100000 - 10x = 95000⇒ 2x = 10000 - 950000 = 5000⇒ x = Rs. 2500

116.2

21a

a+ = 98

⇒21a

a +

- 2 = 98

⇒21a

a +

= 100

⇒1aa

+ = 10

On cubing both sides,

31aa

+

= 1000

⇒ a3 + 31

a + 3 1aa

+

= 1000

⇒ a3 + 31

a = 1000 - 30 = 970

117. x - 1 = 2 3+On squaring,

x2 - 2x + 1 = 2 + 3 + 2 6

⇒ x2 + 2x - 4 = 2 6On squaring again,x4 + 4x2 + 16 + 16 - 4x3 - 8x2 +16x = 24⇒ x4 - 4x3 - 4x2 + 16x - 8 = 0⇒ 2x4 - 8x3 - 8x2 + 32x -16 = 0⇒ 2x4 - 8x3 - 5x2 + 26x - 28 - 3x2

+ 6x + 12 = 0⇒ 2x4 - 8x3 - 5x2 + 26x - 28= 3x2 - 6x - 12= 3(x2 - 2x - 4)

= 3 × 2 6 = 6 6

118. 2 2( 0 ) (0 5 )x − + + = 13

⇒ x2 + 25 = 169⇒ x2 = 169 - 25 = 144

∴ x = 1 44 = 12119. 4x = 18y

⇒18 94 2

xy

= =

∴ 9 71 1

4 2 2x − = − =

120.1xx

+ = 5

⇒ x2 - 5x + 1 = 0⇒ x2 + 3x + 1 = 2x

42

2

1

3 1

xx

x x

+

− +

= 12

42

1xx

x

+

33

1 12

xx

+

= 31 1 13

2x x

x x

+ − +

=1 (125 3 5 )2

− ×

= 1 1102

× = 55

121. x = 3 + 3 , y = 2 - 3x + u = 4; xy = 4 - 3 = 1

∴ 2 2

3 3x yx y

+

+

= 2

3( ) 2

( ) 3 ( )x y xy

x y xy x y+ −

+ − +

= 1 6 2

6 4 3 4−

− × = 1 45 2 =

72 6

122. a2 + b2 + c2 = 2(a - b - c) - 3⇒ a2 + b2 + c2 – 2a - 2b - 2c + 3= 0⇒ a2 + 2a + 1 + b2 + 2b + 2c + 3= 0⇒ a2 - 2a + 1 + b2 + 2b + 1 + c2 +

Page 34: Download SSC CGL Tier I Last 5 Year Papers e Book Www.sscportal.in (1)

1.16: Solved Paper 08-07-2012 (1st Sitting)

Join Online Coaching for SSC Exams:http://sscportal.in/community/courses

Buy Printed Study Material for SSC Exams:http://sscportal.in/community/study-kit

2c + 1 = 0⇒ (a - 1)2 + (b +1)2 + (c + 1)2 = 0[If x2 + y2 + z2 = 0⇒ x = 0; y = 0, z = 0]∴ a - 1 = 0 ⇒ a = 1b + 1 = 0 ⇒ b = -1c + 1 = 0 ⇒ c = -1∴ 2a - 3b + 4c = 2 + 3 - 4 = 1

123. 2x - 1

2 x = 6

⇒ x = 1

4 x = 3

[On dividing by 2]

⇒ x4 + 21 12

41 6x

xx− × × = 9

⇒ x4 + 21

1 6 x = 1 1992 2

+ =

124. 5a + 1

3a =5

On multiplying by 35 ,

3a + 1

5a = 5 × 35 = 3

On squaring,

9a2 + 21 12 3

52 5a

aa+ × × = 9

⇒ 9a2 + 21

2 5a

= 9 - 65 =

45 64−

= 395

125. 5x + 7y = 35(i)4x + 3y = 12 (ii)By equation (i) × 4 - (ii) × 520x + 28y = 140

20x + 15y = 60– – – 13y = 80⇒ 5x + 7 × 0 = 35, ⇒ 5x = 35⇒ x = 7, ∴ (7, 0)Similarly, point of intersection∴ Base = 7 - 3= 4

∴ Area = 1 8 042 1 3

× ×

=1 6 01 3 sq.unit

126. BAC∠ = 180° - BOC∠= 180° - 54°= 126°

127. Ratio of corresponding sides

=9

16

= 34

128. AF = FNAE = EC∴ FE || BC

= 12 BC

∴ AO : OG= 2 : 1

129.

BAC∠ = 50°

∴ BS C∠ = 100°BS = SC = radius

∴ BCS∠ = 1 (180 100)2

− = 40°

130. It is bsed on fundamental concept.131. PQ2 = (r1 + r2)2 - (r1 - r2)2

=4r1r2

132. BOC∠ = 2 BAC∠OB + OC

∴ OBC∠ = OCB∠

∴ OBC∠ = 90° - 2BOC∠

= 90°- BAC∠

∴ BAC∠ + OBC∠

= 90° - BAC∠ + BAC∠= 90°

133.

∴ AP : AQ = 5 : 8

134. BIC∠ = 90° + 2A

= 90° + 30° = 120°

135. BPC∠ = 90° - 2A

= 90° - 40° = 50°136. s = 16 cm

r = 50 m

∴ θ = sr =

165 0

= 8

25 radian

Page 35: Download SSC CGL Tier I Last 5 Year Papers e Book Www.sscportal.in (1)

1.17: Solved Paper 08-07-2012 (1st Sitting)

Join Online Coaching for SSC Exams:http://sscportal.in/community/courses

Buy Printed Study Material for SSC Exams:http://sscportal.in/community/study-kit

= 8

25 × 180°

π

= 8

25 × 180 100872 2 55

× =

= 1 81 85 5

°

= 18° 18 60 '55

×

18 2 0 '≈ °

137. sin (2x - 20°) = cos (2y + 20°)⇒ sin (2x - 20°)= sin (90° - 2y - 20°)= sin (70° - 2y)⇒ 2x - 20° - 70° - 2y⇒ 2(x + y) = 90°⇒ x + y = 45°∴ sec (x + y) = sec 45°

138. 5 tan θ = 4

∴ 5 s in 3 cos5 s in 3 cos

θ − θθ + θ

=

5sin 3coscos cos5sin 3coscos cos

θ θθ θθ θθ θ

+

= 5 tan 35 tan 3

θθ

−+

=

45 3545 35

× −

× + =

17

139. 4sec2 θ + 9cosec2 θ= 4(1 + tan2 θ ) + 9(1 + cot2 θ )= 4 + 4 tan2 θ + 9 + 9 cot2 θ= 13 + 4 (tan2 θ + cot2 θ ) +5cot2 θ= 13 + 4 [tna θ - cos θ )2 + 2] + 5cot2 θ= 13 + 8 + 4 (tan θ - cot θ )2 +5cot2 θ (tan θ - cot θ )2 ≥ 0

140. tan (x + y), tan (x - y) = 1

⇒ tan (x + y) = cot (x - y)= tan (90° - x + y)⇒ x + y = 90° - x + y⇒ 2x = 90°

∴ tan 23x

= tan 30° = 13

141. x2y2 (x2 + y2 + 3)= (cosec θ - sin θ )2 (sec θ -cos θ )2

[(cosec θ - sin θ )2 + (sec θ -cos θ )2 + 3]

= 2 21 1

s in coss in cos

− θ − θ θ θ

2 21 1 coss in 3s in cos

− θ − θ + θ θ

=

22 21 s in 1 coss in cos

− θ − θ θ θ

2 22 21 s in 1 cos 3s in cos

− θ − θ + + θ θ

=

2 22 2cos s ins in cos

θ θ θ θ

2 2cos s in 3s in cos

θ θ + + θ θ

= cos2 θ × sin2 θ

2 6 2 2

2 2cos s in 3 cos . s in

cos . s in

θ + θ + θ θ θ θ

=cos6 θ + sin6 θ + 3cos2 θ sin2 θ

= ( ) ( ){ }3 32 2cos s inθ + θ

+ 3cos2 θ .sin2 θ=(cos2 θ + sin2 θ )3

- 3cos2 θ .sin2 θ(cos2 θ + sin2 θ ) + 3 cos2 θ .sin2 θ=1-3cos2 θ .sin2 θ + 3

cos2 θ .sin2 θ = 1142. 2y cos θ = x sin θ

⇒ x sec θ = 2y = cosec θ∴ 2x sec θ - y cosec θ = 3

⇒ 4y cosec θ - y cosec θ = 3

⇒ 3y cosec θ = 3

⇒ y cosec θ = 1

⇒ y = sin θ∴ sec θ = 2y cosec θ= 2sin θ .cosec θ = 2

⇒ x = 2cos θ∴ x2 + 4y2 = 4cos2 θ = 4

143. AB = Pole = h metreBD = x metre

From ABC∆ ,

tan 30° = 5h

x +

⇒1

53h

x=

+

⇒ x + 5 = 3h (i)

From ABD∆ ,

tan 60° = hx

⇒ 3 = hx

⇒ x = 3h

∴ x + 5 = 3h

⇒ h + 5 3

Page 36: Download SSC CGL Tier I Last 5 Year Papers e Book Www.sscportal.in (1)

1.18: Solved Paper 08-07-2012 (1st Sitting)

Join Online Coaching for SSC Exams:http://sscportal.in/community/courses

Buy Printed Study Material for SSC Exams:http://sscportal.in/community/study-kit

⇒ 2h = 5 3

⇒ h = 5 3

2metre

144. θ = 25° = 2 51 8 0

× πradius

= 536

π radians

θ = st

⇒ r = st =

40536

π = 40 36

5×π

⇒4 0 3 6 7

5 22× ×× metre

= 91.64 metre145. sin θ + sin2 θ = 1

⇒ sin θ = 1 - sin2 θ⇒ sin θ = cos2 θ\cos12 θ + 3cos10 θ + 3cos8 θ +cos6 θ -1= (cos4 θ + cos2 θ )3 – 1= (sin2 θ + cos2 θ )3 – 1 = 1 - 1= 0

146. Q72 lakh = 360°

\ 12 lakh = 360 1272

×

= 60° = S6147. Population of region S1

= 45 72

360× = 9 lakhs

Males = 3 95

× = 5.4 lakhs

Population of region S4

= 1 0 5 7 23 6 0

× = 21 lakh

Male = 2 2 15

× = 8.4 lakhs

Sum = 5.4 + 8.4 = 13.8 lakhs148. Population of region S2

=1 3 5 7 23 6 0

× = 27 lakhs

Females = 1 2 75

× = 5.4 lakhs

Population of region S5

= 60 72

360× = 12 lakhs

Females = 7 12

20× = 4.2 lakhs

Required ratio =5.4 : 4.2 = 9 : 7

149. Population in 2009

Region S1 = 1 0 091 0 5

×

= 607 lakhs

Region S2

= 1 0 0 1 5 7 21 0 8 3 6 0

× ×

=2 59 lakhs

∴ Required ratio = 60 25,7 9

=12 5:7 9 = 108 : 35

150. Number of females:Region S1 = 9 - 5.4 × 3.6 lakhsRegion S2 = 5.4 lakhs

Region S3 ⇒33

10×

= 0.9 lakhsRegion S4 = 21 - 8.4 = 12.6 lakhsRegion S6 = 4.2 lakhsAverage = 5.35 lakhs

Page 37: Download SSC CGL Tier I Last 5 Year Papers e Book Www.sscportal.in (1)

Study Kits for Staff Selection Commission Exams

Study Kit for SSC CGL EXAM (Tier-I)

http://sscportal.in/community/study-kit/cgl

Study Kit for SSC Combined Graduate Level Examination (Tier - II)

http://sscportal.in/community/study-kit/cgl-tier-2

Study Kit for Combined Higher Secondary Level (10+2) Examination

http://sscportal.in/community/study-kit/chsle

आ ( :-1) http://sscportal.in/community/study-kit/cgl/tier-1-hindi

Study Kit For SSC MTS Examination

http://sscportal.in/community/study-kit/mts

Study Kit For Prasar Bharti Examination

http://sscportal.in/community/study-kit/prasar-bharti-exam

Study Kit for Central Armed Police Forces (CAPFs) Examination

http://sscportal.in/community/study-kit/capfs

Study Kit For Sub Inspector in Delhi Police & CAPFs, CISF (Paper-1)

http://sscportal.in/community/study-kit/si-delhi-police-capf

Study Kit of English Language & Comprehension For All SSC Exam

http://sscportal.in/community/study-kit/english-language-comprehension-for-all-

ssc-exam

Study Kit For Postal Assistant Examination

http://sscportal.in/community/study-kit/postal-assistant

For More Information Click Given below link:

http://sscportal.in/community/study-kit

Page 38: Download SSC CGL Tier I Last 5 Year Papers e Book Www.sscportal.in (1)

SOLVED PAPERSSC-CGL

PRE. EXAM 2013Held On: 19-05-2013 (Morning Shift)

Directions (Q. Nos. 1-4) Select therelated letters/ words / numbers from thegiven alternatives.

1. Tanning : Leather : : Pyrotechnics: ?(a) Bombs (b) Fireworks(c) Wool (d) Machinery

2. King : Throne : : Rider : ?(a) Chair (b) Horse(c) Seat (d) Saddle

3. F : 216 : : L : ?(a) 1728 (b) 1700(c) 1600 (d) 1723

4. MOUSE : KPSTC : : LIGHT : ?(a) MGHFU (b) JGEFR(c) JJEIR (d) MJHIU

5. If 264 * 2 = 6, 870 * 3 = 11, thenwhat should 735 * 5 be?(a) 12 (b) 16 (c) 03 (d) 05

6. Which one of the given responsewould be a meaningful order of thefollowing?1. Implementation2. Conceptual Modelling3. Requirements Analysis4. Logical Modelling5. Physical Model6. Schema Refinement(a) 3, 2, 4, 6, 5, 1(b) 1, 3, 2, 6, 5, 4(c) 3, 2, 5, 4, 6, 1(d) 3, 2, 1, 4, 6, 5

Directions (Q. Nos. 7 and 8) Which oneset of letters when sequentially placedat the gaps in the given letter series shallcomplete it?

GENERAL INTELLIGENCE AND REASONING7. b_ab_b_aab_b

(a) abbb (b) abba(c) baaa (d) aabb

8. _bcab_cabc_abca_b(a) abca (b) aabc(c) bbca (d) abac

Directions (Q. Nos. 9-13) Find the oddword/number/number-pair from thegiven alternatives.

9. (a) Head (b) Heed(c) Led (d) Heap

10. (a) Scurvy(b) Rickets(c) Night-blindness(d) Influenza

11. (a) Rain(b) Shower(c) Sleet(d) Raisin

12. (a) 547-563 (b) 71-55(c) 517-523(d) 248-231

13. 10, 13, 234, 681, 997(a) 10 (b) 234(c) 13 (d) 681

Directions (Q. Nos, 14-18) A series isgiven, with one term missing. Choosethe correct alternative from the givenones that will complete the series.

14. AFI, JOR, MRU, ?(a) GJN (b) HMP(c) PMO (d) RJL

15. bc, cde, de, efg, fg, ?(a) ghi (b) fgh(c) hij (d) ijk

16. C - 3, E - 5, G - 7, I - 9, ?,(a) X - 24, M - 21(b) K - 11, M - 13(c) O - 15, X - 24(d) M - 18, K - 14

17. 5, 16, 51, 158, ?(a) 1452 (b) 483(c) 481 (d) 1454

18. 9 * 2 : 9 * 9 : : 9 * 5 : ?(a) 9 × 4 (b) 9 × 6(c) 9 × 7 (d) 9 × 8

19. Priti scored more than Rahul.Yamuna scored as much as Divya.Lokita scored less than Manju.Rahul scored more than Yamuna.Manju scored less than Divya.Who scored the lowest?(a) Manju (b) Yamuna(c) Lokita (d) Rahul

20. A man showed a boy next to himand said “He is the son of mywife’s sister-in-law but I am theonly child of my parents.” How ismy son related to him?(a) Cousin (b) Brother(c) Uncle (d) Nephew

21. Find the correct group of signs tosolve the equation.24 * 16 * 8 * 32(a) ÷ – = (b) – + =(c) × ÷ = (d) + – =

22. Sunil is the son of Kesav. Simran,Kesav’s sister, has a son Marutiand daughter Sita. Prem is thematernal uncle of Maruti. How isSunil related to Maruti?

Page 39: Download SSC CGL Tier I Last 5 Year Papers e Book Www.sscportal.in (1)

1.2: Solved Paper (19-05-2013) (1st Sitting)

Join Online Coaching for SSC Exams:http://sscportal.in/community/courses

Buy Printed Study Material for SSC Exams:http://sscportal.in/community/study-kit

(a) Cousin(b) Maternal Uncle(c) Brother (d) Nephew

Directions (Q. Nos. 23-25) From thegiven alternatives, select the wordwhich cannot be formed using theletters of the given word.

23. MANUSCRIPT(a) PRIMUS(b) SMART(c) RUSTIC(d) MASTER

24. SEGREGATION(a) NATION(b) GREAT(c) GREETINGS(d) SEATING

25. CONSTITUTIONAL(a) TUITION(b) TALENT(c) LOCATION(d) CONSULT

26. If each of the letters in the Englishalphabet is assigned an evennumerical value by giving A = 2,B = 4 and so on, what would bethe total value of the letters for theword ‘LADY’ when similarlycoded?(a) 74 (b) 72 (c) 84 (d) 82

27. If the word ‘LEADER’ is coded as20 - 13 - 9 - 12 - 13 - 26, how wouldyou write ‘LIGHT’?(a) 20 - 16 - 17 - 15 - 27(b) 20 - 15 - 16 - 18 - 23(c) 20 - 17 - 15 - 16 - 28(d) 20 - 16 - 15 - 17 - 22

28. If the first and second letters inthe word ‘COMMUNICATIONS’were interchanged, also the thirdand the fourth letters, the fifth andsixth letters and so on, which letterwould be the tenth letter countingfrom your right?(a) U (b) A(c) T (d) N

29. Which of the followinginterchanges of numbers wouldmake the given equation correct?8 × 20 ÷ 3 + 9 – 5 = 38

(a) 3, 8(b) 8, 9(c) 3, 5(d) 3, 9

30. Put the correct mathematical signsin the following equation from thegiven alternatives.33 ? 11 ? 3 ? 6 = 115(a) +, –, ×(b) ×, ÷, –(c) ÷, ×, ×(d) –, ×, +

31. Select the correct combination ofmathematical signs to replace *signs and to balance the givenequation.15 * 24 * 3 * 6 * 17(a) + ÷ – =(b) + × = ÷(c) – × = +(d) – ÷ + =

Directions (Q. Nos. 32-34) Select themissing number from the givenresponses.

32. 84

14 12

81

18 9

88

? 11

(a) 10(b) 12(c) 14(d) 16

33. 3 5 8 74 6 4 65 2 2 3

58 58 62 ?

(a) 126(b) 122(c) 128(d) 124

34. 4 3 26 9 109 27 ?

(a) 30(b) 20(c) 50 (d) 54

35. Raj is standing in the middle of asquare field. He starts walkingdiagonally to North-East. Then, heturns right and reaches the far endof the field. Then, he turns rightand starts walking. In the midway,he again turns right and startswalking. In halfway, he turns tohis left and reaches new far end. Inwhat direction is Raj now?(a) North(b) South(c) North-West(d) South-West

36. A boy starts from home in earlymorning and walks straight for 8km facing the Sun. Then, he takesa right turn and walks for 3 km.Then, he turns right agian andwalks for 2 km and then turns leftand walks for 1 km. Then, he turnsright, travels 1 km and then turnsright and travels for 4 km straight.How far is he from the startingpoint?(a) 5 km(b) 6 km(c) 2 km(d) 4 km

Directions (Q. Nos. 37 and 38) One /I twostatements are given, followed by two /three conclusions I, II and III. You haveto consider the statements to be trueeven if they seem to be at variance fromcommonly known facts. You have todecide which of the given conclusions,if any, follow from the given statements.

37. StatementSick people need medicine.ConclusionsI. Healthy people do not need

medicine.II. People keep medicine in their

home.(a) Only Conclusion I follows(b) Only Conclusion II follows(c) Both Conclusions I and II

follow(d) Neither Conclusion I nor II

follows

Page 40: Download SSC CGL Tier I Last 5 Year Papers e Book Www.sscportal.in (1)

1.3: Solved Paper (19-05-2013) (1st Sitting)

Join Online Coaching for SSC Exams:http://sscportal.in/community/courses

Buy Printed Study Material for SSC Exams:http://sscportal.in/community/study-kit

38. StatementsI. Some years are decades.II. All centuries are decades.ConclusionsI. Some centuries are years.II. Some decades are years.III. No century is a year.(a) Conclusions I and II follow(b) Conclusions I and III follow(c) Only Conclusion I follows(d) Only Conclusion II follows

39. There is a ball and a rectangular jar.Four positions are shown below tokeep then balanced. Which of thefollowing will not be balancedeasily?

40. Which one of the four boxesgiven below is created by foldingthe given key design in thequestion figure?

41. How many cubes are there in thegroup?

(a) 10 (b) 16(c) 18 (d) 20

42. Which one of the followingdiagrams best depicts therelationship among Earth, Sea andSun?

(a)

(b)

(c)

(d)

43. Read the figure and find the regionrepresenting persons who areeducated and employed but notconfirmed.

(a) ac (b) abc(c) bd (d) adc

44. In the given diagram, circlerepresents professionals, squarerepresents dancers, trianglerepresents musicians and rectanglerepresents Europeans. Differentregions in the diagram arenumbered 1 to 11. Who among thefollowing is neither a dancer nor amusician but is professional andnot a European?

(a) 8 (b) 11(c) 1 (d) 10

Directions (Q. Nos. 45 and 46) In thesequestions which answer figures willcomplete the pattern in the questionfigure?

45.

46.

47. Which of the answer figuresincludes the separate componentsfound in the question figure?

48. A piece of paper is folded andpunched as shown below in thequestion figures. From the givenanswer figures, indicate how it willappear when opened.

49. Choose the right water image ofthe question figure from the givenanswer figures.

Page 41: Download SSC CGL Tier I Last 5 Year Papers e Book Www.sscportal.in (1)

1.4: Solved Paper (19-05-2013) (1st Sitting)

Join Online Coaching for SSC Exams:http://sscportal.in/community/courses

Buy Printed Study Material for SSC Exams:http://sscportal.in/community/study-kit

50. A word is represented by only oneset of numbers as given in anyoneof the alternatives. The sets ofnumbers given in the alternativesare represented by two classes ofalphabets as in two matrices givenbelow. The columns and rows ofMatrix I are numbered from 0 to 4

and that of Matrix II are numberedfrom 5 to 9. A letter from thesematrices can be represented firstby its row and next by its column,e.g., ‘D’ can be represented by 02,14 etc. and ‘R’ can be representedby 57, 76 etc. Similarly, you haveto identify the set for the word‘BEST’.

(a) 24, 22, 76, 97(b) 24, 21, 77, 97(c) 24, 22, 77, 97(d) 24, 22, 77, 96

GENERAL AWARENESS51. Social accounting system in India

classified into is(a) enterprise, households and

government(b) assets, liabilities and debt

position(c) public sector, private sector

and joint sector(d) income, product and

expenditure52. Forced savings refer to

(a) taxes on individual income andwealth

(b) compulsory deposits imposedon income tax payers

(c) provident fund contributionof private sector employees

(d) reduction of consumptionconsequent to a rise in prices

53. The demand for labour is called(a) direct demand(b) derived demand(c) factory demand(d) market demand

54. Which of the following is not aninvestment expenditure in goodsand services?(a) Purchase of a house(b) Purchase of a machinery(c) An increase in business

inventories(d) Expansion of the main plant of

a company55. Which one of the following

represents the savings of theprivate corporate sector?(a) Total profits of a company(b) Undistributed profits

(c) Excess of income overexpenditure

(d) Dividends paid to shareholders

56. Who presides over the jointsession of Indian Parliament?(a) President of India(b) Chairperson of Rajya Sabha(c) Senior most Member of

Parliament(d) Speaker of Lok Sabha

57. Division of powers andindependent Judiciary are the twoimportant features of(a) unitary form of Government(b) democratic form of

Government(c) federal form of Government(d) socialist form of Government

58. Which article of the IndianConstitution did Dr BR Ambedkarterm as the ‘Heart and Soul of theIndian Constitution’?(a) Article 19 (b) Article 356(c) Article 32 (d) Article 14

59. Who was the first to use the term‘State’?(a) Plato(b) Aristotle(c) Machiavelli(d) Hobbes

60. Under which article of theConstitution of India, can thefundamental rights of the membersof the Armed Forces bespecifically restricted?(a) Article 19 (b) Article 21(c) Article 25 (d) Article 33

61. Pulakesin II was the greatest rulerof the(a) Chalukyas of Badami(b) Chalukyas of Kalyani(c) Pallavas of Kanchi(d) Cholas of Tamil Nadu

62. The Uttaramerur inscriptionprovides information on theadministration of the(a) Satavahanas(b) Pallavas(c) Cholas (d) Chalukyas

63. Pitts India Act of 1784 was a/an(a) regulating act(b) ordinance(c) resolution (d) white paper

64. Which one of the followingstatements is not true in respect ofAO Hume?(a) He was a member of the

Indian Civil Service(b) He founded the Indian

National Congress(c) He presided over the

Congress Annual Sessionstwice

(d) He was an ornithologist65. Which of the following

movements saw the biggestpeasant guerilla war on the eve ofindependence?(a) Tebhaga Movement(b) Punnapra Vayalar Movement(c) Telangana Movement(d) Noakhali Movement

66. Maps on large scale, representingboth natural and man-madefeatures are called

Page 42: Download SSC CGL Tier I Last 5 Year Papers e Book Www.sscportal.in (1)

1.5: Solved Paper (19-05-2013) (1st Sitting)

Join Online Coaching for SSC Exams:http://sscportal.in/community/courses

Buy Printed Study Material for SSC Exams:http://sscportal.in/community/study-kit

(a) Topographic maps(b) Thematic maps(c) Atlas maps (d) Wall maps

67. Which river in India flows in a rift-valley?(a) Tapti (b) Narmada(c) Krishna (d) Cauvery

68. A narrow str ip of land thatconnects two larger land masses iscalled(a) peninsula (b) cape(c) isthmus (d) strait

69. Tiny marine animals whichconstitute limestone skeletons arecalled(a) foraminifera(b) coral reefs(c) diatoms(d) clamitomonous

70. Apatanis are the major tribal groupof(a) Nagaland (b) Sikkim(c) Arunachal Pradesh(d) Jharkhand

71. Which of the following plantpigments absorbs in red and far-red region of light?(a) Chlorophyll(b) Phytochrome(c) Cryptochrome(d) Carotenoids

72. The process through whichexcess of light energy isdissipated in photosynthesis isknown as(a) scavenging(b) photolysis(c) photophosphorylation(d) quenching

73. AIDS virus destroys(a) rnonocytes (b) neutrophils(c) basophils (d) lymphocytes

74. Teeth and Bones acquire strengthand rigidity from(a) fluorine (b) chlorine(c) sodium (d) calcium

75. The type of tail found in Shark is(a) homocercal(b) heterocercal(c) diphycercal(d) protocercal

76. The Sigmoid Colon is part of(a) anal canal(b) large Intestine(c) ileum(d) small Intestine

77. A good conductor while carryingcurrent is(a) negatively charged(b) positively charged(c) electrically neutral(d) alternately charged positive

and negative78. The angle between the magnetic

meridian and the geographicalmeridian at a place is(a) dip (b) declination(c) latitude (d) azimuth

79. The device used for measuring thewavelength of X-rays is(a) cyclotron(b) bragg spectrometer(c) mass spectrometer(d) GM counter

80. Alpha particle is the nucleus of anatom of(a) hydrogen (b) helium(c) oxygen (d) lthium

81. Who is the founder of ‘Facebook’which is currently the No.1 socialnetworking website in India?(a) Martin Cooper(b) Orkut Buyukkokten(c) Mark Zuckerberg(d) Bill Gates

82. In programming, repeating somestatements is usually called(a) structure (b) looping(c) control structure(d) compiling

83. Silicon is a polymer of(a) Silicon tetrachloride(b) Dialkyl dichloro silane(c) Silane(d) Tetraalkyl silane

84. Which is a natural colloid?(a) Urea (b) Cane-sugar(c) Blood(d) Sodium chloride

85. Which one of the following doesnot contain silver?

(a) Lunar caustic(b) German silver(c) Horn silver(d) Ruby silver

86. The presence of cobalt in vitaminB12 was established for the firsttime by(a) spectroscopy(b) borax-Bead test(c) sodium nitroprusside test(d) hydrolysis test

87. Which bacterial strain developedfrom natural isolates by geneticmanipulations can be used fortreating oil spills?(a) Agrobacterium(b) Clostridum(c) Nitrosomonas(d) Pseudomonas

88. Coating of solid waste withimpervious material is known as(a) Chemical fixation(b) landfill(c) capping(d) encapsulation

89. Ultraviolet rays can used in watertreatment as(a) precipitator(b) hydrolyser(c) disinfectant(d) flocculator

90. Thiamidine dimer formation inDNA is caused by(a) X-rays (b) β and γ -rays(c) UV-rays (d) IR-rays

91. Winner of the Australian OpenMen’s Singles Title in 2013 is(a) Andy Murray(b) Rogaer Federer(c) Novak Djokovic(d) David Ferrer

92. Who among the followingPresidents of MCC was a non-professional cricketer?(a) Colin Cowdrey(b) Peter May(c) Christopher Martin Jenkin(d) Ted Dexter

93. Who won the Nobel Prize forPeace in 2012?

Page 43: Download SSC CGL Tier I Last 5 Year Papers e Book Www.sscportal.in (1)

1.6: Solved Paper (19-05-2013) (1st Sitting)

Join Online Coaching for SSC Exams:http://sscportal.in/community/courses

Buy Printed Study Material for SSC Exams:http://sscportal.in/community/study-kit

(a) UN Intergovernmental Panelon Climate Change

(b) International Atomic EnergyAgency

(c) Liu Xiaobo(d) European Union

94. Who won the Jnanpith Award for2011?(a) Sitakant Mahapatra(b) Sachidananda Routray(c) Pratibha Ray(d) Gopinath Mohanty

95. Who is the author of the book“No Full Stops in India”?(a) Ved Mehta

(b) Nirad C Choudhuri(c) Mark Tully (d) RK Narayan

96. Which one of the following pairsis wrongly matched?(a) Red Square - Moscow(b) Tiananmen Square – Beijing(c) Tahrir Square - Abu Dhabi(d) Trafalgar Square - London

97. The term of a non-permanentmember of the UN SecurityCouncil is(a) 6 months (b) 1 yr(c) 2 yr (d) 3 yr

98. Who is the Chairman of the 20thLaw Commission?

(a) Justice JS Verma(b) Justice KG Balakrishnan(c) Justice DK Jain(d) Justice Usha Mehra

99. Julia Gillard is the Prime Ministerof(a) Belgium(b) Canada(c) Australia(d) New Zealand

100. Which is the first state in India topass the Food Security Law?(a) Kerala(b) Chhattisgarh(c) Gujarat (d) Punjab

QUANTITATIVE APTITUDE101. A and B together can complete a

piece of work in 12 days, B and Ccan do it in 20 days and C and Acan do it in 15 days. A, B and Ctogether can complete it in(a) 6 days(b) 8 days(c) 10 days(d) 12 days

102. A and B together can complete awork in 3 days. They starttogether but, after 2 days, B leftthe work. If the work is completedafter 2 more days, B alone coulddo the work in(a) 4 days (b) 6 days(c) 8 days (d) 10 days

103. A does 20% less work than B. If Acan complete a piece of work in

172

h, then B can do it in

(a) 4 h (b) 6 h(c) 8 h (d) 10 h

104. A rational number between 3/4 and3/8 is(a) 7/3 (b) 16/9(c) 9/16 (d) 1/7

105. Product of two coprime numbers is117. Then their LCM is(a) 9 (b) 13(c) 39 (d) 117

106. The diameters of two circles arethe side of a square and thediagonal of the square. The ratioof the areas of the smaller circleand the larger circle is(a) 1 : 4(b) 2 : 3(c) 1: 2 (d) 1 : 2

107. The total surface area of a sphereis 8π square unit. The volume ofthe sphere is(a) 8/3 π cu unit(b) 8 3 π cu unit

(c)8 3

5 π cu unit

(d)8 2

3 π cu unit

108. A conical flask is full of water. Theflask has base radius r and heighth. This water is poured into acylindrical flask of base radius mr.The height of water in thecylindrical flask is

(a)2h

m2 (b)2hm

(c) 23hm

(d)2mh

109. A dozen pairs of socks quoted atRs. 180 are available at discount of20%. How many pairs of socks canbe bought for Rs. 48?(a) 4 pairs (b) 2 pairs(c) 5 pairs (d) 3 pairs

110. The marked price of a table is Rs.12000. If it was sold for Rs. 10500after allowing a certain discount,then the rate of discount is(a) 10% (b) 12.5%(c) 15% (d) 17.5%

111. The marked price of a radio set isRs. 480. The shopkeeper allowsdiscount of 10% and gains 8%. Ifno discount is allowed, his gainpercent would be(a) 18% (b) 18.5%(c) 20% (d) 25%

112. The prices of a school bag and ashoe are in the ratio 7 : 5. The priceof school bag is Rs. 200 more thanthe price of a shoe. Then, the priceof a shoe is(a) Rs. 700 (b) Rs. 500(c) Rs. 1200 (d) Rs. 200

113. A sum of Rs. 300 is divided amongP, Q and R in such a way that Qgets Rs. 30 more than P and Rgets Rs. 60 more than Q. The ratioof their shares is

Page 44: Download SSC CGL Tier I Last 5 Year Papers e Book Www.sscportal.in (1)

1.7: Solved Paper (19-05-2013) (1st Sitting)

Join Online Coaching for SSC Exams:http://sscportal.in/community/courses

Buy Printed Study Material for SSC Exams:http://sscportal.in/community/study-kit

(a) 2 : 3 : 5 (b) 3 : 2 : 5(c) 2 : 5 : 3 (d) 5 : 3 : 2

114. The average of nine numbers is 50.The average of the first fivenumbers is 54 and that of the lastthree numbers is 52. Then, thesixth number is(a) 34 (b) 24(c) 44 (d) 30

115. The average of the first nineintegral multiples of 3 is(a) 12 (b) 15 (c) 18 (d) 21

116. An article is sold for Rs. 300 at aprofit of 20%. Had it been sold forRs. 235, the loss percentage wouldhave been(a) 3 (b) 5(c) 6 (d) 16

117. A box has 100 blue balls, 50 redballs and 50 black balls. 25% ofblue balls and 50% of red balls aretaken away. Then, percentage ofblack balls at present is

(a) 25% (b)133 %3

(c) 40% (d) 50%118. Two cars are moving with speeds

v1 and v2 towards a crossing alongtwo roads. If their distance fromthe crossing be 40 m and 50 m atan instant of time, then they donot collide, if their speeds are suchthat(a) v1 : v2 ≠ 4 : 5(b) v1 : v2 ≠ 5 : 4(c) v1 : v2 = 25 : 16(d) v1 : v2 = 16 : 25

119. A certain distance is covered at acertain speed. If half of thisdistance is covered in double thetime, the ratio of the two speeds is(a) 4 : 1 (b) 1 : 4(c) 2 : 1 (d) 1 : 2

120. The simple interest on Rs. 4000 in3 yr at the rate of x% per annumequals to the simple interest onRs. 5000 at the rate of 12% perannum in 2 yr. The value of x is(a) 6% (b) 8%(c) 9% (d) 10%

121. If x2 – 3x + 1 = 0, then value of

x2 + x + 1x

+ 2

1x

is

(a) 2 (b) 6 (c) 8 (d) 10

122. If 4 – 3x

x +

4 – 3yy +

4 – 3zz

= 0,

then the value of 1x

+ 1y +

1z

is

(a) 3 (b) 4 (c) 6 (d) 9123. Find the simlest value of 2 50 +

18 – 72 (given 2 = 1414).(a) 9.898 (b) 10.312(c) 8.484 (d) 4.242

124. If a2 + b2 + 4c2 = 2(a + b – 2c) – 3and a, b, c are real, then the valueof (a2 + b2 + c2) is

(a)1

34

(b) 2

(c)1

24

(d) 3

125. A number x when divided by 289leaves 18 as the remainder. Thesame number when divided by 17leaves y as a remainder. The valueof y is(a) 2 (b) 3 (c) 1 (d) 5

126. An equation of the form ax + by +c = 0. Where, a ≠ 0, b ≠ 0 andc = 0 represents a straight linewhich passes through(a) (2, 4) (b) (0, 0)(c) (3, 2)(d) None of these

127. The numerator of a fraction is 4less than its denominator. If thenumerator is decreased by 2 andthe denominator is increased by 1,then the denominator becomeseight times the numerator. Find thefraction.

(a)37

(b)48

(c)27

(d)38

128. If x2 = y + z, y2 = z + x and z2 =

x + y, then the value of 1

1 x+ +

11 y+ +

11 z+

is

(a) 1 (b) 2(c) 0 (d) – 1

129. In a ∆ABC, ∠A = 90°, ∠C = 55°,

AD BC⊥ . What is the value of∠BAD?(a) 60° (b) 45°(c) 55° (d) 35°

130. If G is the centroid of ∆ABC and∆ABC = 48 cm2, then the area of∆BGC is(a) 8 cm2 (b) 16 cm2

(c) 24 cm2 (d) 32 cm2

131. The diagonals AC and BD of acyclic quadrilateral ABCDintersect each other at the point P.Then, it is always true that(a) AP . CP = BP . DP(b) AP . BP = CP . DP(c) AP . CD = AB . CP(d) BP . AB = CD . CP

132. N is the foot of the perpendicularfrom a point P of a circle withradius 7 cm, on a diameter AB ofthe circle. If the length of thechord PB is 12 cm, the distance ofthe point N from the point B is

(a)2

127

cm (b)5

37

cm

(c)2

107

cm (d)5

67

cm

133. If O be the circumcentre of a ∆PQRand ∠QOR = 110°, ∠OPR = 25°,then the measure of ∠PRQ is(a) 50° (b) 55°(c) 60° (d) 65°

134. A vertical stick 12 cm long casts ashadow 8 cm long on the ground.At the same time, a tower casts ashadow 40 m long on the ground.The height of the tower is(a) 60 m (b) 65 m(c) 70 m (d) 72 m

Page 45: Download SSC CGL Tier I Last 5 Year Papers e Book Www.sscportal.in (1)

1.8: Solved Paper (19-05-2013) (1st Sitting)

Join Online Coaching for SSC Exams:http://sscportal.in/community/courses

Buy Printed Study Material for SSC Exams:http://sscportal.in/community/study-kit

135. A, B, C, D are four points on acircle. AC and BD intersect at apoint E such that ∠BEC = 130° and∠ECD = 20°. Then, ∠BAC is(a) 90° (b) 100°(c) 110° (d) 120°

136. In a triangle, if three altitudes areequal, then the triangle is(a) equilateral(b) right(c) isosceles(d) obtuse

137. The tops of two poles of height 24m and 36 m are connected by awire. If the wire makes an angle of60° with the horizontal, then thelength of the wire is(a) 8 3 m (b) 8 m

(c) 6 3 m (d) 6 m

138. The value of 1

cosec – cotθ θ –

1sinθ

is

(a) cot θ(b) cosec θ(c) tan θ(d) 1

139. If cos θ + sin θ = 2 cos θ, thencos θ – sin θ is(a) – 2 cos θ

(b) – 2 sin θ

(c) 2 sin θ

(d) 2 tan θ140. If tan α = n tan β and sin α Q = m

sin β, then cos2 α is

(a)2

2

mn

(b)2

2

–1–1

mn

(c)2

2

11

mn

++

(d)2

2 1m

n +

141. If cos4 θ – sin4 θ = 2 ,3

then the

value of 1 – 2 sin2 θ is

(a) 0 (b)23

(c)13

(d)43

142. If tan θ = 34

and θ is acute, then

cosec θ is equal to

(a)53

(b)54

(c)43

(d)45

143. The value of 2

1(1 tan )θ+ +

2

1(1 cot )θ+ is

(a) 1 (b) 2

(c)12

(d)14

144. The selling price of 20 articles isequal to the cost price of 22articles. The gain percentage is(a) 12%(b) 9%(c) 10%(d) 11%

145. A man gains10% by selling anarticle for a cetain price. If he sellsit at double the price, then theprofit made is(a) 120%(b) 20%(c) 40%(d) 100%

146. A number increased by 1

137 %2

and the increment is 33. Thenumber is(a) 27(b) 22(c) 24 (d) 25

Directions (Q. Nos. 147-150) Thefollowing pie chart shows the marksscored by a student in differentsubjects- viz. Physics (Ph), Chemistry(Ch), Mathematics (M), Social Science(SS) and English (E) in an examination.Assuming that total marks obtained forthe examination is 810, answer thequestions given below.

Chem 70°

Phy85°

Eng 60°

Social 55°

Math 90°

147. The marks obtained in English,Physics and Social Science exceedthe marks obtained inMathematics and Chemistry by(a) 10%

(b)1

10 %9

(c) 11%

(d)1

11 %9

148. The difference of marks betweenPhysics and Chemistry is same asthat between(a) Physics and English(b) Mathematics and English(c) English and Social Science(d) Chemistry and Social Science

149. The marks obtained inMathematics and Chemistryexceed the marks obtained inPhysics and Social Science by(a) 30(b) 40(c) 45(d) 50

150. Thu subject in which the studentobtained 135 marks is(a) Physics(b) Chemistry(c) Mathematics(d) English

Page 46: Download SSC CGL Tier I Last 5 Year Papers e Book Www.sscportal.in (1)

1.9: Solved Paper (19-05-2013) (1st Sitting)

Join Online Coaching for SSC Exams:http://sscportal.in/community/courses

Buy Printed Study Material for SSC Exams:http://sscportal.in/community/study-kit

GENERAL ENGLISHDirections (Q. Nos. 151-155) In thesequestions some parts of the sentenceshave errors and some are correct. Findout which part of a sentence has anerror. If a sentence is free from error,your answer is (d).151. We have been knowing (a)/ each

other (b)/ since we were children.(c)/ No error (d)

152. Neither of the teams (a)/ aresensible enough (b)/ to do thistask. (c)/ No error (d)

153. If I was he (a)/ I wouldn’t accept(b)/ this project. (c)/ No error (d)

154. The teacher advised to (a)/ thestudent to borrow (b)/ a book fromthe library within three days. (c)/No error (d)

155. I insisted (a)/ on his going (b)/there immediately. (c)/ No error (d)

Directions (Q. Nos. 156-160) In thesequestions sentences are given withblanks to be filled with an appropriateword(s). Four alternatives are suggestedfor each question. Choose the correctalternative out of the four.156. The Information and

Communication Technology has_____ age and employes veryhighly paid technocrats.(a) come of(b) come upon(c) come out of(d) come through

157. They reached the railway stationbefore the train __________ .(a) had left (b) had been left(c) left (d) was leaving

158. He slipped _______ his old waysand started drinking again.(a) into (b) off(c) by (d) in

159. All is not well _______ theautomobile sector.(a) of (b) down(c) in (d) to

160. Many premier educationalinstitutions come forward to havea _______ with flourshingindustries.(a) tie-down (b) tie-up(c) tie-in (d) tie-on

Directions (Q. Nos. 161-163) In thesequestions out of the four alternatives,choose the one which best expressesthe meaning of the given word.161. Stern

(a) Lenient (b) Young(c) Stem (d) Strict

162. Citadel(a) Metropolis (b) Mansion(c) Fortress (d) Palace

163. Aberration(a) Justification(b) Intensification(c) Deviation(d) Rationality

Directions (Q. Nos. 164-166) In thesequestions choose the word opposite inmeaning to the given word and mark it.164. Dormant

(a) Active (b) Inactive(c) Dorsal (d) Domestic

165. Tranquility(a) Disturbance(b) Quiet(c) Serenity(d) Peace

166. Dwindle(a) Decrease (b) Diminish(c) Shrink (d) Increase

Directions (Q. Nos. 167-171) In thesequestions four alternatives are given forthe Idiom/ Phrase bold in the sentence.Choose the alternative which bestexpresses the meaning of the Idiom/Phrase.167. Acquiring a job a cakewalk for a

student who has good academicperformance coupled with thegood attitude.

(a) walk away with a cake(b) a difficult achievement(c) a walkway made with cakes(d) an easy achievement

168. Let sleeping dogs lie.(a) Do not bring up an old

controversial issue(b) Dogs can raise tempers(c) Do not allow dogs to stand(d) Prevent dog mobility

169. To get admission in present dayeducational institutions, allchildren should be be born with asilver spoon in the mouth.(a) be born to silver spoon

manufacturer(b) always hold a silver spoon(c) be born with silver spoon(d) be born in a rich family

170. A man of straw means(a) a worthy fellow(b) an unreasonable person(c) a man of no substance(d) a very active person

171. Children complain about theirparents’ gifts. They should learnnot to look gift horse in themouth.(a) not to look at a horse’s mouth(b) not to find fault with the gifts

received(c) not to ask for more gift(d) not to find goodness in the

giftsDirections (Q. Nos. 172-181) In thesequestions a sentence/ part of thesentence is bold. Below are givenalternatives to the bold sentence/ part ofthe sentence at (a), (b) and (c) whichmay improve the sentence. Choose thecorrect alternatives. In case noimprovement is needed, your answer is(d).172. If I had the money I would have

bought the house.(a) If I had had the money I would

have bought the house

Page 47: Download SSC CGL Tier I Last 5 Year Papers e Book Www.sscportal.in (1)

1.10: Solved Paper (19-05-2013) (1st Sitting)

Join Online Coaching for SSC Exams:http://sscportal.in/community/courses

Buy Printed Study Material for SSC Exams:http://sscportal.in/community/study-kit

(b) If I have the money I wouldhave bought the house

(c) If I have had the money Iwould have bought the house

(d) No improvement173. I wish I knew what is wrong with

my car.(a) I wish I had known what is

wrong with my car(b) I wish I know what is wrong

with my car(c) I wish I knew what was wrong

with my car(d) No improvement

174. Just before he died, Amar, who isa poet, wrote this poem.(a) Just before he died, Amar, who

was a poet, wrote this poem(b) Amar, who is a poet, wrote

this poem just before he died(c) Amar wrote this poem, who is

poet, just before he died(d) No improvement

175. The flag will be risen on the 15thof August.(a) The flag will be roused on the

15th of August(b) The flag will be rising on the

15th August(c) The flag will be raised on the

15th of August(d) No improvement

176. Sushma has wisdom, charm andshe has a good sense of humour.(a) Sushma had a good sense of

humour(b) a good sense of humour(c) has a good sense of humour(d) No improvement

177. They knocked down ten houseswhen they built the new road.(a) ruptured(b) removed(c) pulled down(d) No improvement

178. Don’t sit in the grass. It’s wet.(a) by the side of(b) on(c) beside(d) No improvement

179. The game is more important thanthe winning of the prize.(a) The gaming is more important

than the winning of the prize(b) The game is more important

than winning of the prize(c) Gaming is more important than

winning of the prize(d) No imporovement

180. The actor is out of jail, and notexactly a free man, since he willbe under house arrest for anadditional 90 days.(a) but not exactly a free man(b) though exactly a free man(c) if not exactly a free man(d) No improvement

181. Some players on the team sufferfrom chronic knee problems andwill not play in the next playoff.(a) Some player on the team

suffered(b) Some player on the team suffer(c) Some players on the team

suffers(d) No improvement

Directions (Q. Nos. 182-188) In thesequestions out of the four alternatives,choose the one which can besubstituted for the given words/sentence.182. To reduce to nothing

(a) Cull (b) Lull(c) Null (d) Annul

183. An obviously true or hackneyedstatement(a) Truism (b) Syllogism(c) Iconic (d) Imagism

184. Words inscribed on a tomb(a) Epilogue (b) Epitaph(c) Epitome (d) Epistle

185. The act of producing beautifulhandwriting using a brush or aspecial pen(a) Hieroglyphics(b) Calligraphy(c) Stencilling(d) Graphics

186. A word composed of the firstletters of the words in a phrase.

(a) Acronym(b) Abridgement(c) Almanac(d) Anachronism

187. A person of obscure position whohas gained wealth.(a) Extrabagant(b) Promiscuous(c) Parvenu(d) Sumptuary

188. A study of sounds is known as(a) semantics (b) sylistics(c) linguistics (d) phonetics

Directions (Q. Nos. 189-190) In thesequestions, four words are given in eachquestion, out of which only one word iscorrectly spelt. Find the correctly speltword.189. (a) Busyness

(b) Bussyness(c) Business(d) Bussiness

190. (a) Questionnaire(b) Questionnair(c) Questionaire(d) Questionnare

Directions (Q. Nos. 191-200) In thesequestions you have a passage with 10questions. Read the passage carefullyand choose the best answer to eachquestion out of the four.

The postmaster first took up hisduties in the village of Ulapur. Thoughthe village was a small one, there was anindigo factory nearby and the propritor,an Englishman, had managed to get apost office established.

Our postmaster belonged toCalcutta. He felt like a fish out of water inthis remove village. His office and living-room were in a dark thatched shed, notfar from a green, silmy pond, surroundedon all sides by a dense growth.

The men employed in the indigofactory had no leisure, moreover theywere hardly desirable companions fordecent folk. Nor is a Calcutta boy anadept in the art of associating withothers. Among strangers he appears

Page 48: Download SSC CGL Tier I Last 5 Year Papers e Book Www.sscportal.in (1)

1.11: Solved Paper (19-05-2013) (1st Sitting)

Join Online Coaching for SSC Exams:http://sscportal.in/community/courses

Buy Printed Study Material for SSC Exams:http://sscportal.in/community/study-kit

either proud or ill at ease. At any rate thepostmaster had but little company, norhad he much to do.

At times he tr ied his hand atwriting a verse or two. That themovement of the leaves and clouds ofthe sky were enough to fill life with joy -such were the sentiments to which hesought to give expression. But Godknows that the poor fellow would havefelt it as the gift of a new life, if somegenie of the Arabian Nights had in onenight swept away the trees, leaves andall, and replaced them with amacadamised road, hiding the cloudsfrom view with rows of tall houses.191. What does the idiom ‘fish out of

water’ suggest?(a) In unfamiliar surroundings(b) can die any moment(c) grasping for breath(d) amphibious creature

192. Find a word in the passage whichis the opposite of ‘near’.

(a) Convenient(b) Unknown(c) Close (d) Remote

193. Find a word in the passage whichmeans ‘the owner of a business’.(a) Constructor(b) Businessman(c) Entrepreneur(d) Proprietor

194. The adjective used to describe thepostmaster’s living-room is(a) bright(b) dark(c) light(d) deep

195. What does the phrase ‘ill at ease’in the passage mean?(a) Forward(b) Disease(c) Comfortable(d) Uneasy

196. What does the phrase ‘littlecompany’ in the passage mean?

(a) Bad friendship(b) Hardly any friends(c) Small business(d) Business-like

197. At times, the postmaster wrote(a) poems(b) novels(c) short stories(d) dramas

198. The postmaster wrote on the(a) beauty of nature(b) beauty of himself(c) beauty of the weather(d) beauty of the village

199. The word ‘genie’ means(a) monster (b) spirit(c) ghost (d) soul

200. Which factory was situated nearthe village Ulapur?(a) Chemical(b) Rubber(c) Clothes(d) Dyes

ANSWERS1. (b) 2. (c) 3. (a) 4. (c) 5. (a) 6. (c) 7. (d) 8. (a) 9. (b) 10. (d)

11. (d) 12. (d) 13. (d) 14. (b) 15. (a) 16. (b) 17. (c) 18. (b) 19. (c) 20. (a)21. (d) 22. (c) 23. (d) 24. (a) 25. (b) 26. (c) 27. (c) 28. (d) 29. (c) 30. (b)31. (a) 32. (d) 33. (d) 34. (c) 35. (d) 36. (a) 37. (c) 38. (a) 39. (b) 40. (c)41. (d) 42. (b) 43. (c) 44. (d) 45. (c) 46. (d) 47. (b) 48. (c) 49. (d) 50. (c)51. (c) 52. (d) 53. (c) 54. (c) 55. (a) 56. (d) 57. (c) 58. (c) 59. (c) 60. (d)61. (a) 62. (c) 63. (b) 64. (c) 65. (a) 66. (a) 67. (b) 68. (d) 69. (b) 70. (c)71. (a) 72. (b) 73. (d) 74. (d) 75. (b) 76. (b) 77. (a) 78. (c) 79. (b) 80. (b)81. (c) 82. (a) 83. (a) 84. (b) 85. (b) 86. (b) 87. (a) 88. (b) 89. (c) 90. (c)91. (c) 92. (c) 93. (d) 94. (c) 95. (c) 96. (c) 97. (c) 98. (c) 99. (c) 100. (b)

101. (c) 102. (b) 103. (b) 104. (c) 105. (d) 106. (d) 107. (d) 108. (c) 109. (a) 110. (b)111. (c) 112. (b) 113. (a) 114. (b) 115. (b) 116. (c) 117. (b) 118. (d) 119. (b) 120. (d)121. (d) 122. (b) 123. (a) 124. (c) 125. (c) 126. (b) 127. (a) 128. (a) 129. (c) 130. (b)131. (b) 132. (c) 133. (c) 134. (a) 135. (c) 136. (a) 137. (a) 138. (a) 139. (c) 140. (b)141. (b) 142. (a) 143. (a) 144. (c) 145. (a) 146. (c) 147. (d) 148. (d) 149. (c) 150. (d)151. (a) 152. (b) 153. (a) 154. (a) 155. (b) 156. (a) 157. (c) 158. (a) 159. (c) 160. (b)161. (d) 162. (c) 163. (c) 164. (a) 165. (d) 166. (d) 167. (d) 168. (a) 169. (d) 170. (c)171. (b) 172. (a) 173. (c) 174. (a) 175. (b) 176. (b) 177. (c) 178. (b) 179. (b) 180. (a)181. (d) 182. (c) 183. (a) 184. (b) 185. (b) 186. (a) 187. (c) 188. (d) 189. (c) 190. (a)191. (a) 192. (d) 193. (d) 194. (b) 195. (d) 196. (b) 197. (a) 198. (a) 199. (b) 200. (d)

Page 49: Download SSC CGL Tier I Last 5 Year Papers e Book Www.sscportal.in (1)

1.12: Solved Paper (19-05-2013) (1st Sitting)

Join Online Coaching for SSC Exams:http://sscportal.in/community/courses

Buy Printed Study Material for SSC Exams:http://sscportal.in/community/study-kit

EXPLANATIONS1. As for finishing the ‘Leather’,

‘Tanning’ method is used, in thesame way for ‘Pyrotechnics’,‘Fireworks’ is used.

2. As ‘King’ sits on the ‘Throne’, inthe same way ‘Rider’ sits on the‘Seat’.

3. As, F = 6⇒ (6)3 = 216Same as, L= 12⇒ (12)3 = 1728

4. 13 15 21 19 5M O U S E

11 16 19 20 3K P S T C

Same as,12 9 7 8 20L I G H T

10 10 5 9 18J J E I R

–2

+1–2

+1

–2

5. As,264

2= 132

⇒ 1 + 3 + 2 = 6

and870

3= 290

⇒ 2 + 9 + 0 = 11

Same as735

5= 147

⇒ 1 + 4 + 7 = 126. The meaningful order of the given

words are 3, 2, 5, 4, 6, 1.7. ba/ab/ab/ba/ab/bb8. abc/abb/cab/cca/bca/ab9. ‘Head’ is different from all other.

10. Except ‘Influenza’ all other diseaseare caused by lack of anyvitamins.

11. Here, all options are related to rainor water except Raisin. The

meaning of ‘Raisin’ is a partiallydried grapes which is differentfrom all others.

12. Except option (d) difference of allother numbers are divisible by 2.

13. 10, 13, 234, 681, 997⇒ 1 + 0, 1 + 3, 3 + 4 + 26 + 8 + 1, 9 + 9 + 7

1, 4, 9, 15, 25,

(1)2 (2)2 (3)2 (4)2 (5)2

16⇒

Hence, number 681 is wrong in theseries.

14. From option (b),A F I

+5 +3

J O R

+5 +3

M R U

+5 +3

H M P

+5 +315. bc, cde, de, efg, fg, ghi

In this series, last letter of a term isstarted to new term and next twoletter is included in the term.

16. C - 3, E - 5, G - 7, I - 9, K - 11,M - 13In this series every letter has givenits alphabetical number and oneletter is skip in between.

17. The sequence of the series is asfollows

5 × 3 + 1 = 1616 × 3 + 3 = 5151 × 3 + 5 = 158

158 × 3 + 7 = 481Hence, next term will be 481 in theseries

18. Given equation⇒ 9 × 2 : 9 × 9 : : 9 × 5 : ?⇒ 18 : 18 : : 45 : ?⇒ 18 : 81 : : 45 : 54

9 × 2 : 9 × 9 : : 9 × 5 : 9 6×Hence, 9 × 6 will come in place ofquestion mark.

19. Making equations by giveninformation.Priti > Rahul ...(i)Yamuna = Divya ...(ii)Lokita < Manju ...(iii)Rahul > Yamuna ...(iv)Manju < Divya ...(v)On adding all the equations, wegetPriti > Rahul > Yamuna = Divya >Manju > LokitaHence, it is clear that Lokitascored the lowest.

20. Son of sister-in-law of wife of theman means that boy is the son ofhis brother-in-law and the son ofthe man is cousin to that boy.

21. Given equation = 24 * 16 * 8 * 32Taking the sign of option (d),⇒ 24 + 16 – 8 = 32⇒ 40 – 8 = 32⇒ 32 = 32Hence, correct group of sign isoption (d).

22. Making relation tree from thegiven information.

Kishav Sister Simran Brother Prem

Sunil Maruti Sita

(+) (-) (+)

(+) (+) (-)

From above relation tree it is clearthat Sunil is maternal uncle ofMaruti.

23. Word ‘MASTER’ cannot beformed by using the letters ofgiven word because letter ‘E’ isnot present in the given word.

24. Word ‘NATION’ cannot be formedby using the letters of given wordbecause in given word, letter ‘N’ isuse only once whereas in word‘NATION’, letter ‘N’ is usedtwice.

25. Word ‘TALENT’ cannot be formedby using the letters of given word

Page 50: Download SSC CGL Tier I Last 5 Year Papers e Book Www.sscportal.in (1)

1.13: Solved Paper (19-05-2013) (1st Sitting)

Join Online Coaching for SSC Exams:http://sscportal.in/community/courses

Buy Printed Study Material for SSC Exams:http://sscportal.in/community/study-kit

because letter ‘E’ is not present inthe given word.

26. LADY = 2 (12 + 1 + 4 + 25)= 2 × 42 = 84

27. As, L E A D E

20 13 9 12 13

12 5 1 4 5R

26

18

+8 +8 +8 +8 +8 +8

Same as,

L I G H T

20 17 15 16 28

12 9 7 8 20

+8 +8 +8 +8 +8

28. Given word →COMMUNICATIONSNew word after interchanging theplace of lettersO C M M N U C I T A O I S N

10th from right

Hence, required letter is N.29. Given equation →

8 × 20 ÷ 3 + 9 – 5 = 38From option (c). interchanging thesign of 3 and 5.8 × 20 ÷ 5 + 9 – 3 = 38

⇒ 8 × 205

+ 9 – 3 = 38

⇒ 32 + 9 – 3 = 3⇒ 41 – 3 = 38⇒ 38 = 38

30. Given equation →33 ? 11 ? 3 ? 6 = 115From option (b), subsitute thesigns33 × 11 ÷ 3 – 6 = 115

⇒ 33 × 113

– 6 = 115

⇒ 11 × 11 – 6 = 115⇒ 121 – 6 = 115⇒ 115 = 115

31. Given equation →15 * 24 * 3 * 6 * 17From option (a), substitute thesigns15 + 24 ÷ 3 – 6 = 17

⇒ 15 + 243

– 6 = 17

⇒ 15 + 8 – 6 = 17⇒ 23 – 6 = 17⇒ 17 = 17

32. As,8412

× 2 = 14

and819

× 2 = 18

Same as,8811

× 2 = 16

Hence, number 16 will be come inplace of question mark.

33. As, (3 × 4 × 5) – 2 = 60 – 2 = 58and (5 × 6 × 2) – 2 = 60 – 2 = 58and (8 × 4 × 2) – 2 = 64 – 2 = 62Same as,

(7 × 6 × 3) – 2 = 126 – 2 = 124Hence, number 124 will be come inplace of question mark.

34. 4 × 9 = 36 = (6)2

27 × 3 = 81 = (9)2

x × 2 = 100 = (10)2

x = 5035. Way of Raj is as follows.

N

S

EW

S-W

It is clear from the above figurethat now Raj is in South-Westdirection.

36. Way of the boy is as follows.

N

S

EW

A G B3 km

2 km

3 km4 km

2 km H

C

EF

D

8 km

1

Let point A is the starting pointand G is the final point.Now, distance between startingpoint and final point(AG) = AB – BG = (8 – 3) km= 5 km

37. As per Conclusion I, healthypeople don’t medicine is right but

even, then people keep medicine intheir home is also right.Hence, both Conclusions I and IIfollow.

38.

Years Decades

Centuries

Hence the option (d) is true.41. Number of cubes in first row

= 4 + 3 + 2 + 1 = 10Number of cubes in second row= 3 + 2 + 1 = 6Number of cubes in third row= 2 + 1 = 3Number of cubes in fourth row= 1 = 1∴ Total cubes = 10 + 6 + 3 + 1= 20

102. Work done by A and B in 1 day

= 13

Their 2 days work = 2 × 13

= 23

∴ Remaining work = 1 – 23

= 13

So, B complete 13

work in 2 days.

∴ B alone can do it in 3 × 2 = 6days

103. Efficiency 1Time taken

So, if B is 100% efficient, then A is80% efficient.

So, 80

100 = 15

2

x here, x = Time

taken by B106. Diagonal of a square

= 2 × Side∴ Ratio of area of smaller circle to

larger circle = 2

12

2

rr

ππ

Page 51: Download SSC CGL Tier I Last 5 Year Papers e Book Www.sscportal.in (1)

1.14: Solved Paper (19-05-2013) (1st Sitting)

Join Online Coaching for SSC Exams:http://sscportal.in/community/courses

Buy Printed Study Material for SSC Exams:http://sscportal.in/community/study-kit

=

2

22

22

a

a

π

π

×

×

Here, a = Diameter of smaller circle

= 12

= 1 : 2

107. Total surface area of sphere = 8 π4 πr2 = 8⇒ r = 2 unit

∴ Volume of sphere = 43

πr3

= 43

× π × ( 2 )3 = 8 2

3π cu unit

108. Volume of water = Volume of

conical flask = 13

πr2h

Now, the water is poured incylindrical flask.So, volume of cylinder = Volume ofwater

= π (mr2) × Height = 13

πr2h

∴ Height = 23hm

109. Cost of a dozen pair of socks afterdiscount

= 180 80

100×

= Rs. 144

∴ Cost of 1 pair of socks = 14412

= Rs. 12So, with Rs. 48 total pairs of socks

can be bought = 4812

= 4 pairs

111. Marked price = Rs. 480

So cost after discount = 480 90

100×

= Rs. 432Let actual cost of the radio set

= Rs. x

Then, 108

100x×

= 432

∴ x = 432 100

108×

= Rs. 400

If discount is not allowed, then

gain is = 80400

× 100 = 20%

113. Let the share of P = Rs. xThen Q’s share = Rs. x + 30and R’s share = (x + 30) + 60= Rs. x + 90Sum of money with P, Q and R= Rs. 300∴ x + x + 30 + x + 90 = 300⇒ 3x + 120 = 300

∴ x = 300 –120

3 = Rs. 60

∴ Required ratio= 60 : (60 + 30) : (60 + 90) = 2 : 3 : 5

114. Required number= 50 × 9 – (54 × 5 + 3 × 52)= 450 – (270 + 156) = 450 – 426 = 24

116. Let the cost price of article = Rs. xThen, 120% of x = Rs. 300

⇒ 120

100x×

= 300

∴ x = 300 100

120×

= Rs. 250

Now, if SP = Rs. 235Then, loss percentage

= 250 – 235

250× 100 = 6%

117. After removing 25% of blue balls,total blue ball left

= 75% of 100 = 75 100

100×

= 75

After removing 50% of red balls,total red ball left

= 50% of 50 = 50 50

100×

= 25

∴ Required percentage

= 50

(75 25 50)+ + × 100

= 50

150× 100 =

133 %

3

119. Case I Speed = Distance

Time

Case II Speed = Distance

22 Time×

= Distance4×Time

∴ Required ratio = 14

= 1 : 4

120. Hence, the two simple interests areequal.Then,

4000 3100

x× × =

5000 12 2100× ×

∴ x = 10%121. x2 – 3x + 1 = 0

x2 + 1 = 3xDividing both side by x, we get

x2 + 21x

+ 2 = 9 ...(i)

1x

x+ = 3 ...(i)

x2 + 2

1x

= 7 ...(ii)

Saving both side by, we getSo, from Eqs. (i) and (ii), we get

x2 + x + 1x

+ 21x

= 7 + 3 = 10

122.4 – 3x

x +

4 – 3yy +

4 – 3zz

= 0

⇒ 4xx

–3x

+4yy –

3y +

4zz

–3z

= 0

⇒ 4 + 4 + 4 – 31 1 1x y z

+ + = 0

⇒ 12 = 1 1 1

3x y z

+ +

Page 52: Download SSC CGL Tier I Last 5 Year Papers e Book Www.sscportal.in (1)

1.15: Solved Paper (19-05-2013) (1st Sitting)

Join Online Coaching for SSC Exams:http://sscportal.in/community/courses

Buy Printed Study Material for SSC Exams:http://sscportal.in/community/study-kit

So, 1x

+ 1y +

1z

= 4

123. 2 50 + 18 – 72= 2 5 5 2× × + 3 3 2× × –

6 6 2× ×

= 2 × 5 2 + 3 2 – 6 2= 10 2 + 3 2 – 6 2= 13 2 – 6 2 = 7 2= 7 × 1.414 = 9.898

124. a2 + b2 + 4c2 = 2 (a + b – 2c) – 3a2 + b2 + 4c2 = 2a + 2b – 4c – 3(a2 – 2a + 1) + (b2 – 2b + 1) + (4c2

+ 4c + 1) – 1 – 1 – 1 = – 3(a – 1)2 + (b – 1)2 + (2c + 1)2 = 0Thus,(a – 1) = 0 ⇒ a = 1(b – 1) = 0 ⇒ b = 1(2c – 1 = 0)

∴ c = 1

–2

∴ a2 + b2 + c2 = 12 + 12 + 1

–2

= 1 + 1 + 14

= 1

24

125. By shortcut method, remainder

= 1817

= 1

127. If denominator of fraction = x,Then numerator = x – 4

∴ Fraction = – 4xx

Now, according to the question,

(x – 4) – 2 = ( 1)

8x +

⇒ x – 6 = 1

8x +

⇒ 8x – 48 = x + 1⇒ 8x – x = 48 + 1∴ x = 7

∴ Fraction = 7 – 4

7 =

37

128. x2 = y + z, y2

= z + xand z2 = x + ySatisfies only whenx = y= z = 2

∴ 1

1 x+ +

11 y+ +

11 z+

= 13

+ 13

+ 13

= 1

129. B

D

CA 90° 55°

∠B = 180° – (90° + 55°) = 35°Now, in ∆ADC,∠ADC + ∠ACD + ∠DAC = 180°∠DAC = 180° – 90° – 55° = 35°∴ ∠BAD = 90° – 35° = 55°

130.

G

B C

A

Area of ∆BGC = Area of

3ABC∆

= 483

= 16 cm2

131. A B

D C

P

So, AP . BP = CP . DP

132.

A B

P

ON

7 cm 7 cm

12 cm

On joining AP,Now, ∠APB = 90°

(QAngle in a semicirlce = 90°)AB2 = AP2 + PB2

(14)2 = AP2 + (12)2

AP2 = 52 cmNow, let AN = x cmThen, NO = (7 – x) cmIn ∆APN, AP2 = x2 + PN2

PN2 = AP2 – x2

PN2 = 52 – x2 ...(i)In ∆PNO, PO2 = PN2 + NO2

(7)2 = PN2 + (7 – x)2

49 – (7 – x)2 = PN2 ..(ii)From Eqs. (i) and (ii), we get⇒ 52 – x2 = 49 – (49 – 14x + x2)⇒ 52 – x2 = 49 – 49 + 14x – x2

⇒ 14x = 52

∴ x = 5214

= 267

cm

∴ Length of NO = 7 – 267

= 49 – 26

7 =

237

cm

So, length of NB = 7 + 237

= 2107

cm

133.

Q R

P

O

25°

110°

∠OPR = ∠ORP = 25°(angles opposite of radius)

Now, in ∆QOR,∠QOR + ∠OQR + ∠ORQ = 180°

Page 53: Download SSC CGL Tier I Last 5 Year Papers e Book Www.sscportal.in (1)

1.16: Solved Paper (19-05-2013) (1st Sitting)

Join Online Coaching for SSC Exams:http://sscportal.in/community/courses

Buy Printed Study Material for SSC Exams:http://sscportal.in/community/study-kit

⇒ ∠QOR + ∠ORQ + ∠ORQ= 180°(Q angles opposite to equal sidein a triangle are equal)⇒ 2∠ORQ = 180° – 110°

∠ORQ =702

° = 35°

∴ ∠PRQ = 25° + 35° = 60°134. Let the height of tower = x cm

Then,128

=40x

⇒12 40

= x

∴ x= 60 m135. ∠CED = 180° – 130°

= 50°Now, in ∆CED,∠ECD + ∠CED + ∠CDE = 180°∠CDE = 180° – 50° – 20° = 110°∴ ∠BAC = ∠CDE = 110°

(angles in same segment are equal)

137.

A

B C

D

36 m

12 m

F

24 m

60°

60°

So, in ∆ADE,

sin 60° =DEAD

(AD = length of wire)

⇒ AD =sin 60

DE°

= 12

3

=12 2

= 8 3 m

138.1

cosec – cotθ θ –

1sinθ

= 1

1 cos–sin sin

θθ θ

– 1

sinθ

= sin

1– cosθ

θ –

1sinθ

= 2sin – (1– cos )

(1– cos )sinθ θ

θ θ

= 21– cos –1 cos

(1– cos )sinθ θ

θ θ+

= cos (1– cos )(1– cos )sin

θ θθ θ = cot θ

139. cos θ + sin θ = 2 cos θOn squaring both sides, we get(cos θ + sin θ)2 = ( 2 cos θ)2

cos2 θ + sin2 θ + 2sin θ cos θ= 2cos2 θ

2sin θ cos θ = cos2 θ – sin2 θ2 sin θ cos θ = (cos θ – sin θ)

(cos θ + sin θ)

cos θ – sin θ = 2sin cos

(cos sin )θ θ

θ θ+

= 2sin cos

2 cosθ θ

θ = 2 sinθ

140. tan α = n tan β

sincos

αα

= n sincos

ββ

sincos

m βα

= n sincos

ββ

cos α =mn

cos β

On squaring both sides, we getsin2 α = m2 sin2 β

1 – cos2 α = m2 (1 – cos2 β)1 – cos2 α = m2 – m2 cos2 β

– 2 2

2

(1 – cos – )mm

α = cos2 β

2 2

2

(cos –1)mm

α + = cos2 β ..(ii)

From Eqs. (i) and (ii), we get

cos2 α = 2

2

mn

×2 2

2

(cos –1)mm

α +

n2 cos2 α = cos2 α + m2 – 1

(n2 – 1) cos2 α = m2 – 1

⇒ cos2 α = 2

2

–1–1

mn

141. cos4 θ – sin4 θ = 23

(cos2 θ)2 – (sin2 θ)2 = 23

(cos2 θ – sin2 θ) (cos2 θ + sin2 θ)

= 23

1– sin2 θ − sin2θ = 23

1 – 2 sin2 θ = 23

142.

3

tan θ =34

= PB

H2 = P2 + B2

H2 = 32 + 42

H = 9 16+

= 25 = 5

∴ cosec θ =HP

= 53

Hence,P = PerpendicularH = HypotaneosB = Traea

143. 2

11 tan θ+

+ 2

11 cot θ+

;

2

2

1sin1cos

θθ

+ + 2

2

1cos1sin

θθ

+

= 2

2 2

coscos sin

θθ θ+

+ 2

2 2

sin

sin cos

θ

θ θ+

= cos2 θ + sin2 θ = 1

Page 54: Download SSC CGL Tier I Last 5 Year Papers e Book Www.sscportal.in (1)

1.17: Solved Paper (19-05-2013) (1st Sitting)

Join Online Coaching for SSC Exams:http://sscportal.in/community/courses

Buy Printed Study Material for SSC Exams:http://sscportal.in/community/study-kit

144. Here, x= 20and y = 22,Then

∴ Percent gain = –y xx

× 100%

= 22 – 20

20× 100% = 10%

145. Let CP of the article be Rs. x.Then, SP = 110% of x = 1.1xIf SP be double i.e., 2.2x, then

Profit percent = 2.2 –x x

x× 100%

= 120%146. Let the number be x.

Then, 137.5% of x = 33

∴ x = 33 100

1375×

= 24

147. Marks in English, Physics andSocial Science

= 60 85 55

360+ +

× 810

= 200360

× 810 = 450

Marks in Maths and Chemistry= 810 – 450 = 360∴ Required percentage

= 450 – 360

810× 100 =

90810

× 100

= 111 %9

148. Difference of marks in Physics andChemistry

= 85 – 70

360× 810

= 15360

× 810 = 33.75

They are equal to Chemistry andSocial Science

= 70 – 55

360× 810 = 33.75

149. Required number

= (90 70) – (85 55)

360+ +

× 810

= (160 –140) 810

360×

= 20 810

360×

= 45

Page 55: Download SSC CGL Tier I Last 5 Year Papers e Book Www.sscportal.in (1)

SOLVED PAPERSSC-CGL

PRE. EXAM 2014Held On: 26-10-2014 (Morning Shift)

Directions (Q. Nos. 1-6) In the followingquestions, select the related word/letters/number from the givenalternatives.

1. Optimist : Cheerful : : Pessimist :?(a) Petty (b) Selfish(c) Gloomy (d) Mean

2. YTOJ : XSNI : : WRMH : ?(a) VQLG (b) TOJE(c) RMHC (d) UPKF

3. EIGK : EACY : : RVTY : ?(a) RVSQ (b) RNPL(c) RWUY (d) RMPL

4. 720 : 840 : : 60 : ?(a) 76 (b) 80(c) 70 (d) 74

5. 225 : 256 : : 289 : ?(a) 234 (b) 361(c) 324 (d) 432

6. Money : Misappropriation : :Writing : ?(a) Deception (b) Mistake(c) Plagiarism (d) Theft

Directions (Q. Nos. 7-11) Select the onewhich is different from other threealternatives.

7. (a) Rival(b) Opponent(c) Foe (d) Ally

8. (a) 27 (b) 35(c) 18 (d) 9

9. (a) 9-72 (b) 8-56(c) 11-115 (d) 10-90

10. (a) TUVX (b) OPRS(c) BCDF (d) HIJL

11. (a) Bay (b) Cape(c) Peninsula (d) Island

12. Which one of the given responsewould be a meaningful order ofthe following?1. Seed 2. Plant3. Germination 4. Embryo5. Flower 6. Fruit(a) 1, 5, 3, 4, 2, 6(b) 1, 4, 3, 2, 5, 6(c) 6, 5, 4, 3, 1, 2(d) 1, 5, 6, 4, 2, 3

13. Arrange the following words asper order in the Englishdictionary.1. Live 2. Litter3. Little 4. Literacy5. Living(a) 3, 4, 2, 1, 5(b) 3, 2, 4, 5, 1(c) 4, 3, 5, 2, 1 (d) 4, 2, 3, 1, 5

14. Which one set of letters whensequentially placed at the gaps inthe given letter series shallcomplete it?ipi_upog_pig_pogi_g(a) iupgg (b) upgii(c) puigp (d) giupi

15. In the following series, how manyKGN occur in such a way that ‘G’is in the middle and ‘K’ and ‘N’are adjacent to it on both sides?A K G L M N D Q K G C S N G KT G K G N D Z P U X G K E(a) 3 (b) 5(c) 1 (d) 2

Directions (Q. Nos. 16-18) In theFollowing questions, a series is given,with one term missing. Choose thecorrect alternative from the given onesthat will complete the series.

16. 1, 0, 3, 2, 5, 6, ?, 12, 9, 20(a) 9 (b) 10(c) 7 (d) 8

17. SHG, RIF, QJE, PKD, ?(a) NME (b) NLB(c) OLE (d) OLC

18. 7, 8, 18, 57, ?, 1165(a) 174 (b) 232(c) 224 (d) 228

19. There are five friends – Sachin,Kamal, Mohan, Arun and Ram.Sachin is shorter than Kamal, buttaller than Ram. Mohan is thetallest. Arun is a little shorter thanKamal and a little taller thanSachin. Who is the second tallest?(a) Ram (b) Sachin(c) Kamal (d) Arun

20. A is the brother of B, B is thedaughter of C and D is the fatherof A. Then, how is C related to D?(a) Husband (b) Wife(c) Granddaughter(d) Grandfather

21. From the given alternative words,select the word which can beformed using the letters of thegiven word.DEMOCRACY(a) SECRECY (b) MICRO(c) MARCY (d) DEMON

REASONING

Page 56: Download SSC CGL Tier I Last 5 Year Papers e Book Www.sscportal.in (1)

1.2: Solved Paper (26-10-2014) (1st Sitting)

Join Online Coaching for SSC Exams:http://sscportal.in/community/courses

Buy Printed Study Material for SSC Exams:http://sscportal.in/community/study-kit

22. From the given alternative words,select the word which cannot beformed using the letters of thegiven wordINCARCERATION(a) RELATION(b) TERRAIN(c) INACTION(d) CREATION

23. Mr. Mani’s age is 47 yr andJohn’s age is 13 yr. In how manyyears will Mr. Mani’s age bedouble of John’s age?(a) 20 (b) 21 (c) 10 (d) 15

24. If in a certain code‘INTELLIGENCE’ is written as‘ETNIGILLECNE’, then how can‘MATHEMATICAL’ be written inthe same code?(a) AMHTMETACILA(b) TAMMEHITALAC(c) HTAMTAMELACI(d) LACITAMEHTAM

25. If in a certain code‘CHARACTER’ is written as‘241612376’ and ‘CHILDREN’ iswritten as ‘24859670’, then howcan ‘HIRALAL’ be written in thesame code?(a) 4861551(b) 4861515(c) 4685151(d) 4685511

26. Given below are capital letters inthe first line and symbols in thesecond line. Symbols and lettersare codes for each other. Choosethe correct code for the givenletters.

A C E G H I O N P R T S B D M

+ – ÷ × = ( ) [ ] ≠ || # | > <BEAST(a) | ÷ + ≠ | | (b) | ÷ × # | |(c) | ÷ + # | | (d) | ÷ + ≠ =

27. In an examination, 78% of thetotal students who appeared weresuccessful. If the total number offailures was 176 and 34% got firstclass, then how many students gotfirst class?

(a) 272 (b) 112 (c) 210 (d) 25428. Some letters are given with

numbers from 1 to 6. Select thesequence of numbers whicharranges the letters into ameaningful word1 2 3 4 5 6G I C O D N(a) 2, 1, 4, 3, 6, 5(b) 4, 3, 2, 6, 5, 1(c) 6, 5, 2, 3, 1, 4(d) 3, 4, 5, 2, 6, 1

29. Select the correct combination ofmathematical signs to replace *signs and to balance thefollowing equation(8 * 7 * 6) * 5 * 10(a) × – ÷ =(b) – × ÷ +(c) + – ÷ ×(d) × + = ÷

30. If ‘>’ stands for ‘division’, ‘∨’stands for ‘multiplication’, ‘–’stands for ‘greater than’, ‘×’standsd for ‘less than’, ‘<’ standsfor ‘addition’, ‘∧’ stands for‘subtraction’, ‘+’ stands for ‘equalto’, then which of the followingequation is correct?(a) 10 ∧ 3 < 5 < 4 + 16(b) 4 ∧ 6 < 4 < 4 + 16(c) 12 ∧ 3 < 5 < 4 + 12(d) 11 ∧ 12 < 5 < 4 + 12

31. Some equations are solved on thebasis of a certain system. Find thecorrect answer for the unsolvedequation on that basis.5 * 3 = 125; 4 * 3 = 64; 8 * 2 = ?(a) 28 (b) 16(c) 32 (d) 64

Directions (Q. Nos. 32-34) In thesequestions, select the missing numberfrom the given responses.

32. 9 7 45 4 36 8 ?49 52 25

(a) 6 (b) 7(c) 4 (d) 5

33. 4 10 228 5 14

10 3 ?

(a) 13 (b) 14(c) 11 (d) 12

34. 2 3 43 4 5

13 ? 41

(a) 31 (b) 46(c) 23 (d) 25

35. Some statements are givenfollowed by two Conclusions Iand II. You have to consider thestatements to be true, even if theyseem to be at variance fromcommonly known facts. You areto decide which of the givenconclusions can definitely bedrawn from the given statements.Indicate your answer.StatementsAll files are papers.Some papers are books.Some books are journals.ConclusionsI. All files are journals.II. All books are not journals.(a) Both Conclusions I and II

follow(b) Neither Conclusion I nor II

follows(c) Only Conclusion I follows(d) Only Conclusion II follows

36. Pratap starts from school andwalks 7 km towards East. He takesa left and walks 4 km, then takesa right and walks 2 km, againtakes a right and walks 3 km.Which direction is he facing now?(a) South (b) North(c) East (d) West

37. Vijay travelled 12 km Southward,then turned right and travelled 10km, then turned right and

Page 57: Download SSC CGL Tier I Last 5 Year Papers e Book Www.sscportal.in (1)

1.3: Solved Paper (26-10-2014) (1st Sitting)

Join Online Coaching for SSC Exams:http://sscportal.in/community/courses

Buy Printed Study Material for SSC Exams:http://sscportal.in/community/study-kit

travelled 12 km. How far wasVijay from the starting point?(a) 22 km (b) 44 km(c) 12 km (d) 10 km

38. Fertilizer consumption in India in1984-85 was 8.21 MT. By 1990 itwas 13.75 MT and by 2000 it wasexpected to reach 16 MT. Whatconclusion can you draw fromthis data?(a) Fertilizer consumption does

not show any trend(b) There is a steady increase of

the fertilizer consumption(c) Fertilizer consumption is

steady(d) There is a steady decrease in

the fertilizer consumption.39. How many triangles are there in

the given figure?

(a) 18(b) 20 or more(c) 12(d) 16

40. Among the four answer figures,which figure can be formed thecut-pieces given below in thequestion figure?Question Figure

Answer Figures

41. Choose the cube which will beformed on folding the givenquestion figure.

42. In the given figure, which letterrepresents those Actors who arealso Dancers, Singers as well asViolinists?

(a) S (b) Q(c) P (d) U

Directions (Q. Nos. 43 and 44) In thefollowing questions, identify thediagram that best represents therelationship among the classes givenbelow.

43. Factory, Machinery, Product

44. Travellers, Train, Bus

Directions (Q. Nos. 45 and 46) In thesequestions, which answer figure willcomplete the pattern in the questionfigure?

45.

46.

47. A piece of paper is folded and cutas shown below in the questionfigures. From the given answerfigures, indicate how it willappear when opened.

48. From the given answer figures,select the one is which thequestion figure is hidden/embedded.

49. A word is represented by only oneset of numbers as given in anyone of the alternatives. The sets ofnumbers given in the alternativesare represented by two classes ofalphabets as in two matrices givenbelow. The columns and rows of

Page 58: Download SSC CGL Tier I Last 5 Year Papers e Book Www.sscportal.in (1)

1.4: Solved Paper (26-10-2014) (1st Sitting)

Join Online Coaching for SSC Exams:http://sscportal.in/community/courses

Buy Printed Study Material for SSC Exams:http://sscportal.in/community/study-kit

Matrix I are numbered from 0 to 4and that of Matrix II are numberedfrom 5 to 9. A letter from thesematrices can be represented firstby its row and next by its column.e.g., ‘E’ can be represented by 02,43, 68 etc., and ‘N’ can berepresented by 21, 65, 95 etc.Similarly, you have to identify thenumber set for the word givenbelow.Cost

(a) 04, 30, 31, 41(b) 88, 57, 69, 32(c) 04, 11, 41, 76(d) 88, 78, 99, 89

50. If a mirror is placed on the line MN,then which of the answer figures is

the image of the question figure?

GENERAL AWARENESS51. he famous economist Thomas

Robert Malthus, in his viewsregarding population, is(a) optimistic(b) partly optimistic and partly

pessimistic(c) pessimistic(d) None of the above

52. Non-insurable or uncertainty riskis(a) change in fashion(b) fire(c) flood(d) change in the price of that

commodity53. ‘Gold Bullion Standard’ refers to

(a) gold coin as an unlimited legaltender

(b) gold as the measure of value(c) free coinage of gold(d) no restriction on import and

export of gold54. Laws made by the executive are

called(a) Delegated Legislation(b) Preferred Legislation(c) Rule of Law(d) Administrative Law

55. The word ‘Oikonomia’ means(a) Household Management(b) Individual Management(c) Political Management(d) Fiscal Management

56. The word ‘Economics’ is takenfrom which language?(a) French (b) Greek(c) German (d) English

57. Which of the following is a meritof the presidential form ofgovernment?(a) The fixed tenure of the

executive gives it a greatsense of stability

(b) It ensures speedy execution ofpolicies

(c) It safeguards the liberty of thepeople

(d) All of the above58. The two chambers of the Swiss

Federal Assembly are called(a) National Council and Council

of States(b) House of Lords and House of

Commons(c) Senate and House of

Representatives(d) None of the above

59. Name the Italian traveller whovisited the Vijayanagar empire in1420 AD(a) Domingos Paes(b) Edoardo Barbosa(c) Nicolo de Conti(d) Abdur Razzag

60. Which one of the followingdynasties established anindependent rule in Bijapur?

(a) Imad Shahi (b) Nizam Shahi(c) Qutb Shahi (d) Adil Shahi

61. Who regarded the secondchamber as needless, useless andworse?(a) Laski (b) Maclver(c) Bentham (d) Seeley

62. The tenure of the Swiss Presidentis(a) 3 yr (b) 2 yr(c) 4 yr (d) 1 yr

63. Haemolymph is observed in(a) Arthropods(b) Echinoderms(c) Ascaris (d) Annelids

64. The virus without capsid but onlywith nucleic acids is called(a) Capsomere (b) Nucleoid(c) Prion (d) Virion

65. The immunity caused by the B-lymphocytes is called(a) Sterile immunity(b) Acquired Immunity(c) Cellular Immunity(d) Humoral Immunity

66. Who among the followingenjoyed the patronage of Sultansfrom Balban to Ghiyas-ud-dinTughlaq?(a) Ibn Batutah(b) Badauni(c) Zia-ud-din Barani(d) Amir Khusrow

Page 59: Download SSC CGL Tier I Last 5 Year Papers e Book Www.sscportal.in (1)

1.5: Solved Paper (26-10-2014) (1st Sitting)

Join Online Coaching for SSC Exams:http://sscportal.in/community/courses

Buy Printed Study Material for SSC Exams:http://sscportal.in/community/study-kit

67. The Sufi movement originallystarted from(a) Persia (b) Delhi(c) Lahore (d) Kabul

68. Humayun was driven out ofHindustan in 1540 AD after thebattle of(a) Chausa (b) Dourah(c) Surajgarh (d) Kannauj

69. Poly metallic nodules (also calledmanganese nodules) are found inconcentrations(a) on continental shelves(b) in deep ocean beds(c) on lake beds(d) in beach sands

70. Spot the odd item in the following(a) Meander (b) Delta(c) Bolson (d) Ox-bow lake

71. Kandla port is located at(a) Gulf of Khambat(b) Kori Creek(c) Gulf of Kutch(d) None of these

72. Which of the following forestsgrows in waterlogged areas?(a) Mangrove (b) Evergreen(c) Deciduous (d) Coniferous

73. Nag Tiba and Mahabharat rangesare included in(a) Sub-Himalayas(b) Trans-Himalayas(c) Greater Himalayas(d) Lesser Himalayas

74. Pink mould is the common name for(a) Aspergillus(b) Rhizopus(c) Neurospora(d) Mucor

75. The sleeping sickness is causedby(a) Escherichia(b) Neisseria(c) Entamoeba(d) Trypanosoma

76. Combustion is a(a) physicai and chemical process(b) biological process(c) physical process(d) chemical process

77. Which of the following is not aform of carbon?(a) Soot (b) Hematite(c) Graphite (d) Charcoal

78. What is Dry Ice?(a) Solid carbon dioxide(b) Liquid nitrogen(c) ice cubes and saw dust(d) Ice cubes and salt

79. The gas commonly used inadvertisement sign-boards anddecorative lights is(a) nitrogen (b) chlorine(c) hydrogen (d) neon

80. Flow of blood in dorsal bloodvessel of earthworm is(a) sideward (b) downward(c) backward (d) forward

81. Which of the following isbiodegradable?(a) Leather belts(b) Silver foil(c) Iron nails (d) Plastic mugs

82. Fungi that grow on bark are saidto be(a) Xyiophilous(b) Saxicolous(c) Coprophilous(d) Corticolous

83. To an astronaut in space the skywill appear to be(a) violet (b) red(c) blue (d) black

84. On a rainy day, small oily films onwater show brilliant colours. Thisis due to(a) scattering (b) dispersion(c) refraction (d) polarisation

85. Natural radioactivity wasdiscovered by(a) Rutherford (b) Marie Curie(c) Henri Becquerel(d) Enrico Fermi

86. Rainbow formation is due to(a) absorption of sunlight by

water droplets(b) diffusion of sunlight through

water droplets(c) ionisation of water droplets(d) refraction and reflection of

sunlight by water droplets

87. CAD stands for(a) Computer Automatic Design(b) Computer Aided Decode(c) Computer Automatic Decode(d) Computer Aided Design

88. Unix operating system is generallyknown as(a) Multi User Operating System(b) General Application(c) Single User Operating System(d) Single User Application

Program89. Who holds the world record as

the youngest person to climbMount Everest?(a) Santosh Yadav(b) Phu Dorjee(c) Bachendri Pal(d) Dicky Dolma

90. Hyperplasia means(a) abnormal increase in number

of cells(b) an increase in size of a cell(c) excessive motility of muscle(d) voracious eating

91. The part of root involved in waterabsorption is(a) zone of elongation(b) zone of root cap(c) zone of cell division(d) zone of root hairs

92. Who is the Prime Minister ofAustralia?(a) Tony Abbott(b) Kevin Rudd(c) Julia Gillard(d) None of these

93. Which of the parent country ofmobile giant ‘Nokia’?(a) Finland (b) Sweden(c) Spain (d) Germany

94. Where was the Joint G-20 Financeand Labour Ministers Meetingheld on July 19, 2013?(a) New York (b) Paris(c) Moscow (d) London

95. Which is the largest man-madelake?(a) Baikal (b) Wullar(c) Govind Sagar(d) Rana Pratap Sagar

Page 60: Download SSC CGL Tier I Last 5 Year Papers e Book Www.sscportal.in (1)

1.6: Solved Paper (26-10-2014) (1st Sitting)

Join Online Coaching for SSC Exams:http://sscportal.in/community/courses

Buy Printed Study Material for SSC Exams:http://sscportal.in/community/study-kit

96. Who took oath as the President ofIran on August 4, 2013?(a) Hassan Rouhani (b) Mohammad Bashir Ahmad(c) Mohammad Bagher Ghalibaf(d) Mahmoud Ahmadinejad

97. Who was sworn-in as the UnitedStates Ambassador to the UnitedNations by Joe Biden on August 1,2013?(a) Rosemary DiCarlo(b) Samantha Power

(c) Zalmay Khalizad(d) Susan Rice

98. Who won the Rogers Cupwomen’s singles in lawn tennis in2013?(a) Victoria Azarenka(b) Agnieszka Radwanska(c) Maria Sharapova(d) Serena Williams

99. Who was appointed as theChairperson of the NationalSchool of Drama on August 19,2013?

(a) Adly Mansour(b) Sasikumar Venkat(c) Amal Allana(d) Ratan Thiyam

100. To which channel did theInternational Olympic Committeesell the broadcasting rights for2014 and 2016 Olympics?(a) Set Max(b) Zee sports(c) STAR India(d) Doordarshanz

QUANTITATIVE APTITUDE101. Subhash can copy 50 pages in 10

h; Subhash and Prakash can copy300 pages in 40 h. In how muchtime can Prakash copy 30 pages?(a) 12 h (b) 9 h(c) 13 h (d) 10 h

102. Rajesh is in-charge of buyingbread rolls and buns for a party.There are 10 buns in each box ofbuns and 8 bread rolls in each boxof bread rolls. Rajesh wants to buyexactly the same number of bunsand bread rolls. What is thesmallest number of boxes heshould buy for buns alone?(a) 10 (b) 8 (c) 4 (d) 5

103. Simplify

24

2

–11

–2x

x

+

(a)4

2

12

xx+

(b)2 1

2x

x2

+

(c)4 2

2

2 –12

x xx

+

(d)4

2

–12

xx

104. If a job is to be completed in 10days, it requires 270 persons. If 180persons take up the same job, theywill finish it in

(a) 27 days (b) 12 days(c) 15 days (d) 18 days

105. A certain number of men canfinish a piece of work in 100 days.However, if there were 10 men less,then it would take 10 days more forthe work to be finished How manymen were there originally?(a) 110 (b) 75 (c) 50 (d) 100

106. A square lawn has path 4 m widearound it. If the area of the path is196 m2, then each side of the lawnis(a) 7.25 m (b) 9.25 m(c) 8.25 m (d) 8 m

107. A cloth merchant has announced25% rebate on marked price. If oneneeds to have a rebate of Rs. 40,then how many shirts, each with amarked price of Rs. 32, should hepurchase?(a) 7 (b) 8 (c) 5 (d) 6

108. One dozen pairs of socks quotedat Rs. 160 are available at adiscount of 10%. Find how manypairs of socks can be bought forRs. 60.(a) 6 pairs (b) 8 pairs(c) 5 pairs (d) 7 pairs

109. A machine is marked at Rs. 7500for sale. The shopkeeper allowssuccessive discounts of 8%, 5%and 2% on it. The net selling price(in Rs.) is

(a) 6427.50 (b) 6415.40(c) 6400.30 (d) 6423.90

110. The surface area of two spheresare in the ratio 4 : 9. What is theratio of their volumes?(a) 8 : 27 (b) 16 : 81(c) 2 : 3 (d) 4 : 9

111. On increasing the diameter of acircle by 75%, the percentageincrease in the perimeter is(a) 75% (b) 80%(c) 65% (d) 70%

112. If the perimeter of an equilateraltriangle is 42 cm, its area, (in sqcm) will be(a) 24 3 (b) 49 3(c) 7 3 (d) 14 3

113. If x + 1x

= 1, (x ≠ 0), then the value

of 2

2

3 17 1

x xx x

+ ++ +

is

(a) 1 (b) 2 (c) 0 (d)12

114. If a + b + c = 0, find the value of

3

1a

+ 3

1b

+ 3

1c

+ 3

1( )a b+

+ 3

1( )b c+ + 3

1( )b c+

Page 61: Download SSC CGL Tier I Last 5 Year Papers e Book Www.sscportal.in (1)

1.7: Solved Paper (26-10-2014) (1st Sitting)

Join Online Coaching for SSC Exams:http://sscportal.in/community/courses

Buy Printed Study Material for SSC Exams:http://sscportal.in/community/study-kit

(a) –1 (b) 3abc(c) 1 (d) 0

115. If x + y = 4 and 1x

+ 1y = 4, then

the value of x3 + y3 is(a) 52 (b) 64 (c) 4 (d) 25

116. (a2 + 2a)2 + 12(a2 + 2a) – 45 can beexpressed as(a) (a – 1) (a –3) (a2 + 2a + 15)(b) (a – 1) (a + 3) (a2 + 2a + 15)(c) (a + 1) (a + 3) (a2 + 2a + 15)(d) (a + 1) (a – 3) (a2 + 2a + 15)

117. The ratio of the first and secondclass fares between two stationsis 4 : 1 and that of the number ofpassengers travelling by the firstand second class is 1 : 40. If Rs.11000 is collected as total fare,then the amount collected from thefirst class passengers is(a) Rs. 1375 (b) Rs. 3150(c) Rs. 800 (d) Rs. 1000

118. The average age of three boys is15 yr. If their ages are in the ratioof 3 : 5 : 7, then the age of theyoungest boy (in yr) is(a) 18 (b) 21 (c) 9 (d) 15

119. Of three numbers, the first is twicethe second and the second isthrice the third. If the average ofthe three numbers is 10, then thelargest number is(a) 12 (b) 18 (c) 30 (d) 15

120. A man purchased a car for Rs.135000 and spent Rs. 25000 onrepairs. At what price was the carsold if he suffered 10% loss on it?(a) Rs. 176000 (b) Rs. 144000(c) Rs. 121500 (d) Rs. 150000

121. In the certain examination, 77%candidates passed in English and34% failed in Mathematics. If 13%failed in both the subjects and 784candidates passed in both thesubjects, then the total number ofcandidates was(a) 1600 (b) 1800(c) 1200 (d) 1400

122. A train 120 m long runs through astation at the speed of 54 km/h.

The time taken by the train to passa man standing on the platform(a) 7s (b) 8s(c) 5s (d) 6s

123. A, B and C start together from thesame point to travel around acircular island 30 km incircumference. A and B aretravelling in the same direction andC is in the opposite direction. If Atravels 5 km, B travels 7 km and Ctravels 8 km in an hour, then theyall come together again after(a) 25 h (b) 30 h(c) 15 h (d) 20 h

124. At what rate will a sum of Rs. 1000amount to Rs. 1102.50 in 2 yr atcompound interest?(a) 6% (b) 6.5%(c) 5% (d) 5.5%

125. Radii of two circles are 6.3 cm and3.6 cm. If they touch each otherinternally, then the distancebetween their centres is(a) 9.1 cm (b) 2.7 cm(c) 3.7 cm (d) 10.1 cm

126. There is a circle of radius 5 cm andthe perpendicular distance fromthe centre of the circle to the chordof the circle is 3 cm. Then, thelength of the chord is(a) 6 cm (b) 5 cm(c) 8 cm (d) 4 cm

127. In a ∆ABC, ∠A – ∠20°, ∠B –∠C= 32°. Then, the ∠A is(a) 84° (b) 90°(c) 75° (d) 80°

128. The bisector of ∠BAC of ∆ABCcuts BC at and D and thecircumcircle of the triangle at E. IfDE = 3 cm, AC = 4 cm and AD = 5cm, then the length of AB is(a) 9 cm (b) 10 cm(c) 7 cm (d) 8 cm

129. In two ∆ABC and ∆PQR,

BC = QR , AB = PQ andextended m ∠ABC = 120° andextended m ∠PQS = 120°. Then, bywhich condition are the twotriangles congruent?

(a) Side – Angle – Side(b) Angle – Side – Side(c) Angle – Side – Angle(d) Side – Side – Side

130. If an equilateral triangle ABC isinscribed in a circle with centre at0, then ∠BOC, ∠COA and ∠AOBare respectively(a) 50°, 60°, 70°(b) 120°, 120°, 120°(c) 60°, 60°, 60°(d) 80°, 120°, 160°

131. If x = 3 + 2 2 , then the values of

x3 + 3

1x

and x3 – 3

1x

are

respectively

(a) 140 2, 198(b) 234, 216

(c) 216, 234 (d) 198,140 2132. Given that a + b +c = 2 and ab + bc

+ ca = 1, then the value of (a + b)2

+ (b + c)2 + (c + a)2 is(a) 10 (b) 16(c) 6 (d) 8

133. If x2 + 4x + 3 = 0, then the value of3

6 327 27x

x x+ + is

(a) –1 (b)1

–2

(c) 1 (d)12

134. Find the area of the tr iangle

formed by the graph F = 95C

+ 32

with the F-axis and C-axis.

(a)2760

9 sq unit

(b)2760

9 sq unit

(c)2860

9 sq unit

(d)2560

9 sq unit

Page 62: Download SSC CGL Tier I Last 5 Year Papers e Book Www.sscportal.in (1)

1.8: Solved Paper (26-10-2014) (1st Sitting)

Join Online Coaching for SSC Exams:http://sscportal.in/community/courses

Buy Printed Study Material for SSC Exams:http://sscportal.in/community/study-kit

135. If in a ∆ABC, AB is smaller thanAC, then AC – AB is

(a) = BC (b) < 12

AC

(c) < BC (d) > BC

136. 1 sin1– sin

θθ

+ is equal to

(a) sec θ + tan θ(b) sec θ – tan θ(c) cosec θ + cot θ(d) cosec θ – cot θ

137. If tan θ = 34

and 0 < θ < 2π

and

25x sin2θ cos θ = tan2θ, then thevalue of x is

(a)764

(b)964

(c)3

64

(d)5

64138. A kite flying at a height of 45 m

from the level ground is attachedto string inclined t 60° to thehorizontal. The length of thestring is

(a)30

2 m

(b) 30 5 m

(c) 30 3 m

(d) 30 2 m139. If a sec θ = x and b tan θ = y, then

how are x and y connected with aand b?(a) a2x2 –b2y2 = a2b2

(b) b2x2 – a2y2 = a2b2

(c) a2x2 + b2y2 = a2b2

(d) b2x2 + a2y2 = a2b2

140.tan tancot cot

A BA B

++

is equal to

(a) cot A cot B(b) sec A cosec B(c) tan A tan B(d) None of these

141. The angle subtended at the centreof a circle by an arc length equal inlength to the radius of the circle isdefined as(a) one radian(b) 90 degrees(c) one grade(d) one degree

142. If sin x + sin2 x = 1, then the valueof cos2 x + cos4 x is(a) 2(b) 2.5(c) 1(d) 1.5

143. The value of a sector in a pie-chartis equal to

(a)Angle of the sector

360° × 100

(b)Angle of the sector

360°× Total

value of the data

(c)Angle of the sector

180°× Total

value of the data

(d)Angle of the sector

180°× 100

144. In the figure, AB || CD and EFintersects them. Then, the value ofx is

E

A B

C D

F

2x

4x + 12°

(a) 18° (b) 14°(c) 28°(d) 24°

Directions (Q. Nos. 145-150) Study thefollowing chart and answer thequestions given below the chart.

145. If in the year 1998, the boxes wereexported at the same rate per boxas that in 1997, what would be thevalue (in crore of Rs.) of export in1998?(a) 330 (b) 400(c) 352 (d) 375

146. What is the difference betweenboxes (in lakh) exported in theyears 1996 and 1997?(a) 50(b) 25(c) 100(d) None of these

147. In which year was the value peraluminium box the minimum?(a) 1996(b) 1998(c) 1999(d) 1995

148. What is the difference (in lakh)between the boxes exported in theyears 1997 and 1998?(a) 100000(b) 1000000(c) 10(d) 1000

149. What was the aproximatepercentage increase in exportvalue from the years 1995 to 1999?(a) 375(b) 200(c) 333.3(d) None of these

150. What was the percentage drop inexport quantity from the years 1995to 1996?(a) 75(b) 25(c) 50(d) None of these

Page 63: Download SSC CGL Tier I Last 5 Year Papers e Book Www.sscportal.in (1)

1.9: Solved Paper (26-10-2014) (1st Sitting)

Join Online Coaching for SSC Exams:http://sscportal.in/community/courses

Buy Printed Study Material for SSC Exams:http://sscportal.in/community/study-kit

ENGLISH COMPREHENSIONDirections (Q.Nos. 151-155) In thefollowing questions, some parts of thesentence have errors and some arecorrect. Find out which part of asentence has an error. If a sentence isfree from error, mark (d) as your answer.151. With our great annoyance (a)/we

found the ground (b)/filled withbroken glasses. (c)/No error (d)

152. The new device (a)/aims ateliminating (b)/the risk of short-circuiting. (c)/No error (d)

153. I wish to heartily (a)/congratulateyou for (b)/your astoundingsuccess. (c)/No error (d)

154. The visitor took the vacant seat(a)/next from mine (b)/one of themany huge sofas in the room. (c)/No error (d)

155. He was (a)/a learnt man amonglords, (b)/and a lord amonglearned men. (c)/No error (d)

Directions (Q.Nos. 156-160) In thefollowing questions sentences aregiven with blanks to be filled in with anappropriate word (s). Four alternativesare suggested for each question.Choose the correct alternative out of thefour.156. The book provides an overview of

European wines that should proveinviting to both the virtual ........and the experienced connoisseur.(a) prodigal (b) novice(c) zealot (d) glutton

157. The Sun is shining brightly, please........... the light.(a) put off (b) put out(c) take off (d) put on

158. This practice was brought ........... toprevent certain abuses.(a) about (b) in(c) down (d) off

159. This package was supposed........... yesterday.(a) to have been delivered

(b) to deliver(c) to be delivering(d) not to be delivered

160. Her true feelings manifestedthemselves in her sarcasticremarks, only then was her ........revealed.(a) sweetness (b) bitterness(c) sarcasm (d) charm

Directions (Q.Nos. 161-163) In thefollowing questions out of the fouralternatives, choose the one which bestexpresses the meaning of the givenword.161. Brutalize

(a) Devise (b) Strike(c) Stir (d) III-treat

162. Pinnacle(a) Summit (b) Pit(c) Capusle (d) Heart

163. Garble(a) Communicate(b) Explain(c) Confuse(d) Hide

Directions (Q.Nos. 164-166) In thefollowing questions choose the wordopposite in meaning to the given word.164. Seamy

(a) Unpleasant(b) Sincere(c) Honest(d) Pure

165. Cease(a) Confront(b) Confiscate(c) Abandon(d) Initiate

166. Denounce(a) Grant(b) Praise(c) Signify(d) Confirm

Directions (Q.Nos. 167-171) In thefollowing questions four alternatives are

given for the Idiom/Phrase underlined inthe sentence. Choose the alternativewhich best expresses the meaning of theIdiom/Phrase.167. Ask Mr. Das. He can give you all

the news you need. He is alwaysin the swim.(a) in search of news(b) well-informed and upto date(c) giving news to others(d) at the swimming pool

168. You have been trying to solvethis puzzle for a long time, becareful, so that you avoid barkingup the wrong tree.(a) keep working constantly(b) climbing the wrong tree(c) trying to solve problems(d) trying to find something at a

wrong place169. I’ll be able to do my job very well

when I know the ropes.(a) learn the procedures(b) gain confidence(c) become bolder(d) work in silence

170. The secretary’s thoughtlessremark added fuel to the fire.(a) helped in improving the

situation(b) prevented the situation(c) added humour to the situation(d) worsened the difficult

situation171. He could not get money from his

master because he rubbed him upthe wrong way.(a) suggested a wrong method(b) advised him to be strict(c) Irked or irritated him(d) asked him to speed up

Directions (Q.Nos. 172-181) In thefollowing questions sentence /a part ofthe sentence is underlined. Below aregiven alternatives to the underlined partat (a), (b), (c) which may improve the

Page 64: Download SSC CGL Tier I Last 5 Year Papers e Book Www.sscportal.in (1)

1.10: Solved Paper (26-10-2014) (1st Sitting)

Join Online Coaching for SSC Exams:http://sscportal.in/community/courses

Buy Printed Study Material for SSC Exams:http://sscportal.in/community/study-kit

sentence. Choose the correctalternative. In case no improvement isneeded, your answer is (d).172. The government representative

furnished the reporters all details.(a) provided the reporters all

details(b) furnished reporters all details(c) furnished the reporters with

all the details(d) No improvement

173. It was raining so heavily yesterdaythat I could not move out for mayusual walk.(a) I could not go out for my

usual walk(b) I could not move for my usual

walk(c) I could not go to my usual

walk(d) No improvement

174. Ram filled ink into his pen beforeleaving for school.(a) filled ink in his pen(b) filled his pen with ink(c) filled ink on his pen(d) No improvement

175. The toys he bought for Suzy aretoo to be cheap.(a) are so much good to be cheap(b) are so good that to be cheap(c) were so good to be cheap(d) No improvement

176. I did one mistake in the dictationtest today.(a) I made a mistake in(b) I did a mistake in(c) I did a mistake at(d) No improvement

177. John has been detained at ameeting.(a) by the meeting(b) on a meeting(c) in meeting(d) No improvement

178. The butler was as devoted as afaithful dog.(a) a faithful cat(b) a faithful pet(c) a faithful friend(d) No improvement

179. He denied that he had not forgedmy signature.(a) would not forge(b) had forged(c) did not forge(d) No improvement

180. There is an error in grammar in thesentence.(a) a written error(b) a grammar error(c) a grammatical error(d) No improvement

181. To his astonishment andadmiratior, he got the informationthat it was only the picture of acurtain.(a) found(b) saw clearly(c) received the information(d) No improvement

Directions (Q.Nos. 182-188) In thefollowing questions out of the fouralternatives, choose the one which canbe substituted for the given words/sentence.182. Having a stale small or taste.

(a) Savoury(b) Tepid(c) Rancid(d) Insipid

183. A wooden object used forconnecting animals that arepulling a vehicle.(a) Whip(b) Yoke(c) Rein(d) Leash

184. A man who knows a lot aboutthings like food, music and art.(a) Hostage(b) Connoisseur(c) Priest(d) Optimist

185. A doctor who specializes indiseases of the nose.(a) Pathologist(b) Podiatrist(c) Rhinologist(d) Otologist

186. Study of caves.

(a) Topology(b) Numismatics(c) Speleology(d) Seismology

187. Government be the few.(a) Monarchy(b) Anarchy(c) Oligarchy(d) Autocracy

188. Materials that change naturally bythe action of bacteria.(a) Biological(b) Biodegradiable(c) Inflammable(d) Perishable

Directions (Q.Nos. 189 and 190) In thefollowing questions four words aregiven in each question, out of whichonly one word is correctly spelt. Findthe correctly spelt word.189. (a) Extirpate

(b) Extripaite(c) Exterpate(d) Exitrpeit

190. (a) Intelligibility(b) Inteligibility(c) Inteligibelity(d) Inteligibillity

Directions (Q.Nos. 191-200) You havetwo brief passages with five questionsfollowing each passage. Read thepassages carefully and choose the bestanswer to each question out of the fouralternatives.

Passage IA crucial element that defines the

soap opera is the open-ended nature ofthe narrative, with stories spanningseveral episodes. One of the definingfeatures that makes a television programa soap opera, according to AlbertMoran, is “that form of television thatworks with a continuous open narrative.Each episode ends with a promise thatthe storyline is to be continued inanother episode.”

In 2012, Robert. Lloyd of the LosAngeles Times wrote of daily dramas,“Although melodramatically eventful,

Page 65: Download SSC CGL Tier I Last 5 Year Papers e Book Www.sscportal.in (1)

1.11: Solved Paper (26-10-2014) (1st Sitting)

Join Online Coaching for SSC Exams:http://sscportal.in/community/courses

Buy Printed Study Material for SSC Exams:http://sscportal.in/community/study-kit

soap operas such as this also have aluxury of space that makes them seemmore naturalistic, indeed, the economicsof the form demand long scenes andconversations that a 22-episodes-per-season weekly series might dispensewith half a dozen lines of dialogue maybe drawn out, as here, for pages. Youspend more time even with the minorcharacters the apparent villains grow lessa apparently villainous.

Soap opera storylines runconcurrently, intersect and lead intofurther developments An individualepisode of a soap opera will generallyswitch between several differentconcurrent “.arrative threads that may attimes interconnect and affect oneanother or may run entirely independentof each other. Evening soap operas andserials that run for only a part of the yeartend to bring things to a dramatic end-of-season cliffhanger.191. What does the author mean ‘by

the open -ended nature of soapoperas?(a) Every episode ends shruptly(b) Consecutive episodes nave

no connection(c) Each episode ends with a

promise that the storyline is tobe continued in anotherepisode

(d) Every episode has a differentstory

192. A soap opera has the space for itto be more(a) dramatic(b) tragic(c) artistic(d) naturalistic

193. The economics of a soap operaform demands for it to have(a) melodramatic events(b) promising storylines(c) long scenes(d) luxurious space

194. An individual episode of a soapopera generally switches between(a) successive intersections of

events

(b) different concurrent narrativethreads

(c) more time spent with minorcharacters

(d) apparent villains that growless apparent villainous

195. Soap operas that run for a part ofthe year usually end in(a) a cliffhanger(b) a sequence(c) a cliff(d) an episode

Passage IITwo or three days and nights went

by, 1 reckon I might say they swam bythey slid along so quiet and smooth andlovely. Here is the way we put it in thetime. It was a monstrous big river downthere-sometimes a mile and a half widerwe ran rights, and laid up and hiddaytimes, soon as night was most gonewe stopped navigating and tied up nearlyalways in the dead water under atowhead and then cut young cottonwoods and willows and hid the roft withthem-Then, we set out the lines. Next weslid into the river and had a swim, so asto freshen up and cool, off then we setdown on the sandy bottom where thewater was about knee deep and watchedthe daylight come. Not a soundanywhere-perfectly shill just like thewhole word was asleep, only sometimes,the bullfrogs a cluttering maybe. Thefirst thing to see, looking away over thewater, was a kind of dull line- that was thewoods on other side, you couldn’t makenothing else out, then a pale place in thesky, then more paleness spreadingaround, then the river softened up awayoff and warm black any more, but grayyou could see little dark spots drifingalong ever so far away-trading scoweand such things and long black streaks-rafts sometimes you could hear a sweepcreaking, jumbled up voices, it was sostill and sounds comes so far and byand by you could see a streak on thewater which you know by the look of the

streak that there’s snag there in a currentwhich breaks on it and makes that streaklook that way.196. How did the days and nights go

by, according to the writer?(a) They slid along so smooth

and soft and quietly(b) They slid along so quietly and

smooth and softly(c) They slid along so quiet and

smooth and lovely(d) They slid along so smooth

and quietly197. They stopped navigating

(a) at daytime(b) at dawn(c) at night(d) at dusk

198. After a swim in the moor they(a) set down on the sandy shore

and watched the daylightcome

(b) set down on the sandybottom and watched thedaylight come

(c) set down on the sandybottom where the water wasabout ankle deep and watchedthe daylight come

(d) set down on the sandybottom where the water wasabout knee deep and watchedthe daylight come

199. In the stillness of the night(a) the whole world was asleep(b) a sweep creaking or jumbled

up voices could be heard(c) sound come so far(d) the bullfrogs also were asleep

200. The streak on the water looks as itdoes because(a) of a sang there in the swift

current which breaks on it(b) the streak has been swept by

the swift current(c) the swift current has broken

the streak(d) the streak has been swept by

the swift current to the shore

Page 66: Download SSC CGL Tier I Last 5 Year Papers e Book Www.sscportal.in (1)

1.12: Solved Paper (26-10-2014) (1st Sitting)

Join Online Coaching for SSC Exams:http://sscportal.in/community/courses

Buy Printed Study Material for SSC Exams:http://sscportal.in/community/study-kit

ANSWERS1. (c) 2. (a) 3. (*) 4. (c) 5. (c) 6. (c) 7. (d) 8. (b) 9. (c) 10. (b)

11. (b) 12. (b) 13. (d) 14. (d) 15. (d) 16. (c) 17. (d) 18. (b) 19. (c) 20. (b)21. (c) 22. (a) 23. (b) 24. (c) 25. (b) 26. (c) 27. (a) 28. (d) 29. (a) 30. (a)31. (d) 32. (b) 33. (c) 34. (d) 35. (d) 36. (a) 37. (d) 38. (b) 39. (b) 40. (a)41. (a) 42. (b) 43. (d) 44. (b) 45. (c) 46. (d) 47. (c) 48. (d) 49. (c) 50. (b)51. (c) 52. (c) 53. (b) 54. (a) 55. (a) 56. (b) 57. (a) 58. (a) 59. (c) 60. (d)61. (c) 62. (d) 63. (a) 64. (b) 65. (d) 66. (c) 67. (a) 68. (d) 69. (b) 70. (c)71. (c) 72. (a) 73. (d) 74. (b) 75. (d) 76. (d) 77. (b) 78. (a) 79. (d) 80. (d)81. (a) 82. (d) 83. (d) 84. (b) 85. (c) 86. (d) 87. (d) 88. (a) 89. (d) 90. (a)91. (d) 92. (a) 93. (a) 94. (c) 95. (c) 96. (a) 97. (b) 98. (d) 99. (d) 100. (c)

101. (a) 102. (c) 103. (a) 104. (c) 105. (a) 106. (c) 107. (c) 108. (c) 109. (d) 110. (a)111. (a) 112. (b) 113. (d) 114. (d) 115. (a) 116. (b) 117. (d) 118. (c) 119. (b) 120. (b)121. (d) 122. (b) 123. (b) 124. (c) 125. (b) 126. (c) 127. (a) 128. (b) 129. (b) 130. (c)131. (d) 132. (c) 133. (a) 134. (d) 135. (c) 136. (a) 137. (d) 138. (c) 139. (b) 140. (c)141. (a) 142. (c) 143. (b) 144. (c) 145. (c) 146. (d) 147. (d) 148. (c) 149. (c) 150. (b)151. (a) 152. (c) 153. (b) 154. (b) 155. (b) 156. (b) 157. (b) 158. (b) 159. (b) 160. (b)161. (d) 162. (a) 163. (c) 164. (d) 165. (d) 166. (b) 167. (b) 168. (d) 169. (a) 170. (d)171. (c) 172. (c) 173. (a) 174. (b) 175. (d) 176. (a) 177. (d) 178. (d) 179. (b) 180. (c)181. (a) 182. (c) 183. (b) 184. (b) 185. (c) 186. (c) 187. (c) 188. (b) 189. (a) 190. (a)191. (c) 192. (d) 193. (c) 194. (b) 195. (a) 196. (c) 197. (b) 198. (d) 199. (a) 200. (a)

EXPLANATIONS1. As, Optimist is always Cheerful, in

the same way Pessimist is alwaysGloomy.

2. As

Same as,

3. As,

Same as,

4. As,

Same as,

5. As, 225 : 256 → (15)2 : (15 + 1)2

Same as, 289 : 324→ (17)2 : (17 + 1) 2

6. As, Money can beMisappropriation, in the same wayWriting can be Plagiarism.

7. Except Ally, all other are harmfulfor us.

8. Except 35, all other are multiples of9.

9. Except 11-115, in all others optionsright side number can get bymultiplication of left number andone less from left number.

10. Except OPRS, all other areincreasing by + 1, + 1, + 2.

11. Cape is different from others.12. The meaningful order of the given

words is as followsSeed → Embryo → Germination →Plant → Flower → Fruit

13. The order of given words as perthe English dictionary is asfollowsLiteracy → Litter → Little → Live→ Living

14. ip/ig/up/og/ip/ig/up/og/ip/igHence, option (d) is the rightanswer.

Page 67: Download SSC CGL Tier I Last 5 Year Papers e Book Www.sscportal.in (1)

1.13: Solved Paper (26-10-2014) (1st Sitting)

Join Online Coaching for SSC Exams:http://sscportal.in/community/courses

Buy Printed Study Material for SSC Exams:http://sscportal.in/community/study-kit

15. A K G L M N D Q K G C S

N G K T G K G N D Z P U XG K EThus, in the given series two‘KGN’ occur in such a way that ‘G’is in the middle and ‘K’ and ‘N’ areadjacent to it on both sides.

16.

Therefore, in the above series, 7will be come in the place ofquestion mark.

17.

Hence, in the above series, OLCwill be come in the place ofquestion mark.

18. The sequence of given series is asfollows7 × 1 + 1 = 8⇒ 8 × 2 + 2 = 1818 × 3 + 3 = 51

⇒ 57 × 4 + 4 = 232232 × 5 + 5 = 1165Hence, in the series, 232 will become in the place of questionmark.

19. According to the question,Mohan > Kamal > Sachin > Ram

...(i)andKamal > Arun > Sachin ..(ii)So, from Eqs. (i) and (ii),Mohan > Kamal > Arun > Sachin

> Ram ...(iii)It is clear from Eq. (iii) that thesecond tallest is Kamal.

20. Making the relation diagram frominformation given,

Hence, it is clear from diagram thatC is wife of D.

21. ‘MARCY’ word can be formedusing the letters of the given mainword DEMOCRACY.

22. Due to absence of letter ‘L’ in thegiven main word, ‘RELATION’word cannot be formed.

23. Given, Mr. Mani’s present age= 47 yrand John’s present age = 13 yrLet, in x yr Mr. Mani’s age bedouble of John’s age.According to the question,47 + x = 2 (13 + x)⇒ 47 + x = 26 + 2x⇒ 2x – x = 47 – 26⇒ x = 21 yrHence, after 21 yr Mr. Mani’s agebe double of John’s age.

24. As,

Similarly,

25. As,C H A R A C T E R↓ ↓ ↓ ↓ ↓ ↓ ↓ ↓ ↓2 4 1 6 1 2 3 7 6andC H I L D R E N↓ ↓ ↓ ↓ ↓ ↓ ↓ ↓2 4 8 5 9 6 7 0Similarly,H I R A L A L↓ ↓ ↓ ↓ ↓ ↓ ↓4 8 6 1 5 1 5

26. According to the question,

27. Let total student appeared in anexam = xThen, 22% of x = 176

⇒22

100× x = 176

⇒ x=176 100

22×

= 8 × 100⇒ x= 800Now number of student got firstclass = 34% of 800

=34

100 × 800

= 34 × 8 = 27228. Meaningful word can be formed

from the given word is CODING.Sequence of this word is as belowC O D I N G3 → 4 → 5 → 2 → 6 → 1

29. Given equation, (8 * 7 * 6) * 5 * 10Now, replacing * sign with themathematical sign of option (a),

(8 × 7 – 6) ÷ 5 = 10⇒ (56 – 6) ÷ 5 = 10⇒ 50 ÷ 5 = 10⇒ 10 = 10Hence, option (a) is the rightanswer.

30. After changing the sign ofequation given in option (a) as perthe question,

10 – 3 + 5 + 4 = 16⇒ 7 + 5 + 4 = 16⇒ 16 = 16Hence, option (a) is the rithanswer.

31. As, 5 * 3 = 125⇒ (5)3 = 125and 4 * 3 = 64⇒ (4)3 = 64Same as, 8 * 2 = ?

⇒ (8)2 = 64∴ ? = 64

32. As, 9 × 6 – 5 = 49and 7 × 8 – 4 = 52Same as 4 × ? – 3 = 25⇒ 4 × ? = 25 + 3 = 28

⇒ ? =284

= 7

? = 7

Page 68: Download SSC CGL Tier I Last 5 Year Papers e Book Www.sscportal.in (1)

1.14: Solved Paper (26-10-2014) (1st Sitting)

Join Online Coaching for SSC Exams:http://sscportal.in/community/courses

Buy Printed Study Material for SSC Exams:http://sscportal.in/community/study-kit

33. As, 4 + 22 – 10 = 16and 8 + 14 – 5 = 17Same as,

10 + ? – 3 = 18⇒ 7 + ? = 18⇒ ? = 18 – 7 = 11

⇒ ? = 1134. As, (2)2 + (3)2 = 4 + 9 = 13

and (4)2 + (5)2 = 16 + 25 = 41Same as,

(3)2 + (4)2 = 9 + 16 = 25

35.

Files

Papers

Books Journals

It is clear from the diagram thatonly Conclusion II follows.

36. Walking diagram of Pratap is asfollow

7 km

4 km3 km

2 kmN

S

EW

It is clear from diagram that he isfacing South direction now.

37. Walking diagram of Vijay is asfollow.

N

S

EW

10 km

10 km

12 km12 km

D

C

A

B

Point A is the starting point and Dis the ending point of walking ofVijay.Now, Vijay’s distance from startingpoint = AD = BC = 10 km

38. As per the conclusion we mayconclude that ....... there is a steadyincrease of the fertilizerconsumption.

39. A

G

D

B

H

CF

E

J

K

I

L

O

Number of such triangles are asfollows∆AGI, ∆AEI, ∆EBJ, ∆BHJ, ∆HCK,∆CFK, ∆FDL, ∆DLG, ∆OGE, ∆OEH,∆OHF, ∆GAE, ∆EBH, ∆HCF, ∆FDG,∆FGE, ∆GEH, ∆EHF, ∆HFG = 20

40. Answer figure (a) can be formedfrom the cut-pieces given in thequestion figure.

41. Answer figure (a) will be formedon folding the question figure.

42. Letter Q represents those Actorswho are also Dancers, Singers aswell as Violinists.

43. Machinery comes under the groupof Factory and Product can beformed by the Machinery.

Product

MachineryFactory

44. Both Train and Bus are differenttypes of transport medium used byTravellers.

Travellers

Train Bus

45. Answer figure (c) will complete thepattern of the question figure.

46. Answer figure (d) will complete thepattern of the question figure.

47. Answer figure (c) will appear whena piece of paper is folded and cutas shown in the question figure.

48. Question figure is hidden/embedded in the answer figure (d).

49. C → 04, 88,O → 11, 30, 57, 78, 89,S → 14, 23, 41, 69, 99,T → 31, 33, 76, 96∴ COST → 04, 11, 41, 76

50. Answer figure (b) is the correctmirror image of the questionfigure.

101. Number of pages copied by

Subhash in 1 h = 5010

= 5 and

number of pages copied bySubhash and Prakash in 1 h

= 30040

= 152

Subhash + Prakash can copy 15/2Pages in 1 hour. So Prakash can

copy = 15 52

− = 52

pages is 1

hour.Therefor Prakash can copy 30pages in 12 hour

102. Smallest number of boxes for bunsalone

= LCM of 10 and 8

Number of buns in a box

= 4010

= 4

103.24

2

–11

–2x

x

+ =

4 2

4

( –1)1

4x

x+

= 4 8 4

4

4 1– 24

x x xx

+ +

= 4 2

2 2

( 1)(2 )x

x+

= 4

21

2x

x

+

= 4

2

12

xx+

104. Men 270180

Days10x

270180

=10x

Page 69: Download SSC CGL Tier I Last 5 Year Papers e Book Www.sscportal.in (1)

1.15: Solved Paper (26-10-2014) (1st Sitting)

Join Online Coaching for SSC Exams:http://sscportal.in/community/courses

Buy Printed Study Material for SSC Exams:http://sscportal.in/community/study-kit

⇒ x × 180 = 270 × 10

=270 10

180×

= 15

⇒ x= 15 days105. Let, men in beginning = x

Men x

(x–10)

Days100110

∴–10x

x=

110100

⇒ 100x = 110x – 1100⇒ 110x – 100x = 1100⇒ 10x = 1100⇒ x= 110

106. Let each side of the square lawn= x mThen, according to the question,⇒ (x + 4 + 4)2 – (x)2 = 196⇒ (x + 8)2 – x2 = 196⇒ x2 + 64 + 16x – x2 = 196

x4

(x+4+4)

⇒ 16x = 196 – 64 = 132

⇒ x = 13216

= 8.25

⇒ x = 8.25 m

107. Rebate on one shirt = 32 × 25

100= Rs. 8Now, rebate of Rs. 8 gets = 1 shirt

∴ Rebate of Rs. 1 gets = 18

shirts

∴ Rebate of Rs. 40 gets = 18

× 40

= 5 shirts108. Cost price of 1 dozen pairs of

socks.

= 160 – 160 10

100×

= 160 – 16

= Rs. 144

Now, cost of 1 pair of socks = 14412

= Rs. 12Q Number of socks obtain inRs. 12 = 1 pair∴ Number of socks obtain in

Rs. 1 = 1

12 pair

∴ Number of socks obtain in

Rs. 60 = 1

12× 60 pair = 5 pairs

109. Selling price of machine

= 7500 × 100 – 8

100 ×

100 – 5100

× 100 – 2

100

= 7500 × 92

100 ×

95100

× 98

100

= 3 92 95 98

100 4× × ×

×

= 3 23 95 98

100× × ×

= 642390

100= Rs. 642390

110. Given,

Surface area of first sphereSurface area of second sphere

= 2

12

2

44

rr

ππ =

49

2

1

2

rr

=

49

⇒1

2

rr =

23

...(i)

Now, Volume of first sphere

Volume of second sphere

=

31

32

4343

r

r

π

π =

3

1

2

rr

⇒1

2

VV =

323

[From Eq. (i)]

⇒1

2

VV =

827

⇒ V1 : V2 = 8 : 27111. Let, diameter of circle = d

Perimeter = 2πr = π2d [Qd = 2r]

Then, new diameter = d + 75

100d ×

= d + 34d

= 74d

Now, new perimeter = π × 74d

= 7

4dπ

∴ Increase in perimeter

= 7

4dπ

– πd = 7

–4d

= 34d

π

= 3

4dπ

= πd × 34

Now, percentage increase in

perimeter =

34

d

d

π

π

× × 100%

= 34

× 100% = 75%

112. We know that perimeter ofequilateral triangle = 3 × side⇒ 42 = 3 × side

⇒ Side =423

= 14

⇒ Side = 14 cmNow, area of equilateral triangle

= 34

× (side)2

= 3

4 × (14)2 =

34

× 14 × 14

= 49 3 cm2

Page 70: Download SSC CGL Tier I Last 5 Year Papers e Book Www.sscportal.in (1)

1.16: Solved Paper (26-10-2014) (1st Sitting)

Join Online Coaching for SSC Exams:http://sscportal.in/community/courses

Buy Printed Study Material for SSC Exams:http://sscportal.in/community/study-kit

113. Given, x + 1x

= 1

∴ 2

2

3 17 1

x xx x

+ ++ +

=

13

17

x xx

x xx

+ + + +

=

1 3

1 7

xx

xx

+ +

+ + =

1 31 7

++

11x

x + = Q

= 48

= 12

114. Given, a + b + c = 0∴ a + b = – c

b + c = – ac + a = – b

∴ 3

1a

+ 3

1b

+ 3

1c

+ 3

1( )a b+ +

3

1( )b c+ + 3

1( )c a+

= 3

1a

+ 3

1b

+ 3

1c

+ 3

1(– )c +

3

1(– )a + 3

1(– )b

= 3

1a

+ 3

1b

+ 3

1c

– 3

1c

– 3

1a

3

1b

= 0

115. Given, x + y = 4 ...(i)

and1x

+ 1y = 4

⇒y xxy+

= 4

⇒ x + y = 4xy⇒ 4 = 4xy [From eq. (i)]⇒ xy = 1 ...(iii)

We know that,x3 + y3 = (x + y) (x2 – xy + y2)x3 + y3 = (x + y) [(x2 + y2) – xy]= (x + y) [x2 + y2 - xy + 2xy - 2xy]= (x + y) [x2 + y2 + 2xy - 3xy]= (x + y) [(x + y)2 - 3xy]= (x + y) [x + y)2 - 2xy - xy]= (x + y) [x + y)2 - 3xy]= (4) [(4)2 - 3 × 1]= 4 (16 - 3) = 4 × 13 = 52

116. Given, (a2 + 2a)2 + 12(a2 + 2a) – 45Let, a2 + 2a = xx2 + 12x – 45 = x2 + 15x – 3x – 45= x (x + 15) – 3(x + 15)= (x + 15) (x – 3)Now, putting the value of x inthese factors.= (a2 + 2a + 15) (a2 + 2a – 3)= (a2 + 2a – 3) (a2 + 2a + 15)= (a2 + 3a – a – 3) (a2 + 2a + 15)= [a(a + 3) – 1(a + 3)] (a2 + 2a + 15)= (a + 3) (a – 1) (a2 + 2a + 15)

117. Let number of passengers in firstclass be x and number ofpassengers in second class be 40x.Then, total amount of first class= 4xand total amount of second class= 40x∴ Ratio of the amounts collectedfrom the first class and the secondclass passengers = 4 : 40∴ Amount collected from the firstclass passengers

= a

a b+ × 11000

Where, a = 4 and b = 40

= 4

4 40+ × 11000 =

4 1100044

×

= Rs. 1000118. Let, ages of all three boys are 3x,

5x and 7x respectively.According to the question,

3 5 73

x x x+ + = 15

⇒ 15x = 45⇒ x = 3∴ Age of youngest boy = 3x

= 3 × 3 = 9 yr119. Let, third number = x

Then, second number = 3x andfirst number = 6xNow, according to the question,

6 33

x x x+ + = 10

⇒ 10

3x

= 10

⇒ x = 3∴ Largest number = 6x = 6 × 3 = 18

120. Total CP of car = 135000 + 25000= Rs. 160000 and Loss = 10%Now,

SP of car = CP × (100 – Loss percent)

100

= 160000 × (100 –10)

100 = 1600 × 90

= Rs. 144000121. Let, total number of candidates = x

candidate passed in English= 77%∴ Candidates failed in English= (100 – 77)% = 23%and candidates failed inMathematics = 34%Now, candidate failed in both thesubjects = 13%∴ candidates passed in both thesubjects= {100 – (23 + 34 + 13)}%= (100 – 44)% = 56%Now, it is given that 56% of x = 784

⇒ 56

100x ×

= 784

⇒ 784 100

56×

= 14 × 100

⇒ x = 1400Hence Total number of candidates= 1400

122. Here, Distance = Length of train= 120 m and speed of train= 54 km/h

= 54 × 5

18 m/s = 15 m/s

∴ Time taken by train to pass a

Page 71: Download SSC CGL Tier I Last 5 Year Papers e Book Www.sscportal.in (1)

1.17: Solved Paper (26-10-2014) (1st Sitting)

Join Online Coaching for SSC Exams:http://sscportal.in/community/courses

Buy Printed Study Material for SSC Exams:http://sscportal.in/community/study-kit

man standing on the plateform

= DistanceSpeed =

120 m15 m/ s = 8 s

123. Time taken by A = 305

h, Time

taken by B = 307

h

Time taken by C = 308

h

Time to come to all together

= LCM of 305

, 307

and 308

= LCM of 30, 30, 30

HCF of 5, 7, 8 = 30 h

124. Given, P = Rs. 1000, t = 2 yr,A = Rs. 110250, r = ?

By A = 1100

trP +

⇒ 110250 = 1000 2

1100

rP +

⇒ 1102501000

= 2100

100r+

⇒ 1102510000

= 2100

100r+

⇒ 2105

100 =

2100100

r+

⇒ 105100

= 100

100r+

⇒ 105 = 100 + r⇒ r = 105 – 100 = 5%⇒ r = 5%

125. From figure,

A B C6.3

3.6

AC = 6.3 cm, BC = 3.6 cmNow, distance between centre ofboth the circles = AB= AC – BC = 6.3 – 3.6 = 2.7 cm

126. In figures, OM ⊥ AB, radiusOC = 5 cmand perpendicular distance fromcentre to chord AB = 3 cm

A BM4 cm

5 cm

3 cm

5 cm

C

O

Now, OA = OC = 5 cmQ Point M, bisect chord AB∴ AB = 2 AMNow, In ∆OAM,(OA)2 = (AM)2 + (OM)2

[By Pythagoras Theorem]⇒ (AM)2 = (OA)2 – (OM)2

⇒ (AM)2 = (5)2 – (3)2

= 25 – 9 = 16⇒ (AM)2 = (4)2

⇒ AM = 4 cmNow, length of chordAB = 2 × AM = (2 × 4) cm = 8 cm

127. Given, ∠A – ∠B = 20° ...(i)and ∠B – ∠C = 32° ...(ii)We know that,∠A + ∠B + ∠C = 180° ...(iii)Now, solving to Eqs. (i), (ii) and (iii)We get ∠A = 84°

128. From figure,AE = AD + DE = 5 + 3 = 8 cm∠ABC = ∠CEDNow, In ∆ADB and ∆AEC,ABAE

= ADAC

[Similar

Triangles]A

CB D

5 cm

4 cm

3 cm

⇒ 8

AB =

54

⇒ AB = 8 5

= 10

⇒ AB = 10 cm

129. A

B C D

P

Q R S

In figure, ∆ABC and ∆PQR aresimilar triangle.Here, BC = QR and AB = PQand ∠B = ∠Q = 120°Hence, both triangle are congruentby the condition of Angle-Side-Side.

130. Since, given ∆ABC is anequilateral triangle.

A

B C

O

So, angle formed by incentre isalso equal.∴ ∠BOC, ∠COA and ∠AOB isequal to 60°.

131. Given, x = 3 + 2 2

∴ x3 = (3 + 2 2 )3 = 33 + (2 2 )3

+ 3.3.2 2 (3 + 2 2 )

= 27 + 16 2 + 18 2 (3 + 2 2 )

= 27 + 16 2 + 54 2 + 72

= (99 + 70 2 )

Now, 3

1x

= 1

(99 70 2)+

= (99 – 70 2)

(99 70 2)(99 – 70 2)+

= 2 2

(99 – 70 2)(99) – (70 2)

Page 72: Download SSC CGL Tier I Last 5 Year Papers e Book Www.sscportal.in (1)

1.18: Solved Paper (26-10-2014) (1st Sitting)

Join Online Coaching for SSC Exams:http://sscportal.in/community/courses

Buy Printed Study Material for SSC Exams:http://sscportal.in/community/study-kit

= 99 – 70 2

9801– 9800 = 99 – 70 2

∴ x3 + 3

1x

= (99 + 70 2 ) + (99 –

70 2 )

= 99 + 70 2 + 99 – 70 2 = 198

and x3 – 3

1x

= (99 + 70 2 ) – (99

– 2 )

= 99 + 70 2 – 99 + 70 2= 140 2

132. (a + b)2 + (b + c)2 + (c + a)2

= a2 + b2 + 2ab + b2 + c2 + 2bc +c2 + a2 + 2ca

= a2 + b2 + c2 + 2ab + 2bc + 2ca +a2 + b2 + c2

= (a + b + c)2 + a2 + b2 + c2

= 2(a + b + c)2 – 2(ab + bc + ca)= 2(a + b + c)2 – 2(ab + bc + ca)= 2 × (2)2 – 2 × (1) = 8 – 2 = 6

133. Q x2 + 4x + 3 = 0∴ x2 + 3x + x + 3 = 0⇒ x(x + 3) + 1(x + 3) = 0⇒ (x + 3)(x + 1) = 0⇒ x = – 3 and – 1s

Now, 3

6 327 27x

x x+ +

= 3

3 33

2727

x

x xx

+ +

= 3

3

12727xx

+ +

Since, we have x = – 3 and x = – 1So, putting x = – 3, we get

= 3

3

127(–3) 27

(–3)+ +

= 1

27–27 27 –27

+ =

1–1

= – 1

= putting x = 1

= 33

127

( 1) 27( 1)

− + +−

= 1

1 27 27− + − = –1

136.1 sin1– sin

θθ

+ =

(1 sin )(1 sin )

(1 – sin )(1 sin )

θ θ

θ θ

+ +

+

= 2

2

(1 sin )1 – sin

θθ

+ = 2

(1 sin )cos

θθ

+

= (1 sin )

cosθ

θ+

= 1

cosθ +

sincos

θθ

= sec θ + tan θ137. Given,

AB

C

3

4

5

tan θ = 34

= BCAB

Now, in ∆ABC(AC)2 = (AB)2 + (BC)2

(AC)2 = (4)2 + (3)2 = 16 + 9 = 25AC = 25 = 5

Again in ∆ABC, sin θ = 35

= BCAC

and cos θ = 45

= ABAC

Now, 25x sin2 θ cos θ = tan2 θ

⇒ 25x . 23

5 ×

45

= 23

4

⇒ 25x × 925

× 45

= 9

16

⇒ 9 45

x × × =

916

⇒ x = 5 9

9 4 16×

× × =

564

138. Let, length of string = x m

Then, in ∆ABC,A

BC60°

45 mx

sin 60° = ABAC

= 45x

⇒ 3

2 =

45x

x = 2 45

⇒ x = 2 45 3

3× ×

= 30 3

⇒ x = 30 3 m

Hence, length of string is 30 3 m.139. Taking option (b), which is

b2x2 – a2y2 = a2b2 ...(i)Now, putting the value of x and yin Eq. (i)⇒ b2(a sec θ)2 – a2(b tan θ)2

= a2b2

⇒ b2a2 sec2 θ – a2b2 tan2 θ = a2b2

⇒ a2b2 (sec2 θ – tan2 θ) = a2b2

⇒ a2b2 = a2b2 [Qsec2θ– tan2 θ = 1]

Hence, option (b) is the rightanswer.

140.tan tancot cot

A BA B

++

= tan tan

1 1tan tan

A B

A B

+

+

= tan tantan tantan tan

A BB AA B

++⋅

= tan A . tan B

141. The length subtended at thecentre of a circle by an arc lengthequal in length to the radius of thecircle is defined as one radian.

142. Q sin x + sin2 x = 1∴ sin x = 1 – sin2 x = cos2 x ...(i)Now, cos2 x + cos4 x = sin x + cos4x

Page 73: Download SSC CGL Tier I Last 5 Year Papers e Book Www.sscportal.in (1)

1.19: Solved Paper (26-10-2014) (1st Sitting)

Join Online Coaching for SSC Exams:http://sscportal.in/community/courses

Buy Printed Study Material for SSC Exams:http://sscportal.in/community/study-kit

= sin x + sin2 x [From eq. (i)]= 1

143. The value of a sector in a pie chart

= 2

360rπ θ

°

= Angle of the sector

360°× Total

value of the data144. From figure,

∠EOB = ∠EMD = 2x°(Corresponding angle)

E

BA

DC

F

4x+12°

2x°

M

Now, ∠EMD + ∠EMC = 180°∴ 4x + 12° + 2x = 180°⇒ 6x + 12° = 180°⇒ 6 x = 180° – 12°⇒ 6 x = 168°

⇒ x=168

= 28°

⇒ x= 28°145. Per box export rate in 1997

= 330150

= 2.2 lakh

Now, boxes exported in the year1998 = 160∴ Export value in 1998= 160 × 2.2 = 352 crore

146. Number of boxes exported in year1997 = 150and number of boxes exported inyear 1996 = 75∴ Required difference= 150 – 75 lakh

147. Per box value in 1995 = 150100

= Rs. 1.5 lakh

Per box value in 1996 = 15075

= Rs. 2 lakh

Per box value in 1997 = 330150

= Rs. 2.2 lakh

Per box value in 1998 = 400160

= Rs. 2.5 lakh

Per box value in 1999 = 500200

= Rs. 2.5 lakhSo, it is clear from above that the

value of per box was minimum inthe year 1995.

148. Boxes exported in the year 1998= 160 lakhBoxes exported in the year 1997= 150 lakh∴ Required difference= (160 – 150) lakh = 10 lakh

149. Export value in 1995= Rs. 150 croreExport value in 1999= Rs. 500 crore∴ Required percentage increase

= 500 –150

150 × 100%

= 350150

× 100% = 333.33%

150. Quantity of export in 1995 = 100lakh boxQuantity of export in 1996 = 75lakh box∴ Required percentage drop inexport quantity

= 75 –100

100 × 100%

= –25100

× 100% = – 25%

Here, negative sign show thepercentage drop in export quantity.

Page 74: Download SSC CGL Tier I Last 5 Year Papers e Book Www.sscportal.in (1)

SSC CGL Exams Online Coaching

(Starting at just Rs.100 per month) - English & Hindi

Medium

Online Crash Course for SSC CGL Tier-1 Exam

http://sscportal.in/community/courses/crash-course-for-cgl-tier-1

SSC CGL (Tier-1) http://sscportal.in/community/courses/ssc-cgl-tier-1-hindi

For More Information Click Given below link:

http://sscportal.in/community/courses

Page 75: Download SSC CGL Tier I Last 5 Year Papers e Book Www.sscportal.in (1)

Downloaded From: http://sscportal.in

Downloaded From: http://sscportal.in

Page 76: Download SSC CGL Tier I Last 5 Year Papers e Book Www.sscportal.in (1)

Downloaded From: http://sscportal.in

Downloaded From: http://sscportal.in

Page 77: Download SSC CGL Tier I Last 5 Year Papers e Book Www.sscportal.in (1)

Downloaded From: http://sscportal.in

Downloaded From: http://sscportal.in

Page 78: Download SSC CGL Tier I Last 5 Year Papers e Book Www.sscportal.in (1)

Downloaded From: http://sscportal.in

Downloaded From: http://sscportal.in

Page 79: Download SSC CGL Tier I Last 5 Year Papers e Book Www.sscportal.in (1)

Downloaded From: http://sscportal.in

Downloaded From: http://sscportal.in

Page 80: Download SSC CGL Tier I Last 5 Year Papers e Book Www.sscportal.in (1)

Downloaded From: http://sscportal.in

Downloaded From: http://sscportal.in

Page 81: Download SSC CGL Tier I Last 5 Year Papers e Book Www.sscportal.in (1)

Downloaded From: http://sscportal.in

Downloaded From: http://sscportal.in

Page 82: Download SSC CGL Tier I Last 5 Year Papers e Book Www.sscportal.in (1)

Downloaded From: http://sscportal.in

Downloaded From: http://sscportal.in

Page 83: Download SSC CGL Tier I Last 5 Year Papers e Book Www.sscportal.in (1)

Downloaded From: http://sscportal.in

Downloaded From: http://sscportal.in

Page 84: Download SSC CGL Tier I Last 5 Year Papers e Book Www.sscportal.in (1)

Downloaded From: http://sscportal.in

Downloaded From: http://sscportal.in

Page 85: Download SSC CGL Tier I Last 5 Year Papers e Book Www.sscportal.in (1)

Study Kits for Staff Selection Commission Exams

Study Kit for SSC CGL EXAM (Tier-I)

http://sscportal.in/community/study-kit/cgl

Study Kit for SSC Combined Graduate Level Examination (Tier - II)

http://sscportal.in/community/study-kit/cgl-tier-2

Study Kit for Combined Higher Secondary Level (10+2) Examination

http://sscportal.in/community/study-kit/chsle

आ ( :-1) http://sscportal.in/community/study-kit/cgl/tier-1-hindi

Study Kit For SSC MTS Examination

http://sscportal.in/community/study-kit/mts

Study Kit For Prasar Bharti Examination

http://sscportal.in/community/study-kit/prasar-bharti-exam

Study Kit for Central Armed Police Forces (CAPFs) Examination

http://sscportal.in/community/study-kit/capfs

Study Kit For Sub Inspector in Delhi Police & CAPFs, CISF (Paper-1)

http://sscportal.in/community/study-kit/si-delhi-police-capf

Study Kit of English Language & Comprehension For All SSC Exam

http://sscportal.in/community/study-kit/english-language-comprehension-for-all-

ssc-exam

Study Kit For Postal Assistant Examination

http://sscportal.in/community/study-kit/postal-assistant

For More Information Click Given below link:

http://sscportal.in/community/study-kit

Page 86: Download SSC CGL Tier I Last 5 Year Papers e Book Www.sscportal.in (1)

Downloaded From: http://sscportal.in

Downloaded From: http://sscportal.in

Page 87: Download SSC CGL Tier I Last 5 Year Papers e Book Www.sscportal.in (1)

Downloaded From: http://sscportal.in

Downloaded From: http://sscportal.in

Page 88: Download SSC CGL Tier I Last 5 Year Papers e Book Www.sscportal.in (1)

Downloaded From: http://sscportal.in

Downloaded From: http://sscportal.in

Page 89: Download SSC CGL Tier I Last 5 Year Papers e Book Www.sscportal.in (1)

Downloaded From: http://sscportal.in

Downloaded From: http://sscportal.in

Page 90: Download SSC CGL Tier I Last 5 Year Papers e Book Www.sscportal.in (1)

Downloaded From: http://sscportal.in

Downloaded From: http://sscportal.in

Page 91: Download SSC CGL Tier I Last 5 Year Papers e Book Www.sscportal.in (1)

Downloaded From: http://sscportal.in

Downloaded From: http://sscportal.in

Page 92: Download SSC CGL Tier I Last 5 Year Papers e Book Www.sscportal.in (1)

Downloaded From: http://sscportal.in

Downloaded From: http://sscportal.in

Page 93: Download SSC CGL Tier I Last 5 Year Papers e Book Www.sscportal.in (1)

Downloaded From: http://sscportal.in

Downloaded From: http://sscportal.in

Page 94: Download SSC CGL Tier I Last 5 Year Papers e Book Www.sscportal.in (1)

Downloaded From: http://sscportal.in

Downloaded From: http://sscportal.in

Page 95: Download SSC CGL Tier I Last 5 Year Papers e Book Www.sscportal.in (1)

Downloaded From: http://sscportal.in

Downloaded From: http://sscportal.in

Page 96: Download SSC CGL Tier I Last 5 Year Papers e Book Www.sscportal.in (1)

SSC CGL Exams Online Coaching

(Starting at just Rs.100 per month) - English & Hindi

Medium

Online Crash Course for SSC CGL Tier-1 Exam

http://sscportal.in/community/courses/crash-course-for-cgl-tier-1

SSC CGL (Tier-1) http://sscportal.in/community/courses/ssc-cgl-tier-1-hindi

For More Information Click Given below link:

http://sscportal.in/community/courses

Page 97: Download SSC CGL Tier I Last 5 Year Papers e Book Www.sscportal.in (1)

General Awareness Magazine for SSC CGL Exam

Various Categories:

National

International

Economy

India And The World

Sports

In The News

Science and Technology

Burning Issues (Editorials From Different Newspapers)

What You Will Get:

You will get (12 Issues) PDF Only no Hard Copy

For More Information Click Given below link:

http://sscportal.in/community/general-awareness-magazine